KSAP VOL 3 MOD 1 UPDATE Flashcards

1
Q

A 54-year-old man with ESRD is seen in follow-up. He performs home HD 4 times weekly using a machine that utilizes low dialysate volume.
His current prescription specifies a blood flow of 400 mL/min, dialysate flow of 160 mL/min, with a flow fraction of 40%. He uses 25 L of dialysate per treatment session. His residual creatinine clearance is 2.59 mL/min, a decline from 3 months ago. His standardized weekly Kt/V dialysis is 1.34, and his standardized weekly Kt/V renal is 0.56, for a total of 1.9. His treatment time has ranged from 115 to 162 minutes per session.
On exam, he appears well. His BP is 126/82 mm Hg and his heart rate is 84/min. There is no lower extremity edema.
Which of the following is the BEST strategy to improve his standardized weekly Kt/V dialysis?
Select one:
A. Increase flow fraction from 40% to 50%
B. Increase blood flow to 450 mL/min
C. Increase dialysate volume to 30 L per treatment
D. Decrease blood flow rate but maintain flow fraction of 40%
E. Reduce target weight to reduce his volume of distribution for urea

A

The correct answer is: C
Increase dialysate volume to 30 L per treatment

The best strategy to improve the standardized weekly Kt/V in this patient who is being treated with a low dialysate volume hemodialysis platform is to increase the dialysate volume per treatment.
Home HD with a low volume dialysate platform can be an attractive alternative to other strategies for dialysis. Unlike conventional HD, this method does not require special home electrical or plumbing modifications. Optimization of dialysis using this modality requires additional knowledge of the kinetics and the unique features of this platform.
Although current dialysis guidelines do not address the low dialysate volume approach, most caregivers target a standardized weekly Kt/V urea of 2.0 per week or 0.6 per treatment.
There are several components of the prescription for low volume dialysate approach to consider:
1. The volume of dialysate is the chief determinate of dialysis dose. The initial prescription begins with an estimate of the volume of distribution of urea in the patient, which is similar to the volume of distribution of water. This volume is multiplied by the target of 0.6 to provide an initial volume of dialysate, which it is assumed will achieve 100% urea saturation. Thus, rather than prescribing a treatment duration, a volume of dialysate is prescribed.
2. The dialysate can be either a prepackaged fluid, similar to that used for PD, or generated with a self-contained system. Lactate is used as the buffer because bicarbonate cannot be stored with calcium or magnesium due to precipitation.
3. The low volume dialysate approach depends on nearly complete saturation of the dialysate by urea. This is achieved by maintaining a low ratio of the dialysate flow rate (Qd) to blood flow rate (Qb). This ratio is called the flow fraction (FF), which is calculated as Qd/Qb. A decrease in the dialysate rate will lower the flow fraction but maintain the volume of dialysate. This would result in an increase in the time required to deliver the dialysis. However, it would not increase the clearance because the volume of dialysate, which is essentially 100% saturated, is fixed. This is analogous to PD, in which it is necessary to increase the effluent volume in order to increase the delivered dialysis. According to data from NxStage Medical, the company that manufactures and distributes the most widely used low dialysate platform in the United States, dialysate is 90% saturated at a FF of 40% and 95% saturated at a FF of 20%.
4. If the ultrafiltration (UF) is significant, the FF calculation is expanded to (Qd+UF)/Qb. Increases in UF will contribute to clearance, again analogous to PD, but this effect will not be as great as an increase in dialysate volume unless the UF increase is substantial.
5. Unlike conventional HD, the clearance of urea is dependent on dialysate total effluent volume (dialysate + UF) rather than Qb and dialysis time.
Applying these fundamental principles, only an increase in dialysate volume for each treatment will increase the Kt/V in this patient.
Decreasing the blood flow rate but maintaining the FF will slightly decrease the rate of dialysate consumption and increase the treatment time. This is a reasonable strategy for a patient with poor tolerance of UF because it also extends the time for fluid removal, which may be better tolerated hemodynamically. However, it does not substantially affect the clearance of urea because the dialysate saturation remains approximately 90% and the dialysate volume is unchanged, thus the net urea removal is unchanged.
An increase in FF will decrease the treatment time and decrease the saturation of dialysate, thereby decreasing the clearance of urea.
An increase in Qb will require a corresponding increase in Qd if the FF remains fixed. This will shorten treatment time, but will not substantially change the clearance because the total effluent volume and saturation are unchanged. Shorter treatment may be desirable if UF goals can be met but there is little knowledge of the impact of treatment time on clearance of substances other than urea.
Reducing the target weight can potentially impact the clearance of urea in two ways: it can reduce the volume of distribution for urea (V) and it can potentially increase the UF volume per treatment, which increases the effluent volume. However, the numerical impact of these effects is likely to be small unless a major change is undertaken. The target weight should be modified to maintain euvolemia, not to address clearance problems. This patient appears euvolemic on exam and has a normal BP; therefore, a change in target weight would be ill-advised.

How well did you know this?
1
Not at all
2
3
4
5
Perfectly
2
Q

A 26-year-old woman presents to the nephrology clinic for evaluation as a potential kidney donor to her father, who has ESRD due to autosomal dominant polycystic kidney disease (ADPKD). She is well and takes no medications. Her father began dialysis at age 49. A paternal cousin also carries the diagnosis of ADPKD and hypertension but has normal kidney function at age 35. No other family members are known to have ADPKD.
The potential donor appears healthy and has a normal physical examination, including BP. Serum creatinine is 0.8 mg/dL and the urinalysis is normal. Ultrasonography of the kidneys reveals one cyst in each kidney.
Which of the following is strategies is best regarding this prospective kidney donor and recipient pair?
Select one:
A.
Proceed with kidney donation

B.
The family should undergo genetic linkage analysis
C.
The father should undergo direct mutation analysis

D.
The daughter should undergo direct mutation analysis

A

The correct answer is: C
The father should undergo direct mutation analysis

For this prospective kidney donor with a family history of ADPKD, the best approach would be to perform direct mutation analysis for PKD1 and PKD2 mutations in her father. Depending on results, the prospective donor could then undergo direct mutation testing to see if she carries the same (identified) mutation. Linkage analysis for PKD mutations requires at least 3 affected family members across at least 2 generations, and would not be an option in this case.
A family history of ESRD before age 58 in at least one family member is highly predictive of a PKD1 genotype, whereas a family history of ESRD after the age of 68 is highly predictive of a PKD2 genotype. Thus, a PKD1 mutation is likely in this family. The kidney ultrasound finding of only 1 cyst in each kidney is not sufficient to exclude a diagnosis of ADPKD at this age because a negative ultrasound (no cysts) in people younger than 30 is only 94% sensitive for ADPKD due to a PKD1 mutation (and only 70% sensitive for a PKD2 mutation). Therefore, it would not be appropriate to proceed with further workup or donation until a more definitive conclusion can be made regarding the potential donor’s PKD status. Because there can be nonpathogenic mutations in PKD1 or PKD2, it would not make sense to test the patient without first testing her father for the presence of a mutation. If he has an identifiable mutation in either PKD1 or PKD2, the potential donor can be tested for that specific mutation. Finding the same mutation in the daughter would exclude her as a potential donor.
Note that an alternative strategy would be to perform either magnetic resonance imaging or computed tomography with contrast, both of which are more sensitive than ultrasound for the detection of small cysts in the kidneys and other organs. The presence of more than 10 cysts in both kidneys establishes the ADPKD diagnosis and would exclude donation, whereas the absence of any cysts appears to exclude the diagnosis. However, identification of fewer than 10 cysts would leave the diagnosis still in doubt. The use of cross-sectional imaging criteria in the evaluation of prospective donors is not well-established and genetic testing may be preferred if the initial screening ultrasound is not diagnostic.

How well did you know this?
1
Not at all
2
3
4
5
Perfectly
3
Q

A 70-year-old woman is admitted with multi-lobar pneumonia. She weighs 75 kg; her vital signs include temperature 38.4°C, BP 108/82 mm Hg, heart rate 96/min, and oxygen saturation of 94% on 2 L/min nasal oxygen. She has scattered coarse breath sounds and decreased air movement at both bases with dullness to percussion.
Laboratory data on presentation:
Result Reference Range
Leukocyte count 3,800 µL 4000–11,000
Hemoglobin 10 g/dL 12–16
Platelet count 140,000 µL 150,000–450,000

Sodium 125 mEq/L 136–145
Potassium 3.6 mEq/L 3.5–5.0
Chloride 100 mEq/L 98–106
Total CO2 23 mEq/L 23–30
BUN 15 mg/dL 8–20
Creatinine 1.1 mg/dL 0.5–1.1
Glucose 85 mg/dL 70–99 (fasting)
Calcium 10.2 mg/dL 8.6–10.2
Albumin 3.0 g/dL 3.5–5.5
Osmolality 296 mOsm/kg 275–295

Urine
Sodium 35 mEq/L Varies
Osmolality 350 mOsm/kg H2O 38–1400
Intravenous saline is administered at 200 mL/hour. She develops respiratory distress after 2 hours. A repeat chest x-ray demonstrates pulmonary congestion superimposed on pneumonia. An arterial blood gas (ABG) is obtained and a repeat sodium measurement on this specimen is 141 mmol/L.
What is the MOST likely cause of the rapid change in the sodium measurement?
Select one:

A. Overly rapid correction from normal saline
B. Metabolism of an osmolal gap secondary to an ingestion
C. Pseudohyponatremia on initial sample
D. Contamination from saline in ABG sample

A

The correct answer is: C
Pseudohyponatremia on initial sample

On presentation, this patient has a low serum sodium but the serum osmolality is in the normal range and higher than predicted based on calculation of the serum osmolality. This finding is consistent with pseudohyponatremia. The apparent increase in her sodium concentration occurs when her serum sodium is measured by a different technique, which eliminates the error.
The initial serum chemistries were measured using indirect potentiometry, the most common technique. With this method, a diluent is added to the serum to limit the amount of blood that is required for the measurement. Normally, serum contains a solid phase of approximately 7%, composed of proteins and lipids. If this solid phase is considerably larger, the liquid phase will be smaller. When the standard volume of diluent is added to a smaller aliquot of plasma (containing both liquid and solid phases), the components of the liquid phase will be relatively more diluted and the laboratory will report a falsely low serum sodium. In contrast, most laboratories use direct potentiometry for blood gas analysis, which eliminates this error. The serum osmolality is typically measured by freezing point depression and is not affected by changes in the solid phase.
Pseudohyponatremia can be caused by extreme elevation in plasma proteins (as with plasma cell dyscrasias), triglycerides (as with lipemic serum), or total cholesterol and lipoprotein X (as with obstructive jaundice). Note that an abnormality related to lipoprotein X does not result in lipemic serum. The patient may have occult multiple myeloma, as evidenced by her pancytopenia, low serum anion gap, and mild hypercalcemia (corrected calcium is 11 mg/dL). Multiple myeloma can result in hyperproteinemia and cause pseudohyponatremia.
Infusion of 400 mL of normal saline provides an additional 62 mEq of sodium, which would not be sufficient to cause a 16 mEq rise in serum sodium in a 75 kg woman. Although an osmolal gap and a normal anion gap are associated with isopropyl alcohol ingestion, this would not lead to either hyponatremia or pseudohyponatremia. Contamination of arterial blood samples can occur from the saline flush used for arterial line but not from the IV fluid. In addition, this would not explain the normal serum osmolality on the initial sample.

How well did you know this?
1
Not at all
2
3
4
5
Perfectly
4
Q

A 20-year-old woman with a history of systemic lupus erythematosus (SLE) since age 16 is referred for evaluation of hematuria and proteinuria. She is 10 weeks pregnant and has recently transferred from pediatric to adult care. She has no history of nephritis. Her current primary symptom is nausea, but she also has some joint aches. Hydroxychloroquine 200 mg daily, amlodipine 5 mg daily, and azathioprine 50 mg daily were all discontinued when her pregnancy was recognized 2 weeks previously. She remains on prednisone 10 mg daily and now also takes a daily prenatal vitamin.
Her BP is 140/90 mm Hg. She weighs 50 kg. Mild lower extremity edema is present.
Her urinalysis shows 300 mg/dL protein and moderate blood by dipstick. The urine sediment has occasional cellular casts and acanthocytes. The serum creatinine is 0.8 mg/dL. Her 24-hour urine protein is 5 g.
Which of the following is the MOST appropriate next step in the management of this patient?
Select one:
A.
Obtain an ultrasound-guided kidney biopsy
B.
Defer evaluation to the postpartum period

C.
Give pulse methylprednisolone and delay biopsy until later in pregnancy
D.
Increase prednisone to 50 mg daily

E.
Resume hydroxychloroquine and azathioprine

A

The correct answer is:A
Obtain an ultrasound-guided kidney biopsy.

This patient with underlying SLE, an active urinary sediment, and significant proteinuria should undergo an ultrasound-guided kidney biopsy to define her diagnosis and guide treatment. Indications for biopsy during pregnancy include new-onset glomerulonephritis (including lupus nephritis), severe nephrotic syndrome that requires therapy, and an acute decline in kidney function. Kidney biopsy can be safely undertaken during the first trimester. Biopsy can also be performed later in pregnancy, but the procedure may need to be performed with the patient lying in the lateral decubitus position rather than prone because of the gravid uterus. In addition, the risk for complications may be increased after mid-trimester. It would be inappropriate to defer the kidney biopsy when there is evidence of active nephritis that may adversely affect both the patient and the pregnancy if it is not treated.
In this case, the urinalysis is consistent with acute glomerulonephritis, and the patient’s history of SLE makes lupus nephritis of one form or another the most likely consideration. Serologic studies, including complement and anti-double-stranded DNA antibody levels, can be useful adjuncts, but cannot themselves classify glomerulonephritis adequately to guide therapy. If a severe, diffuse proliferative lesion were identified on biopsy, treatment would be warranted. This would also prompt difficult decisions in the context of the pregnancy. Both cyclophosphamide and mycophenolate mofetil are contraindicated during pregnancy.
Empirical pulse therapy would not be justified in the absence of AKI. Similarly, empiric increase in her prednisone to 50 mg would expose the patient (and fetus) to the side effects of high dose corticosteroids without any certainty of necessity or effectiveness, and potentially could hamper the ability to make a definitive diagnosis in the future if biopsy were eventually pursued.
Hydroxychloroquine should be continued during pregnancy and is considered safe. Azathioprine is potentially useful in the treatment of lupus during pregnancy and is considered reasonably safe in doses up to 2 mg/kg, but it would not be adequate treatment for severe lupus nephritis at the present dose. Mothers and families should be informed of reports of late developmental delay in some offspring of patients treated with azathioprine during pregnancy, though this association is not proven to be causal.

How well did you know this?
1
Not at all
2
3
4
5
Perfectly
5
Q

A 50-year-old man develops fevers, hypotension, and altered mentation. He has multiple myeloma and received a stem cell transplant 1 week ago. He remains pancytopenic. Urine and blood cultures have grown a gram-negative rod that has not yet been identified. He is transferred to the medical intensive care unit, where he is intubated and receives IV saline, vasopressin, and norepinephrine. A dose of gentamicin 240 mg IV is administered and piperacillin-tazobactam 3.375 g IV every 8 hours is ordered.
On examination, he is sedated and receiving mechanical ventilation. His BP is 90/60 mm Hg and his weight is 80 kg. There are coarse breath sounds that are decreased at the bases, and he has severe generalized edema.
Result Reference Range
Potassium 5 mEq/L 3.5–5.0
Total CO2 15 mEq/L 23–30
Blood urea nitrogen 30 mg/dL 8–20
Creatinine 2 mg/dL 0.7–1.3
Glucose 110 mg/dL 70–99 (fasting)
Lactate 5 mmol/L <0.7–2.1
Arterial blood gas
Fi02 50%, 5 cm H2O positive end-expiratory pressure
pH 7.35 7.38–7.44
PaCO2 30 mm Hg 38–42
PaO2 100 mm Hg 75–100
Chest x-ray reveals a right lower lobe opacity and bilateral pulmonary vascular congestion.
Continuous venovenous hemofiltration is begun utilizing 2.5 L/h of replacement solution, administered pre-filter. One day later, the gram-negative rod is identified as a Klebsiella species, sensitive to piperacillin-tazobactam, so further gentamicin therapy is not administered.
Which of the following options is the BEST strategy to optimize the pharmacodynamic properties of the piperacillin-tazobactam in this case?
Select one:
A.
Administer piperacillin-tazobactam as a prolonged infusion

B.
Administer piperacillin-tazobactam as a rapid bolus
C.
Give all replacement fluid post-filter

D.
Continue current piperacillin-tazobactam prescription
E.
Increase blood flow from 200 to 250 mL/h

A

The correct answer is: A
Administer piperacillin-tazobactam as a prolonged infusion

Target levels of antibiotics including piperacillin-tazobactam (piptazo) are associated with more effective bacterial killing and are more likely to be achieved with extended infusions (3 hours) during continuous renal replacement therapy (CRRT).
Inadequate antibiotic dosing for patients with sepsis may lead to adverse outcomes, and there is strong evidence that under-dosing of antibiotics is common among patients on CRRT.
Beta-lactam antibiotics, including piptazo, exhibit time-dependent killing and the drug concentration should exceed the minimal inhibitory concentration (MIC) for at least 50% of the dosing interval. Because of the potentially high clearance of antibiotics by CRRT, conventionally dosed antibiotics may fail to achieve this goal. Extension of infusion time increases the time at which the concentration of pip-tazo exceeds the MIC. This strategy has shown benefit for some patients with severe infections and sepsis.
Multiple factors affect drug clearance in CRRT, including residual renal function, modality, dose, drug properties, and volume of distribution.
The clearance of a drug (C) by CVVH can be estimated as follows:
C=effluent rate x sieving coefficient
For a patient receiving prefilter replacement fluid, the equation becomes:
C= effluent Rate x sieving coefficient x blood flow
blood flow + prefilter replacement
This equation demonstrates that giving all replacement fluid postfilter would increase the clearance, which would tend to reduce the piperacillin-tazobactam concentration.
Blood flow during CRRT does not influence drug clearance unless the replacement fluid is administered pre-filter, in which case a higher blood flow will increase the clearance and this will tend to reduce the time that the pip-tazo concentration exceeds the MIC.
Giving the piptazo as a bolus would achieve a higher peak level but the concentration would fall more rapidly below the target. This strategy may be more appropriate for drugs such as aminoglycosides that exhibit concentration-dependent rather than time-dependent killing.

How well did you know this?
1
Not at all
2
3
4
5
Perfectly
6
Q

A 43-year-old woman with a history of hypertension and bipolar disorder is admitted to the hospital after her husband noticed an alteration in her behavior. He states that she has had some vomiting and diarrhea in the last 24 hours and she has not been able to eat anything for the past 2 days. She has taken lithium for the last 20 years and has attempted suicide on two prior occasions.
Medications include lithium, hydrochlorothiazide, and a multivitamin.
On physical examination, she is somnolent but arousable. She is afebrile, her BP is 110/60 mm Hg, and her heart rate is 110/min. Her weight is 80 kg. She has no edema. The neurologic examination is notable for stuttering speech, tremulousness, and hyperreflexia.
Laboratory data is notable for blood urea nitrogen 35 mg/dL (reference range, 8–20) and creatinine 2.3 mg/dL (reference range, 0.5–1.1). Her lithium level is 3.5 mEq/L (therapeutic range, 0.6–1.2). There is no evidence of other ingestions.
In addition to volume expansion with IV fluid, which is the MOST appropriate initial treatment plan for this patient?
Select one:
A.
Initiate continuous venovenous hemofiltration and continue for 48 hours
B.
Recheck kidney function and lithium level in 4 hours
C.
Prescribe daily HD treatments for 4 hours at a blood flow of 350 mL/min
D.
Begin HD with first treatment 2.5 hours at a blood flow of 250 mL/min

E.
Prescribe HD for 6 hours with blood flow of 350 mL/min

A

The correct answer is: E
Prescribe HD for 6 hours with blood flow of 350 mL/min

This patient should receive HD for a minimum of 6 hours with a blood flow of 350 mL/min and treatments should be repeated until the serum lithium level is <1 mEq/L.
This patient manifests acute on chronic lithium toxicity evidenced by acute neurologic symptoms and an elevated lithium level superimposed on chronic lithium use. AKI and volume depletion can contribute to lithium poisoning among patients receiving chronic lithium therapy. In this case, the recent vomiting and diarrhea as well as her diuretic may have resulted in renal impairment which then led to lithium toxicity. Alternatively, the gastrointestinal symptoms may have been symptoms of emerging lithium toxicity, which then contributed to volume depletion and further lithium accumulation.
Although there are no randomized controlled trials that address the treatment of lithium toxicity, the mainstay of treatment involves volume resuscitation and prolonged dialytic therapy to achieve adequate removal.
Lithium is a monovalent cation, which distributes widely in the body and is not protein-bound. The volume of distribution is 0.5 L/kg; however, this can increase with chronic use to 0.7–0.9 L/kg. Lithium is freely filtered by the glomerulus but reabsorbed in the proximal tubules, loop of Henle, and the principal cells. The half-life of lithium varies widely from as short as 12 hours to nearly 60 hours depending on age, kidney function, and duration of therapy. Lithium has both acute and chronic toxicity. Acute toxicity includes AKI as well as neurologic and cardiac manifestations. Chronic lithium toxicity may lead to nephrogenic diabetes insipidus or CKD from chronic tubulointerstitial nephropathy. It can also lead to hyperparathyroidism, hypercalcemia, and hypocalciuria.
Indications for dialytic therapy include a serum lithium level >4 mEq/L among patients with AKI or CKD or the presence of altered mentation, seizures, or dysrhythmias regardless of lithium levels. For patients with acute on chronic lithium toxicity, plasma levels may not correlate with toxicity as closely as with acute toxicity. Because of more efficient (rapid) removal of lithium, intermittent HD is the preferred modality for extracorporeal treatment of lithium poisoning. The initial dialytic treatment should be extended until the lithium level is <1 mEq/L; a minimum treatment time of 6 hours is recommended if levels are not immediately available. Levels should be checked serially over the subsequent 12 hours to identify a rebound increase in levels that would require additional dialysis. Rebound of lithium levels can be caused by ongoing absorption from the gastrointestinal tract if an extended release formulation has been ingested. Redistribution from extravascular location to vascular space also contributes to level rebound. Continuous renal replacement appears to be less efficient at lithium removal but expert opinion suggests that it is a reasonable alternative if intermittent HD is not readily available, or if additional therapy is required after the initial conventional dialysis treatment.
A short dialysis treatment with low blood flow is appropriate for individuals who initiate dialysis for the treatment of uremia in the setting of CKD when rapid removal of uremic toxins could provoke dialysis disequilibrium syndrome.
Conservative management without dialysis is not appropriate for this patient with neurologic manifestations of lithium toxicity.

How well did you know this?
1
Not at all
2
3
4
5
Perfectly
7
Q

A 40-year-old man is referred for proteinuria and hematuria that was incidentally found on routine urinalysis. Review of systems is negative.
Examination is normal except for BP 140/85 mm Hg and trace pretibial edema. His hemoglobin is 13.5 g/dL (reference range, 14–18) and creatinine is 1.2 mg/dL (reference range, 0.7–1.3). Urinalysis shows 3+ protein and microscopy shows 4–10 RBC/HPF.
Twenty-four-hour urine shows 4.2 g protein.
A kidney biopsy is performed and a representative light microscopic image is shown.

Masani N, Jhaveri K, Fishbane S: Update on Membranoproliferative GN. Clin J Am Soc Nephrol 9(3):600—608, 2014
Light microscopy shows hypercellularity, hypersegmentation, thickened basement membranes and increased mesangial material in a membranoproliferative pattern of glomerular pathology.
Electron microscopy shows immune deposits in the mesangial, para-mesangial, and subendothelial regions. Occasional subepithelial deposits are also identified.
Immunofluorescence microscopy shows 4+ IgG, 2+ C3, 2+ C1q, 4+ kappa, and 1+ lambda granular fluorescence in the capillaries and mesangium, but is negative for IgM, IgA, fibrinogen, and albumin.

Which of the test below would be MOST helpful in determining the underlying etiology of this patient’s kidney disease?
Select one:
A. Serum protein electrophoresis
B. Antinuclear antibody
C. Factor H activity
D. Hepatitis C antibody
E. Blood cultures

A

The correct answer is: A
Serum protein electrophoresis

Serum protein electrophoresis is indicated for this patient with immune complex-mediated membranoproliferative glomerulonephritis (MPGN) with predominant kappa deposition.
Historically, MPGN was categorized based on pathologic features. However, the current classification of MPGN is based on the pathogenesis of the disease and whether it is mediated by immune complexes, complement, or endothelial dysfunction.
The immunofluorescence (IF) pattern of the biopsy provides clues to the pathogenesis and helps define the subtypes.
1. Immunoglobulin-positive, complement-positive (these can be monoclonal immunoglobulins as with a plasma cell dyscrasia, or polyclonal as with the case of hepatitis C)
2. Immunoglobulin-negative, complement-positive (disorders of complement, as with dense deposit disease or C3 glomerulonephritis)
3. Immunoglobulin-negative, complement-negative (as with a thrombotic microangiopathy)
Etiologies of immune complex MPGN include autoimmune disease such as lupus nephritis. Lupus nephritis, however, usually demonstrates IgA and C1q in addition to IgG, IgM, C3, kappa, and lambda by IF, often referred to as a “full house pattern.” MPGN related to hepatitis C (with or without cryoglobulins) is associated with the deposition of polyclonal immune complexes, and shows granular deposition of IgM, C3, and relatively equal kappa and lambda; IgG deposition is variable. MPGN related to chronic bacterial infection as with bacterial endocarditis would also produce polyclonal immune complex deposition, rather than a monoclonal pattern by IF. Deposition of a monoclonal immunoglobulin can also result in an immunoglobulin-positive, complement-positive MPGN-pattern of inflammation, as in this case.
C3 glomerulopathies are characterized by dysregulation of the alternative complement pathway and C3 deposition in the absence of significant immunoglobulin. Abnormal function of factor H, an inhibitor of the alternative pathway, can lead to unregulated complement activation, leading to C3 glomerulopathy. This disorder is characterized by C3 deposition without other significant immune deposition. Although C3 is present here, the additional immunoglobulin and monoclonal kappa deposition are not characteristic of C3 glomerulopathy and testing for factor H is not indicated.

How well did you know this?
1
Not at all
2
3
4
5
Perfectly
8
Q

A 44-year-old woman is evaluated for resistant hypertension. She feels well and is adherent to her antihypertensive regimen, which includes amlodipine 10 mg each night, carvedilol 25 mg twice daily, hydrochlorothiazide 25 mg daily, hydralazine 50 mg 3 times daily, and potassium chloride 20 mEq twice daily. She follows a 2 g sodium diet and consumes one glass of wine daily. She does not consume licorice or other herbal products. Her average BP by ambulatory monitor is 145/87 mm Hg. She has a family history of hypertension but details are not known to the patient.
On physical examination, she appears well. Her BP is 150/90 mm Hg, pulse is 70/min, and body mass index is 24 kg/m2. The remainder of the examination is normal.
Laboratory data (see table):

Which ONE of the following management strategies is the MOST appropriate for this patient?
Select one:

A. Replace hydralazine with lisinopril
B. Restrict alcohol intake to less than one drink daily
C. Replace hydrochlorothiazide with amiloride
D. Restrict sodium intake to 1 g daily
E. Replace hydrochlorothiazide with spironolactone

A

The correct answer is: C
Replace hydrochlorothiazide with amiloride

The presence of hypertension, hypokalemia, and metabolic alkalosis is consistent with hyperaldosteronism, yet here the renin level is suppressed and aldosterone level is not elevated. These findings are consistent with the diagnosis of Liddle syndrome, which is optimally treated with an epithelial sodium channel (ENaC) blocker such as amiloride.
This patient has resistant hypertension as defined by the failure to achieve BP control despite adequate doses of at least 3 antihypertensive medications including a diuretic. The ambulatory BP monitor excludes pseudo-resistance. The presence of resistant hypertension should prompt consideration of interfering substances and secondary causes. In this case, Liddle syndrome is the likely diagnosis based on the clinical and laboratory findings of spontaneous hypokalemia and metabolic alkalosis (before therapy is initiated) and the absence of elevation of renin, aldosterone, and cortisol.
Liddle syndrome is a rare, autosomal dominant cause of hypertension. It usually presents as early onset hypertension, but occasionally, this syndrome may escape detection until adulthood. Liddle syndrome results from an activating mutation of ENaC in the distal nephron. Increased ENaC activity results in increased sodium reabsorption. In addition, the increased luminal electronegativity leads to potassium secretion from the principal cells and favors H+ excretion via the H+ATPase at the apical membrane of the α-intercalated cells. Because of the resultant volume expansion, mineralocorticoid activity is suppressed. Thus, treatment with spironolactone is ineffective. Liddle’s syndrome is treated with blockade of ENaC by agents such as amiloride or triamterene.
Although it would be reasonable to consider substituting an angiotensin-converting enzyme inhibitor for hydralazine in a patient with hypertension and hypokalemia, it would not address the underlying issue in this case and is unlikely to be sufficient because renin is already suppressed.
Excessive sodium or alcohol intake may contribute to resistant hypertension, but this patient’s intake is already limited.

How well did you know this?
1
Not at all
2
3
4
5
Perfectly
9
Q

A 73-year-old woman with history of vaginal cancer that had been treated by hysterectomy and radiation therapy presents with constipation and abdominal pain. She is found to have a colonic stricture and undergoes resection and diverting colostomy.

Her medical history is notable for a kidney transplant 10 years ago, CKD stage G3a, and hypertension.

Her medications include aspirin, losartan, mycophenolate mofetil, and cyclosporine.

Her postoperative course is complicated by high-volume output from her colostomy.

Examination is significant for BP 100/58 mm Hg and pulse 100/min. The abdomen is soft and the ostomy is clean, dry, and intact with a healing stoma. The remainder of the examination is normal.

Lab data (see table):

Urinalysis: specific gravity 1.020, pH 5, negative blood, glucose and protein, and no cells or casts.

Which ONE of the following is the MOST likely cause of this patient’s metabolic acidosis?

Select one:

A. Calcineurin inhibitor
B. Chronic kidney disease
C. Type 1 (distal) renal tubular acidosis
D. Gastrointestinal losses
E. Type 4 renal tubular acidosis

A

Answer: D
Gastrointestinal losses

The patient has hypokalemia and a non-anion gap metabolic acidosis that is most likely caused by gastrointestinal losses.

A metabolic acidosis in which the fall in bicarbonate is matched by an equivalent rise in chloride is referred to as a non-anion gap metabolic acidosis (NAGMA) or a hyperchloremic metabolic acidosis. This can occur from a loss of bicarbonate from the gastrointestinal tract or the kidney, impaired net acid excretion, or intake of HCl or substances that are metabolized to HCl.

The history often provides key clues to the cause of a non-anion gap metabolic acidosis. For example, bicarbonate losses are expected from certain urinary diversions or the use of acetazolamide. In this case, there are multiple potential causes of a NAGMA, including gastrointestinal losses, CKD, and cyclosporine. Gastrointestinal losses can also cause extracellular volume depletion and hypokalemia, which can provoke additional superimposed metabolic abnormalities.

If the history, physical examination, and routine laboratory data do not fully explain the cause of the NAGMA, the next step is to assess renal acidification.

Although recent data suggest that direct measurement of urinary ammonium is much more accurate, and this analysis is becoming more widely available, renal acidification is traditionally assessed by using the urine anion gap, which compares the major ions that are easily measured in urine.

The urine anion gap is calculated by the following equation:

Urine anion gap = [Na+] + [K+] – [Cl-] = 46 + 20 – 106 = -40

In the setting of a metabolic acidosis, the expected, normal kidney response is to increase urinary excretion of ammonium with chloride, which results in urinary chloride levels that exceed the sum of urinary sodium and potassium levels. The urine anion gap among patients with metabolic acidosis from a non-renal source, therefore, is usually negative and in the -20 to -50 mEq/L range, as seen here. It should be noted, however, that ammonium excretion might be impaired among patients who have significant hypovolemia. Moreover, recent data suggest that direct measurement of urinary ammonium is considerably more accurate than estimation using the urine anion or osmolal gap.

Because this patient had a normal serum bicarbonate level on admission, it is unlikely that CKD is the cause of the acute metabolic acidosis. In addition, CKD would not explain the development of hypokalemia.

Type 1 (distal) renal tubular acidosis (RTA) also results in a hypokalemic non-anion gap metabolic acidosis because of the inability to excrete an acid load. In this situation, the urine pH would be >5.5 and the urine anion gap would be inappropriately high.

Type 4 RTA, defined as hyporeninemic hypoaldosteronism, is characterized by hyperkalemia, and the metabolic acidosis, if present, is typically mild. Calcineurin inhibitors can contribute to the development of a type 4-pattern RTA as well.

How well did you know this?
1
Not at all
2
3
4
5
Perfectly
10
Q

A 40-year-old man is referred to the nephrology clinic for evaluation of hematuria and proteinuria. He has a history of successfully treated hepatitis C (HCV) and well-compensated cirrhosis. His medications are tramadol, furosemide, and pantoprazole. His symptoms include arthralgias, a rash on his legs, and occasional numbness in his fingers.
On examination, his BP is 154/97 mm Hg and pulse is 76/min. There are multiple petechial lesions on his feet, legs, and back. The remainder of the examination is unremarkable.
Laboratory data (see table):

Which ONE of the following is the MOST likely cause of this patient’s hematuria and proteinuria?
Select one:
A. Focal and segmental glomerulosclerosis
B. Lupus nephritis
C. Henoch-Schönlein purpura
D. Membranous nephropathy

E. Cryoglobulinemic glomerulonephritis

A

The correct answer is: E
Cryoglobulinemic glomerulonephritis

Nephritic sediment in the presence of systemic manifestations, low complement, and high rheumatoid factor levels in a patient with a history of HCV infection, makes cryoglobulinemic glomerulonephritis the most likely cause of the hematuria and proteinuria.
Cryoglobulins are immunoglobulins that precipitate at a temperature lower than 37oC. There are 3 types of cryoglobulins, as shown below:
(see table)

Cryoglobulinemia can be asymptomatic or result in tissue damage from hyperviscosity (mainly with type I) or immune-complex mediated inflammation (mainly type II, strongly associated with HCV infection). Other associated conditions include infections, such as hepatitis B virus and HIV, and autoimmune disorders, such as Sjögren syndrome, systemic lupus erythematosus, and rheumatoid arthritis.
Renal manifestations of cryoglobulinemia may include hematuria, proteinuria, hypertension, and AKI. Hypocomplementemia, especially C4 depletion, and elevated rheumatoid factor are common. Blood cryoglobulin levels (sometimes referred to as the “cryocrit”) can be high, but reliable detection requires appropriate collection in pre-warmed syringes. The light microscopic appearance of a kidney biopsy specimen most commonly shows a membranoproliferative pattern, but may also show vasculitis, hyaline thrombi, and crescents. Immunofluorescence reveals deposition of immune complexes in the mesangium and subendothelium that are often IgM-dominant; electron microscopy may reveal substructure to the electron-dense deposits with a microtubular or fibrillary pattern. It is important to note that because of independent B-cell proliferation, cryoglobulins can persist even after HCV has been eradicated by antiviral therapy. Treatment for cryoglobulinemia that persists or appears after antiviral therapy may include steroids, rituximab, and plasma exchange in severe cases.
Although membranous nephropathy is sometimes associated with HCV, it does not explain the hypocomplementemia, high rheumatoid factor level, or this patient’s systemic signs and symptoms.
The presentation of lupus nephritis can be very similar to that of cryoglobulinemic glomerulonephritis, but lupus nephritis is unlikely in the setting of a low titer ANA and normal C3, as well as the absence of other auto-antibodies commonly found in lupus nephritis.
Pauci-immune glomerulonephritis may have systemic symptoms and can occasionally be ANCA- negative, but would not cause hypocomplementemia or an elevated rheumatoid factor.
Although complement activation is part of the pathophysiology of Henoch-Schönlein purpura, C3 and C4 levels are generally normal and rheumatoid factor is not be elevated in this condition.

How well did you know this?
1
Not at all
2
3
4
5
Perfectly
11
Q

A 27-year-old man is brought to the emergency room with altered mental status. He is known to have a history of depression but is unable to answer questions and further information is unobtainable.
His BP is 136/88 mm Hg. His temperature, heart rate, and pulse oximetry are normal. Skin turgor and capillary refill are also normal. His speech is slurred and is disoriented, but there are no other neurologic abnormalities and the remainder of his physical examination is unrevealing.
(see table)

In addition to administration of fomepizole, which of the following is the MOST appropriate next step in treatment?
Select one:
A. Isotonic saline
B. Activated charcoal
C. Continuous venovenous hemofiltration
D. Hemodialysis
E. Gastric lavage

A

The correct answer is: D
Hemodialysis

This patient should undergo HD urgently to treat a presumptive toxic alcohol ingestion. His presentation includes altered mentation in the presence of a severe anion gap metabolic acidosis and an elevated osmolal gap, all of which point to a toxic alcohol ingestion. The osmolal gap is determined by subtracting the calculated plasma osmolality from the measured plasma osmolality (Posm) as follows:
Calculated Posm = 2 × [Na] + glucose (mg/dL)/18 + BUN (mg/dL)/2.8 + ethanol (mg/dL)/3.7
Here, the calculated Posm = 2 × 145 + 118/18 + 8/2.8 + 0 = 299 mOsm/kg
The osmolal gap = 336 – 299 = 37 mOsm/kg. Normally, the osmolal gap is <10.
Many toxic ingestions are accompanied by co-ingestion of ethanol. If there is a measurable ethanol level, the serum ethanol level should also be included in calculated plasma osmolality after converting to mOsm/kg. Although the molecular weight of alcohol is 46 mg/mmol, in clinical practice, dividing the alcohol concentration measured in mg/dL by 3.7 rather than 4.6 appears to best account for the osmolal contribution of ethanol.
The combination of AKI, severe metabolic acidosis, an osmolal gap, and calcium oxalate crystals in the urine suggest that the patient ingested ethylene glycol (EG). EG is not directly toxic but generates toxic metabolites, including glycolic acid and oxalic acid.
Fomepizole is an alcohol dehydrogenase inhibitor and is used to attenuate the metabolism of alcohols. It has a 500–1000 times greater affinity for alcohol dehydrogenase than ethanol. It helps prevent the metabolism of ethylene glycol (or methanol) to its toxic byproducts. Administration of alkali to correct acidosis is often recommended, as it may promote renal excretion of metabolites. Pyridoxine and thiamine are given to promote further metabolism of glycolic acid to non-toxic derivatives. HD is effective at removing EG and its toxic metabolites and is recommended if there is severe metabolic acidosis (pH <7.25), AKI, ethylene glycol level >50 mg/dL, or glycolate level >10 mg/dL (if available). HD is generally continued until the EG level is <20 mg/dL. Continuous renal replacement therapy can be used but is less efficient than HD.
Gastric lavage and activated charcoal are unlikely to be effective unless given in the first 30–60 minutes after an ingestion. Volume expansion with isotonic saline might promote renal excretion but is not the best treatment for this patient with signs of severe intoxication.
Because of the high morbidity and mortality of toxic alcohol ingestions, it is important for clinicians to have a high index of suspicion for an ingestion.

How well did you know this?
1
Not at all
2
3
4
5
Perfectly
12
Q

A 59-year-old man returns for a scheduled follow-up appointment for management of immunosuppression. He has a history of anti-proteinase 3-(anti-PR3) positive ANCA vasculitis and pauci-immune glomerulonephritis. He feels well except for occasional heartburn.
ANCA vasculitis was diagnosed 8 months ago when he presented with dyspnea, wheezing, rash, AKI, and active urinary sediment. Serum creatinine at the time of his presentation was 2.3 mg/dL. Kidney biopsy demonstrated pauci-immune, crescentic glomerulonephritis and ATN.
Induction therapy consisted of pulse methylprednisolone 7 mg/kg intravenously for 3 days followed by oral prednisone at a dose of 1 mg/kg per day, which has been slowly tapered. Three doses of IV cyclophosphamide were given at monthly intervals. His rash and pulmonary symptoms rapidly improved. His physicians felt that he had achieved a complete remission, as he had a Birmingham Vasculitis Activity Score for Granulomatosis with Polyangiitis (BVAS/GPA) score of 0. Around the time of his third dose of cyclophosphamide, he also received a dose of rituximab for maintenance of remission and his prednisone dose was reduced to 10 mg daily, and subsequently tapered to further to 5mg daily. Current medications also include calcium, trimethoprim-sulfamethoxazole, vitamin D, alendronate, and omeprazole.
On exam, he has moon facies and a prominent dorsocervical fat pad. His BMI is 31 kg/m2. His BP is 145/84 mm Hg and his heart rate is 76/min. He has no rash and the remainder of the examination is normal.

(See Lab data)
Which one of the following is the MOST appropriate next step in the treatment of this patient?
Select one:
A. Prescribe lisinopril
B. Discontinue omeprazole
C. Prescribe azathioprine
D. Discontinue prednisone
E. Prescribe glipizide

A

The correct answer is: D
Discontinue prednisone

Prednisone should be completely discontinued with a taper in this patient with ANCA vasculitis. He has achieved a complete remission based on the BVAS/GPA, a tool used to measure disease activity based on clinical symptoms and signs in 9 organ systems. Furthermore, he has evidence of long-term side effects of steroid use, including Cushingoid features.
In an observational study of nearly 150 patients with ANCA-associated disease, the length of glucocorticoid therapy after 6 months appeared to have no effect on time to relapse, relapse-free survival, ESRD, or death. Conversely, steroid therapy beyond 6 months was associated with a significantly greater risk of infections. Based on this study and other observational data, most experts recommend that clinicians begin to taper prednisone once the patient has achieved remission, with a goal of discontinuing the steroids after 4–8 months of therapy.
Although the patient is hyperglycemic on a random measurement of blood glucose, it would be premature to initiate drug therapy for diabetes mellitus (DM). Rather, the possibility of DM should be investigated with either a glucose tolerance test or measurement of hemoglobin A1c. Lifestyle interventions such as diet and exercise are likely to be indicated regardless of results, but if the DM diagnosis is established, additional therapy may be warranted. It is also possible that hyperglycemia will improve or resolve with discontinuation of prednisone.
Azathioprine is a reasonable choice for maintenance therapy to prevent relapse of ANCA-associated vasculitis, but it would be used as an alternative to rituximab, rather than in combination. This patient received rituximab 2 months ago and is within the expected window of activity for this long-acting agent.
Omeprazole and other proton-pump inhibitors are often prescribed prophylactically for patients treated with high-dose oral corticosteroids to reduce the risk of gastrointestinal bleeding. There is epidemiologic evidence that this class of medication is linked to CKD, but a causal relationship has not been established. It is reasonable to plan to stop the PPI when steroid therapy has been completed. However, it would be premature to discontinue this therapy for this patient who has dyspepsia without replacing it with an alternative medication or investigating the symptom further.
Angiotensin-converting enzyme inhibitors are considered first-line agents for the treatment of hypertension in patients with kidney disease and proteinuria. However, this patient also has mild hyperkalemia, which is a relative contraindication for this agent. In addition, his BP may improve when the prednisone is discontinued.

How well did you know this?
1
Not at all
2
3
4
5
Perfectly
13
Q

A 61-year-old man with a long-standing history of rheumatoid arthritis is referred for kidney biopsy when his serum creatinine was noted to be elevated and accompanied by proteinuria.
He had taken aspirin 6 days previously for a headache but otherwise had not taken any non-steroidal anti-inflammatory medications.
On examination, his BP is 144/86 mm Hg. His examination is notable for basilar rales and marked lower extremity edema. The hemoglobin is 12.5 g/dL (reference range, 14–18). His platelet count and coagulation studies are normal. His estimated GFR is 26 mL/min/1.73 m2.
The kidney biopsy is performed.
Six hours after the biopsy, he develops severe flank pain. A CT scan shows a large perinephric hematoma, and his repeat hemoglobin is 8 g/dL. His kidney biopsy reveals amyloidosis.
Which of the following factors is the MOST likely to have increased the risk of bleeding in this patient?
Select one:
A. Male sex
B. Aspirin use
C. Amyloidosis
D. Reduced kidney function
E. Hypertension

A

The correct answer is: D
Reduced kidney function

For this patient with multiple potential risk factors, reduced kidney function has the strongest association with kidney biopsy-related bleeding.
Percutaneous kidney biopsy is an essential tool for the diagnosis of various kidney diseases, but there are associated risks. Bleeding is the primary complication, likely related to the vascular nature of the renal parenchyma. The reported rate of kidney biopsy complications varies in the literature, ranging from 3–6%. Serious complications, such as bleeding requiring a transfusion, occur less frequently, in just 1–3% of cases. AKI and CKD have both been independently associated with an increased risk of bleeding after kidney biopsy. The risk increases with higher stages of CKD. This increased risk has been attributed to increased interstitial fibrosis and cortical thinning. In more advanced disease, uremic platelet dysfunction may also play a role. Desmopressin is recommended by some authorities to prevent bleeding for patients with an eGFR <30 mL/min/1.73 m2. However, trial data is lacking and a recent systematic review found insufficient evidence to support this practice. Interestingly, some studies have found that use of desmopressin in patients with relatively preserved renal function can increase the risk of bleeding. Prior studies have shown that older age and female sex are also associated with an increased risk of bleeding after biopsy.
A large meta-analysis including more than 9000 patients did not find a significant association between elevated BP prior to biopsy and the risk of bleeding requiring a transfusion, although half the studies included used a systolic BP cutoff of 130 mm Hg. Smaller studies have shown that a BP >160/100 mm Hg is associated with an increased risk of bleeding.
The role of aspirin as a risk factor for post-biopsy bleeding was examined in a large retrospective study of more than 1000 patients involving 2 centers, in 1 of which it was routine to hold aspirin prior to kidney biopsy. An increased risk of post-biopsy hematoma and a greater fall in hemoglobin following biopsy was found among aspirin-treated patients, but there was no increased risk of major bleeding complications, including the need for intervention or transfusion. Most guidelines continue to recommend cessation of anti-platelet agents 5–7 days prior to the biopsy. For this patient, aspirin exposure 6 days prior to biopsy would be unlikely to contribute significantly to the development of a post-biopsy hematoma.
Amyloidosis was not shown to be associated with an increased risk of post-biopsy bleeding complications in a large retrospective study. A previously described association was likely due to reporting bias.

How well did you know this?
1
Not at all
2
3
4
5
Perfectly
14
Q

A 77-year-old woman presents to the emergency department with hemoptysis. She has had several weeks of worsening dyspnea on exertion, progressive lower extremity edema, and dark urine. She reports a history of sinus congestion, but has never required antibiotics or experienced epistaxis. She has not had any rash or joint symptoms.
Past medical history is significant for hypertension and hypothyroidism. She has a 20 pack-year history of smoking.
On physical examination, she appears anxious and tachypneic. Her BP is 178/100 mm Hg, pulse is 60/min, temperature is 37°C, and oxygen saturation is 78% on ambient air. Cardiovascular exam reveals no murmurs or rubs. Coarse crackles are present bilaterally. She has 1+ lower extremity edema bilaterally. There is no rash or synovitis.
Laboratory data:

Chest x-ray reveals bilateral alveolar opacities.
She is admitted to the intensive care unit. Bronchoscopy shows evidence of diffuse alveolar hemorrhage.
Serologic studies for ANCA and anti-GBM antibodies are pending as are blood and urine cultures. Gram stain of a bronchoalveolar lavage specimen is negative for organisms.
In addition to supporting oxygenation, which one of the following is the most appropriate NEXT step in the management of this patient?
Select one:
A. Await diagnostic serologic data
B. Pulse methylprednisolone and plasma exchange
C. Broad spectrum antibiotics
D. Kidney biopsy
E. Rituximab

A

The correct answer is: B
Pulse methylprednisolone and plasma exchange

The initiation of therapy with pulse methylprednisolone and therapeutic plasma exchange (TPE) is the most appropriate next step in the management of the patient with diffuse alveolar hemorrhage and evidence of acute glomerulonephritis. Kidney biopsy and serologic evaluation should be pursued, but treatment for the possibility of an autoimmune pulmonary-renal syndrome should be initiated even before a diagnosis is established.
The role of TPE in the management of autoimmune disorders continues to evolve. A randomized trial (“PEXIVAS”) for ANCA-related vasculitis (AAV) that included patients with severe kidney impairment and pulmonary hemorrhage, but excluded patients with anti-GBM disease did not find that TPE conferred benefit beyond that of immunosuppression. However, as of 2019, guidelines from the American Society for Apheresis (ASA) lists AAV with severe renal involvement as a class I indication for TPE, as it does for pulmonary hemorrhage related to either anti-GBM or ANCA antibodies. In this patient, for whom the clinical suspicion of an autoimmune process is high, and in whom anti-GBM disease has not been excluded, empirical TPE and corticosteroids would be indicated. Either Rituxan or cyclophosphamide (or both) is often added once the diagnosis is established. Renal biopsy can often be deferred until a patient is stabilized without a loss of diagnostic utility. Serologic data can be sufficient in some circumstances. Renal replacement therapy should be initiated if complications of AKI ensue, but should not delay empirical corticosteroids and plasma exchange. Based on the results of the PEXIVAS trial, some clinicians would forgo continuing TPE if AAV was identified, but others would continue TPE due to the pulmonary hemorrhage. New guidelines from the ASA may add clarity.

How well did you know this?
1
Not at all
2
3
4
5
Perfectly
15
Q

A 61-year-old woman with a history of hypertension, hypothyroidism, and diabetes presents to her primary care physician with dysuria and is prescribed a 5-day course of trimethoprim-sulfamethoxazole. Her medications include lisinopril, metformin, and levothyroxine. Four days later, she presents to the emergency department with nausea, diaphoresis, and generalized muscle weakness.
Her BP is 135/50 mm Hg and pulse is 55/min and regular. She appears unwell, sweaty, and anxious. The examination is notable for generalized weakness.

Plasma Result Reference Range
Potassium 6.0 mEq/L 3.5–5.0
Total CO2 22 mEq/L 23–30
Anion gap 10 mEq/L 7–13
Creatinine 1.8 mg/dL (baseline 1.5) 0.5–1.1

Her 12-lead electrocardiogram shows sinus bradycardia with peaked T waves and mild prolongation of the PR and QRS intervals.
Which of the following BEST explains this patient’s hyperkalemia?
Select one:
A. Competitive inhibition of the aldosterone receptor in the collecting tubule
B. Decreased distal delivery of sodium due to relative volume depletion
C. Direct toxicity to adrenal zona glomerulosa cells reducing aldosterone secretion
D. Competitive inhibition of the epithelial sodium channel in the collecting tubule
E. Inhibition of angiotensin II-mediated aldosterone release

A

The correct answer is: D
Competitive inhibition of the epithelial sodium channel in the collecting tubule

Trimethoprim is structurally similar to amiloride, and competitively inhibits the luminal epithelial sodium channel in the collecting duct. This leads to impairment of distal potassium excretion and can provoke hyperkalemia. Hyperkalemia was first reported primarily in patients receiving high-dose trimethoprim therapy for Pneumocystis jiroveci pneumonia in the setting of AIDS. Although the occurrence of severe hyperkalemia in outpatients is rare, caution is warranted when other risk factors for hyperkalemia are present, such as angiotensin-converting enzyme (ACE) inhibitors, reduced kidney function, or type IV pattern renal tubular acidosis. Note that trimethoprim also inhibits tubular secretion of creatinine so it is common for the serum creatinine to rise without actual kidney injury or a change in GFR.
ACE inhibitors block angiotensin-mediated aldosterone release by preventing conversion of angiotensin I to angiotensin II, which normally stimulates the cells of the zona glomerulosa of the adrenal gland to secrete aldosterone. Hyperkalemia is an uncommon side effect of ACE inhibitors (or angiotensin receptor blockers) in the absence of additional risk factors such as diabetes, decreased kidney function, decompensated heart failure, or co-administration of potassium-sparing diuretics. In this case, addition of trimethoprim is the most likely cause of the acute rise in potassium.
Eplerenone and spironolactone competitively inhibit the aldosterone receptor in the collecting duct.
Heparin and low molecular weight heparins can lead to hyperkalemia due to direct toxicity to zona glomerulosa cells, leading to reduced aldosterone production.
Decreased distal sodium delivery leads to a reduction in potassium secretion in the setting of volume depletion. However, there is no evidence of hypovolemia, such as a clinical history of poor oral intake or physical examination findings consistent with volume depletion.

How well did you know this?
1
Not at all
2
3
4
5
Perfectly
16
Q

A 45-year-old woman is evaluated for hypercalcemia. She initially presented to the hospital with cough, weight loss, lethargy, and dyspnea. On chest x-ray, she has bilateral hilar lymphadenopathy. A radiograph of her kidneys, ureter, and bladder is notable for calcifications in both kidneys. A lymph node biopsy reveals noncaseating granulomas.
Result (Reference Range)
Creatinine 1.4 mg/dL
[0.9 mg/dL 3 months earlier] (0.5–1.1)
Calcium 12 mg/dL (8.6–10.2)
Phosphorus 4.8 mg/dL (3–4.5)
PTH 10 pg/mL (10–65)
PTH-related protein <1.5 pmol/L (<1.5)
25-hydroxyvitamin D 25 ng/mL (30–60)
1,25-dihydroxyvitamin D 72 pg/mL (15–60)

Which of the following is the MOST appropriate therapy to treat the cause of this patient’s hypercalcemia?
Select one:

A. Prednisone
B. Denosumab
C. Furosemide
D. Cinacalcet
E. Sevelamer

A

The correct answer is: A
Prednisone

The most appropriate treatment for this patient’s hypercalcemia is prednisone. Based on this patient’s history, laboratory results, and histology, the most likely diagnosis is sarcoidosis. Hypercalcemia, in this setting, is mediated by extra-renal production of 1,25-dihydroxyvitamin D3 [1,25-(OH)2D] in granulomatous tissue. Corticosteroids are currently considered the best initial management for sarcoidosis and associated hypercalcemia. Corticosteroids reduce the 1-alpha hydroxylation that leads to 1,25-(OH)2D formation. By reducing 1,25-(OH)2D production, less calcium is absorbed or reabsorbed from the gastrointestinal tract, kidneys, and bone. Note that the patient’s elevated phosphorus is also partially due to 1,25-(OH)2D-mediated gastrointestinal phosphorus absorption.
Prednisone can be initiated at 0.3–0.5 mg/kg/day, with higher doses for more significant organ impairment. Steroid therapy may also be useful to treat interstitial nephritis and extra-renal manifestations of sarcoidosis as well. There are few studies that provide data on specific dosing and duration of steroids for the treatment of sarcoidosis. The disease often remits spontaneously, but relapses are common. Generally, a starting dose is maintained for at least 4 weeks with a gradual taper to a 5–10 mg maintenance dose for up to 12 months.
Other therapies that can reduce conversion of 25-(OH)D to 1,25-(OH)2D include ketoconazole and hydroxychloroquine. Azathioprine, mycophenolate mofetil, and tumor necrosis factor-α inhibitors such as infliximab can also be used in addition to, or in place of, corticosteroids for those patients who are intolerant of or do not respond to corticosteroids.
Patients should be counseled to stop any vitamin D supplements and to avoid sunlight exposure to avoid increased vitamin D substrate. While excessive calcium intake should be avoided, dietary calcium should not be strictly limited because osteopenia from bone resorption may ensue. Bisphosphonates may also help prevent osteoporosis in patients treated with corticosteroids (and possibly those not treated with steroids as well).
Isotonic saline can be used to treat acute hypercalcemia. This strategy improves intravascular volume depletion caused by high serum calcium levels and promotes urinary calcium excretion. Furosemide can be added to address volume overload that may occur with IV resuscitation. However, neither of these strategies would address the cause of hypercalcemia in this case nor prevent its recurrence.
Cinacalcet can improve hypercalcemia secondary to hyperparathyroidism by binding to the calcium-sensing receptor in the parathyroid glands. PTH in this patient is already appropriately suppressed.
Denosumab, an antibody that binds to the receptor activator of nuclear factor kappa-B ligand (RANK-L), is used to treat the hypercalcemia of malignancy (as well as osteoporosis).
Sevelamer is used to treat hyperphosphatemia in CKD. It could be used in this setting but would not improve the hypercalcemia. Reduction of 1,25-(OH)2D production would reduce phosphorus absorption without sevelamer.

How well did you know this?
1
Not at all
2
3
4
5
Perfectly
17
Q

A 60-year-old man with a history of severe chronic obstructive lung disease has been admitted to the intensive care unit (ICU) with respiratory failure. He is intubated, sedated, and receiving mechanical ventilation, corticosteroids, and antibiotics.
On day 2 of his ICU course, he remains sedated and ventilated. His BP is 88/50 mm Hg and pulse is 110/min. Diminished breath sounds are present on auscultation. There is no peripheral edema.
Chest x-ray demonstrates clear but hyperinflated lungs.
Which ONE of the following is the MOST appropriate treatment for this patient’s acid-base disorder?
Select one:
A. Dilute hydrochloric acid
B. Continuous renal replacement therapy
C. 0.45% sodium chloride
D. 0.9% sodium chloride
E. Acetazolamide

A

The correct answer is: D
0.9% sodium chloride

The patient has post-hypercapnic metabolic alkalosis and volume depletion, which are best treated with 0.9% sodium chloride infusion.
Patients with chronic respiratory acidosis, such as those with advanced chronic obstructive lung disease, develop a compensatory increase in bicarbonate, which raises the pH toward normal. The kidneys achieve this by increasing ammonia production and thereby net acid secretion, which results in the generation of “new bicarbonate” and loss of chloride with ammonium. The kidneys maintain the higher serum bicarbonate by increasing reabsorption of the greater filtered load of bicarbonate. This process begins immediately with retention of carbon dioxide, but it takes several days to achieve the maximal compensation. If the PaCO2 is rapidly lowered (corrected), which may occur when mechanical ventilation is applied, the retained bicarbonate results in an elevated pH, and is referred to as post-hypercapnic metabolic alkalosis.
If there is no stimulus to retain the high serum bicarbonate and kidney function is normal, the kidneys will begin to excrete the excess bicarbonate. In the setting of volume depletion or reduced kidney function, the elevated serum bicarbonate can persist. This patient has evidence of effective circulating volume depletion (hypotension and tachycardia)and should be treated with an infusion of 0.9% sodium chloride. Potassium chloride can also help correct the metabolic alkalosis and should be prescribed for this patient with hypokalemia.
The urine electrolytes in this case support the diagnosis; the urine chloride is low when metabolic alkalosis has resulted from chloride depletion as with vomiting, prior diuretics, or the chloride depletion from chronic respiratory acidosis. The urine sodium is low because of volume depletion and activation of the renin-angiotensin-aldosterone axis. However, urine sodium may be paradoxically high during the development of metabolic alkalosis when bicarbonaturia is occuring, mandating increased excretion of sodium as a counterion despite the presence of hypovolemia.
Acetazolamide is a good choice for patients with metabolic alkalosis who have evidence of total body volume overload but maintain a metabolic alkalosis because of reduced effective arterial blood volume (e.g, congestive heart failure).
Continuous renal replacement therapy can correct metabolic alkalosis but is rarely required. It can be used if this patient does not respond to volume expansion and develops additional electrolyte abnormalities and AKI with persistent hemodynamic instability.
Dilute hydrochloric acid has been used to treat severe metabolic alkalosis for patients with contraindications to either dialysis or infusion of saline or potassium chloride, but must be infused via a central line. Many hospitals no longer stock this medication.
In the setting of hypotension and no apparent free water deficit, isotonic 0.9% saline is preferred to hypotonic 0.45% sodium chloride.
In addition to infusion of 0.9% saline, adjustment in ventilator parameters to permit a degree of hypercapnia which more closely approximates the patient’s baseline may facilitate weaning from the ventilator when the acute illness resolves.

How well did you know this?
1
Not at all
2
3
4
5
Perfectly
18
Q

A 65-year-old woman with ESRD due to polycystic kidney disease has received HD for 3 months. She finds her appetite has improved since she started dialysis. Urine output has decreased slightly.
Medications include a renal vitamin daily, cholecalciferol 1000 IU daily, lisinopril 10 mg daily, and erythropoietin 5000 IU at each dialysis session.
Monthly blood work reveals:
Result Reference Range
Calcium 10.0 mg/dL (8.6–10.2)
Phosphorus 6.5 mg/dL (3–4.5)
PTH 500 pg/mL (10–65)
25-hydroxyvitamin D 28 ng/mL (30–60)

Which of the following is the BEST initial therapy for this patient’s CKD-MB?
Select one:
A. Initiate calcitriol 0.25 mcg by mouth 3 times per week at dialysis
B. Increase cholecalciferol to 2,000 international units by mouth daily
C. Initiate sevelamer carbonate 800 mg by mouth with each meal
D. Initiate cinacalcet 30 mg by mouth daily with dinner
E. Initiate calcium acetate 667 mg by mouth with each meal

A

The correct answer is: C
Initiate sevelamer carbonate 800 mg by mouth with each meal

The best initial therapy for this patient with metabolic abnormalities including secondary hyperparathyroidism and hyperphosphatemia is sevelamer carbonate with each meal.
Treatment of CKD-MBD is a routine but challenging task. As the name implies, this is not just a bone disorder but a constellation of mineral metabolism abnormalities that can have a major adverse effect on cardiovascular outcomes. Current treatment goals and therapies are largely opinion-based and not supported by data from large, well-controlled studies. In addition, treatment often places a heavy pill burden on the patient, making adherence difficult.
The optimal intact PTH level has not been defined. Various studies have found an association between elevated PTH levels and increased mortality at a cutoff ranging from 400–600 pg/mL. KDOQI guidelines target a broad intact PTH range of 2–9 times the upper limit of normal (a range of 130–585 pg/mL in most labs). In this case, although the intact PTH is within this range, it is relatively high and accompanied by hyperphosphatemia. Therefore, the optimal choice of therapy will lower the intact PTH level and improve (or at least not worsen) the other parameters. Hyperphosphatemia is associated even more strongly with poor cardiovascular outcomes for dialysis patients than is high PTH.
A physiologic approach is to address those metabolic abnormalities that enhance PTH secretion. These include high serum phosphorus, low serum calcium, and low 1,25-dihydroxyvitamin D. A phosphate binder is recommended as first-line therapy because it may lower the serum phosphorus and intact PTH level. There is an increasing array of choices for phosphate binders, which can be broadly divided into calcium-containing (calcium carbonate and calcium acetate) and non-calcium-containing (sevelamer carbonate, lanthanum carbonate, and emerging iron-based compounds). Data supporting the use of non-calcium-containing binders is accumulating. In this case, sevelamer is the preferred option because the serum calcium is already at the upper range of normal and hypercalcemia could result if a calcium-based binder were added.
Treatment with calcitriol (1,25-dihydroxyvitamin D3) may reduce the PTH. However, because it also enhances absorption of calcium and phosphorus from the gastrointestinal tract, it is contraindicated in the setting of hyperphosphatemia (and it can cause or worsen hypercalcemia). Calcitriol should not be prescribed until the phosphorus level is closer to normal (or ideally at least <5.5 mg/dL).
An increase in the cholecalciferol dose could potentially result in a reduction in the PTH level, but the 25-hydroxyvitamin D level is already normal, and it is uncertain that supplementation would result in more conversion to 1,25-dihydroxyvitamin D, which would then suppress PTH production. In addition, this conversion might also cause undesirable phosphorus and calcium absorption from the gut as described for calcitriol.
Cinacalcet, a calcimimetic agent that acts directly on the parathyroid glands to reduce parathyroid hormone secretion, is an effective means of lowering the intact PTH level. It is an appropriate option if initial adjustment of phosphorus, calcium, or vitamin D levels is not successful. With a brisk reduction in PTH, the patient may develop a mild “hungry bone syndrome” in which the phosphorus and calcium levels may both decrease significantly. If this occurs and the calcium level falls below 8.0 mg/dL and the phosphorus is less than 5.3 mg/dL, a change to a calcium-containing binder, such as calcium acetate or calcium carbonate, may be indicated.

How well did you know this?
1
Not at all
2
3
4
5
Perfectly
19
Q

A 34-year-old woman with no past medical history presents to the emergency department with weakness. She reports a 2-month history of unintentional weight loss and progressive fatigue with intermittent fevers. She denies diuretic or laxative use.
On physical examination, she is a thin woman who appears well. Her BP is 118/55 mm Hg and pulse is 72/min. Lungs are clear. Mucous membranes are dry and she has no lower extremity edema.
Result (Reference Range)
Sodium 135 mEq/L (136–145)
Potassium 3.0 mEq/L (3.5–5.0)
Chloride 113 mEq/L (98–106)
Total CO2 15 mEq/L (23–30)
Creatinine 0.85 mg/dL (0.5–1.1)
Albumin 3.7 g/dL (3.5–5.5)
Osmolality 299 mOsm/kg (275–295)
Venous blood gas
pH 7.27 (7.38–7.44)
PaCO2 33 mm Hg (38–42)
Urine
Sodium 74 mEq/L (Varies with intake)
Potassium 48 mEq/L (Varies with intake)
Chloride 86 mEq/L (Varies with intake)
Osmolality 374 mOsm/kg (38–1400)

Urine dipstick: pH 6.5 and 1+ protein

What is the MOST likely etiology of this patient’s electrolyte disorders?

Select one:
A. HIV infection
B. Paroxysmal nocturnal hemoglobinuria
C. Diabetes mellitus
D. Lead toxicity
E. Sjögren syndrome

A

The correct answer is: E
Sjögren syndrome

This patient most likely has Sjögren syndrome, which may cause a distal renal tubular acidosis (RTA).
This patient has a non-anion gap metabolic acidosis with hypokalemia. The normal kidney response to metabolic acidosis is to maximally acidify the urine by increasing urinary ammonium excretion. In this case, the relatively high urine pH of 6.5 points to a failure of this mechanism.
It is now possible to measure urinary ammonium concentration in many laboratories, but it remains most common to estimate urinary ammonium by calculating the urine anion gap (also called “net urine charge”). This calculation is useful to classify non-anion gap metabolic acidosis and distinguish between renal and non-renal causes. The urine anion gap calculation may not be reliable in the setting of significant bicarbonaturia, such as early proximal RTA, because it adds an additional unknown variable (urinary bicarbonate).
The urine anion gap is calculated by the following equation:
Urine anion gap = [Na+] + [K+] – [Cl-]
Because metabolic acidosis should result in increased urinary excretion of ammonium which is accompanied by chloride, urinary chloride levels are expected to exceed the sum of urinary sodium and potassium levels when metabolic acidosis is present. The urine anion gap, therefore, is usually negative and in the -20 to -60 mEq/L range among patients with acidosis if kidney function is normal. In this case, the urine anion gap is positive (urine anion gap = 74 + 48 – 86 = 36), implying a failure of the kidney to augment ammonium excretion.
It is also notable that the urinary potassium is elevated in the setting of hypokalemia. Spot urinary potassium measurements can be misleading, but a value greater than 40 mEq/L implies that urinary potassium wasting is present. Hypokalemia and inability to excrete ammonium are characteristics of a classic, distal RTA, also known as a type 1 RTA.
The most frequent causes of adult-onset hypokalemic distal RTA are autoimmune disease (most commonly Sjögren syndrome) and hypercalciuria. The prevalence of distal RTA in Sjögren’s syndrome is estimated at 25%.
HIV is not associated with a distal RTA; however, proximal tubular dysfunction associated with HIV infection has been described. In addition, proximal tubulopathy is a well-known complication of tenofovir disoproxil fumarate, a nucleotide reverse transcriptase inhibitor.
Paroxysmal nocturnal hemoglobinuria is a rare cause of proximal renal tubular acidosis. The kidney biopsy of affected individuals reveals hemosiderin deposition in proximal tubular cells.
Diabetes mellitus is associated with hyporeninemic hypoaldosteronism, also known as a type 4 RTA. This disorder is characterized by hyperkalemia and relatively mild metabolic acidosis.
Lead-related nephrotoxicity is typically associated with CKD, gout, hypertension, mild proteinuria, and hyperuricemia.

How well did you know this?
1
Not at all
2
3
4
5
Perfectly
20
Q

A 30-year-old woman presents for kidney donor evaluation. She is interested in donating to her brother, who has ESRD at age 30 secondary to Alport syndrome. She has no history of tobacco use, diabetes, or hypertension and takes no medications.
Her BP is 122/68 mm Hg, pulse 72/min, and BMI 24 kg/m2. Physical examination is normal.
Laboratory data reveal creatinine 0.9 mg/dL (reference range, 0.5–1.1) and estimated GFR 100 mL/min/1.73 m2. Urinalysis reveals no red blood cells and no protein. Kidney ultrasound is normal.
A skin biopsy reveals the segmental presence of the α5 chain of type 4 collagen along the epidermal basement membrane.
Which of the following is the MOST appropriate next step in the management of this prospective donor?
Select one:

A. Counsel against kidney donation
B. Measure anti-glomerular basement membrane (GMB) antibody titer
C. Obtain kidney biopsy
D. Obtain 24-hour urine for creatinine and total protein
E. Start an angiotensin-converting enzyme inhibitor

A

The correct answer is: A
Counsel against kidney donation

Based on the skin biopsy, this patient should be counseled against donation. This potential donor’s skin biopsy is consistent with mosaic expression of the α5 chain of type 4 collagen, indicating a heterozygous carrier state for the mutation. Although the clinical course is less severe than for affected males, females with X-linked Alport syndrome have a risk of ESRD approaching that of a patient with diabetes, reaching 30% by age 60 and 40% by age 80. For this reason, female carriers under age 45 are discouraged from kidney donation. This patient, as a heterozygous carrier, has a significantly higher lifetime risk of ESRD compared with the general population.
Skin biopsy is a less invasive diagnostic test for Alport syndrome, and kidney biopsy would only be necessary to confirm the diagnosis if the results revealed normal staining for the α5 chain of type 4 collagen along the epidermal basement membrane.
Anti-GBM disease is a potential post-transplant complication in kidney transplant recipients with Alport syndrome, but there is no role for measuring titers in the potential donor. Moreover, this test measures antibodies against the α3 chain of type 4 collagen, which occur in native anti-GBM disease. Anti-GBM disease in kidney transplant recipients with underlying Alport syndrome, is caused by antibodies to the α5 chain.
Angiotensin-converting enzyme inhibitor therapy is indicated for patients with Alport syndrome who have proteinuria, but there is no role for blockade of the renin-angiotensin-aldosterone system in the absence of proteinuria or hypertension.
Many kidney transplant programs require 24-hour urine collections to assure adequate kidney function if the estimated glomerular filtration rate is at the lower range of normal. In this case, the abnormal skin biopsy finding precludes kidney donation and therefore a 24-hour collection is unnecessary.

How well did you know this?
1
Not at all
2
3
4
5
Perfectly
21
Q

A 55-year-old man presents for follow-up of CKD. His medical history includes hypertension and type 2 diabetes mellitus that was diagnosed at age 31. Three months ago, he was placed on a low potassium diet to address serum potassium of 5.6 mEq/L. He reports that he is adhering to his diet. He feels well and has no additional concerns.
Medications include losartan, amlodipine, insulin glargine, and chlorthalidone.
On physical examination, he appears well. Blood pressure is 142/85 mm Hg, pulse is 85/min, and BMI is 36 kg/m2. Lungs are clear and cardiovascular examination is normal. There is no edema of the extremities.

Laboratory data:
Serum Result (Reference Range)
Sodium 138 mEq/L (136–145)
Potassium 5.5 mEq/L (3.5–5.0)
Total CO2 21 mEq/L (23–30)
Creatinine 1.5 mg/dL
(increased) (0.7–1.3)

Urine albumin-to-creatinine ratio 450 mg/g (<30)

Patiromer 8.4 g once daily is added at this visit.
In addition to a gradual reduction in serum potassium, which of the following is MOST likely following this change in his regimen?

Select one:
A. Increase in enteral magnesium absorption
B. Increase in enteral oxalate absorption
C. Increase in enteral sodium absorption
D. Increase in enteral calcium absorption

A

The correct answer is: D
Increase in enteral calcium absorption

Patiromer is a potassium binding polymer that utilizes calcium sorbitol as a counterion to exchange for potassium. It is associated with a modest increase in calcium absorption and also therefore urinary excretion.
Patiromer is an oral non-absorbable potassium binding agent approved for the treatment of chronic hyperkalemia and is expected to decrease the serum potassium by 0.5–1 mEq/L over the course of 3 days to 2 weeks. This agent has been studied in patients at risk for hyperkalemia associated with renin-angiotensin-aldosterone blockade and allows for the potential continuation of this blockade in settings where hyperkalemia may otherwise limit or prohibit this strategy.
Patiromer has been studied at varying doses based on baseline serum potassium. Mean serum potassium decrease is 0.73 mEq/L, with more accentuated decrease among patients starting with higher potassium levels. Patiromer carries a black box warning against use for immediate treatment of life-threatening hyperkalemia, given that decrease in serum potassium is not immediate.
Patiromer releases calcium when it binds potassium. Recent studies demonstrate a moderate increase in calcium excretion among normal subjects receiving patiromer. So far, there are no recognized effects of patiromer on oxalate absorption.
One of the most commonly reported adverse effects of patiromer is a reduction in serum magnesium as stool losses of magnesium are increased. This may be particularly important among patients taking proton-pump inhibitors chronically, who commonly have hypomagnesemia because the subsequent change in gastric pH appears to limit magnesium absorption by enterocytes.
Sodium polystyrene sulfonate is associated with enhanced enteric sodium absorption because sodium is exchanged for potassium.

How well did you know this?
1
Not at all
2
3
4
5
Perfectly
22
Q

A 37-year-old woman with stage 2 CKD due to autosomal dominant polycystic kidney disease (ADPKD) is referred for management of an intracranial aneurysm (ICA). Her father has also been diagnosed with ADPKD, but there is no family history of ICA or subarachnoid hemorrhage (SAH). The patient does not smoke. She works as a bus driver. Because of her occupation, she underwent MRI and angiography of the brain (MRI/MRA) 5 years ago, which revealed a 3-mm anterior circulation ICA. She currently feels well and denies headache, visual changes, or vomiting.
On physical examination, blood pressure is 130/84 mm Hg and heart rate is 72/min. She has no edema, and heart and lung findings are normal. Neurologic examination is normal.
Her serum creatinine is 1.0 mg/dL (reference range, 0.5–1.1) and urinary albumin is 56 mg/g creatinine (reference range, <30).

Which one of the following is the MOST appropriate next step in management of ICA?

Select one:
A. Lisinopril
B. Repeat MRI/MRA
C. Angiography and endovascular intervention
D. Surgical clip of aneurysm
E. No further evaluation

A

The correct answer is: B
Repeat MRI/MRA

The most appropriate next step in management of this patient with ADPKD and an ICA is to repeat the MRI/MRA.
The prevalence of ICA among patients with ADPKD is high compared with the general population (9–12% versus 2–3%) and is believed to result from polycystin dysfunction in the arterial endothelium and vascular smooth muscle. Risk factors for the development of ICA include smoking, hypertension, and family history of ICA or SAH.
Although ICA rupture may be catastrophic, not all ICA have a high risk of rupture, and screening is not required for all patients with ADPKD. Kidney Disease Improving Global Outcomes (KDIGO) guidelines recommend ICA screening only for patients without limited life expectancy who have a family history of ICA or SAH, are employed in high-risk occupations (e.g., airline pilots or bus drivers), or those who request screening. Patients with family history as their only risk factor are advised to have MRI/MRA every 5–10 years. Patients who have known aneurysms <7 mm in diameter, engage in high-risk occupations, or are planning conception, transplantation, or other major surgery are advised to have MRI/MRA every 1–5 years. ICA >10 mm in diameter have a relatively high risk of rupture and, if feasible, should be repaired by surgical clipping or endovascular intervention such as deployment of a coil.
Patients with ADPKD who have an anterior circulation ICA <7 mm in diameter, such as this one, have a low risk of rupture (0.5%), so MRI/MRA every 1–5 years would be appropriate.
In the HALT-PKD trial, lisinopril to target blood pressure 95–110/60–75, compared with 120–130/70–80, conferred a slower increase in kidney volume and improved left ventricular mass index and urine albumin excretion among patients with ADPKD and eGFR ≥60 mL/min/1.73 m2 at 5 years. However, risk of ICA rupture was not assessed in this trial, and there is no evidence that aggressive blood pressure control affects the natural history of ICA.
Cerebral angiography is invasive and not recommended for screening. Prophylactic repair of a small ICA is unnecessary and is associated with procedural risk. Conversely, it would not be appropriate to forego periodic screening in a patient with ADPKD with a known ICA

How well did you know this?
1
Not at all
2
3
4
5
Perfectly
23
Q

A 56-year-old man with diabetes mellitus is admitted to the vascular surgery service for management of a chronic diabetic foot ulcer. Wound cultures have grown Enterococcus and Klebsiella. Antibiotics and wound care are initiated and revascularization is planned. His usual medications, which include insulin, metformin, amlodipine, aspirin, atorvastatin, and metoprolol, are continued. In addition, he is treated with piperacillin-tazobactam and subcutaneous heparin.
His medical history includes myocardial infarction, hypertension, and deep vein thrombosis.
On physical examination, he appears well. His blood pressure is 169/85 mm Hg and pulse is 85/min. Body mass index is 35 kg/m2. The lungs are clear and the heart is normal. The left lower extremity is wrapped with a dressing over the left heel. There is bilateral mild pretibial edema. A post-void residual bladder volume by bladder scan is 100 mL. Kidney ultrasonography demonstrates normal kidney size and contour without hydronephrosis.

Which of the following options is the BEST management of this patient’s hyperkalemia?

Select one:
A. Add furosemide
B. Discontinue heparin
C. Discontinue metoprolol
D. Insert a urinary catheter
E. Add oral potassium binder

A

The correct answer is: A
Add furosemide

The best management for this patient’s hyperkalemia is the addition of a diuretic such as furosemide.
The findings of hyperkalemia and mild hyperchloremic metabolic acidosis are consistent with a type 4 renal tubular acidosis (RTA) and is commonly identified in patients with diabetic nephropathy. Hyperkalemia is a result of hyporeninemic hypoaldosteronism, leading to impaired potassium excretion in the distal nephron. Metabolic acidosis, if present, is typically mild and has been attributed to impaired ammonium production secondary to intracellular alkalosis induced by hyperkalemia.
First-line treatment includes the addition of a kaliuretic diuretic. Both thiazide and loop diuretics can be effective, but loop diuretics may be more potent for patients with CKD. In addition to inducing natriuresis and kaliuresis, diuretics improve volume overload and correct the metabolic acidosis. In a volume-neutral or volume-depleted patient, oral sodium bicarbonate can be added to address the metabolic acidosis and provide a source of sodium so that diuretic therapy does not result in symptomatic volume contraction.
Aldosterone production by the zona glomerulosa cells of the adrenal cortex may be suppressed by heparin. This may result in hyperkalemia, but this patient’s hyperkalemia predated the exposure to heparin. Furthermore, this patient is at high risk of venous thrombosis, so discontinuation of heparin prophylaxis would be considered only if other strategies were unsuccessful.
Non-selective beta-blockers can contribute to hyperkalemia by interfering with the β2-adrenergic facilitation of potassium entry into cells. This effect is typically mild unless other factors are present. Metoprolol is β1-selective, so discontinuation is unlikely to affect the hyperkalemia. Further, the history of myocardial infarction and stable angina are strong indications to continue beta-blockade.
Although the post-void residual is abnormal, the degree of urinary retention is less than that normally associated with a need for intervention. It would be premature to insert a urinary catheter with the attendant risk of infection. It would be reasonable to consider stopping medications, such as amitriptyline, which may impair bladder function.
Sodium polystyrene sulfonate (SPS) resin has been used to manage both acute and chronic hyperkalemia, but the effectiveness and safety have been questioned. The exchange of sodium for potassium can aggravate volume overload. This strategy would be considered only if the hyperkalemia could not be managed with other methods. Alternative potassium binding medications, including patiromer and sodium zirconium cyclosilicate, appear to be safer and more effective than SPS, but diuretic therapy is the first approach to a type IV RTA, particularly in patients such as this who also have hypertension and edema.

How well did you know this?
1
Not at all
2
3
4
5
Perfectly
24
Q

A 56-year-old man with a history of CKD attributed to type 2 diabetes presents to the clinic for a routine follow-up visit. Diabetic complications include retinopathy, neuropathy, and albuminuria.
Medications include amlodipine, metoprolol, atorvastatin, and metformin.
On physical examination, his blood pressure is 134/78 mm Hg and heart rate is 82/min. There is mild lower extremity edema, and heart and lung exams are normal.
Result Reference Range
Creatinine 1.4 mg/dL 0.7–1.3
Hemoglobin A1c 7.7% 4–5.6
Urine albumin-to-creatinine ratio 510 mg/g <30
Two weeks ago, the patient’s endocrinologist prescribed empagliflozin 10 mg daily to improve glycemic control.
Which of this patient’s medications is associated with a reduction in hyperfiltration and delayed progression of diabetic nephropathy in this patient?
Select one:

A. Empagliflozin
B. Metformin
C. Metoprolol
D. Atorvastatin
E. Amlodipine

A

The correct answer is: A
Empagliflozin

The sodium-glucose transporter (SGLT2) inhibitor class of medications decreases glomerular hyperfiltration and the risk of progression of diabetic nephropathy.
This patient has evidence of increased glomerular capillary pressure based on the elevated urinary albumin. Persistently elevated glomerular capillary pressures are felt to contribute to glomerular damage secondary to diabetic nephropathy. Recent studies suggest that this process can be mitigated by inhibitors of the low affinity, high capacity Na+-glucose cotransporter (SGLT2). A major factor in this process appears to be the increase in distal tubule NaCl delivery seen with initiation of empagliflozin.
Under normal circumstances, most filtered glucose (90%) is reabsorbed in the proximal tubule by SGLT2. Early in the course of diabetic nephropathy, the proximal tubule cells undergo hyperplasia and hypertrophy to absorb the higher glucose load present in the filtrate. Enhanced glucose reabsorption proximally is matched by increased Na+ reabsorption (and paracellular Cl- absorption). Thus, less Na+ and Cl- are delivered to the macula densa. This can inhibit tubuloglomerular feedback, which leads to afferent arteriolar vasodilation. Intraglomerular pressure rises and leads to hyperfiltration, albuminuria, and ultimately sclerosis. SGLT2 inhibitors appear to normalize distal tubule NaCl delivery, and in Akita diabetic mice, decrease albuminuria, kidney weight, and glomerular size. Proximal tubule hypertrophy does not increase with SGLT2 inhibitors.
The Empagliflozin, Cardiovascular Outcome Event Trial in Type 2 Diabetes Mellitus Patients enrolled 7020 patients with type 2 diabetes (hemoglobin A1c 7–10%) and high cardiovascular risk. Patients randomly assigned to empagliflozin, compared with those receiving placebo, had lower risks of all-cause mortality (32%), cardiovascular mortality (38%), and hospitalization for heart failure (35%). A prespecified subgroup of patients with CKD (estimated GFR <60 mL/min/1.73 m2) experienced these benefits, as well as a lower risk of renal endpoints (doubling of creatinine, worsening nephropathy, ESRD, or renal death). Other agents in this class have also been shown to reduce cardiovascular and renal risk in patients with type 2 diabetes and use of this class in renal impairment has been expanded to include patients with eGFR not less than 30 mL/min/1.73 m2. More recent trials with similar medications, including Canagliflozin and Renal Outcomes in Type 2 Diabetes and Dapagliflozin in Patients with Heart Failure and Reduced Ejection Fraction, found these benefits extend to diabetic patients with eGFR as low as 30 mL/min/1.73 m2 as well as congestive heart failure patients, with or without diabetes.
Control of blood pressure is an important strategy to reduce progression of diabetic nephropathy (and CKD in general), but neither metoprolol nor amlodipine has been reported to act specifically to reduce hyperfiltration.
Metformin may retard progression of diabetic nephropathy by facilitating glycemic control, which may indirectly reduce hyperfiltration, but a direct effect has not been reported.
Statins are indicated to reduce the risk of cardiovascular events among patients with diabetes and CKD, but a direct effect on hyperfiltration has not been reported. The effect of statins on the progression of CKD and proteinuria is the subject of ongoing investigation; there is some evidence of benefit, although the mechanism is not known.

How well did you know this?
1
Not at all
2
3
4
5
Perfectly
25
Q

A 28-year-old man with type 1 diabetes is brought to the emergency department following a syncopal episode. He notes that he has been nonadherent to his insulin regimen. He has been diabetic for several years but has not had recognized complications. He reports feeling lightheaded, thirsty, and weak. Until yesterday, he was urinating frequently and drinking copiously, but urine output has become scant in the past day.
On physical exam, blood pressure and heart rate are 108/62 mm Hg and 88/min (seated) and 88/50 mm Hg and 116/min (standing), and weight is 72 kg. There is no edema, and heart and lung exams are normal with the exception of orthostatic tachycardia. Mucous membranes are dry and skin turgor is decreased.

Result (Reference Range)
Sodium 140 mEq/L (136–145)
Potassium 3.2 mEq/L (3.5–5.0)
Creatinine 2.8 mg/dL (0.7–1.3)
Glucose 750 mg/dL (70–99 (fasting))
Osmolality 350 mOsm/kg (275–295)

Urinalysis is significant for 3+ glucose, 3+ ketones, and specific gravity 1.015

Which one of the following is the MOST closely approximates this patient’s water balance on presentation?

Select one:

A. 3 L deficit
B. 1.5 L gain
C. 1.5 L deficit
D. 0 L (no deficit)
E. 3 L gain

A

The correct answer is: A
3 L deficit

This patient has developed a 3 L water deficit related to his uncontrolled diabetes and osmotic diuresis with inadequate fluid intake despite his thirst.
Severe hyperglycemia is commonly associated with significant volume depletion, and both hyponatremia and hypernatremia can occur depending on water intake compared with losses. Sodium, other electrolytes, and water are lost in the urine during the osmotic diuresis, but the net loss is unpredictable and may depend on the duration and severity of the hyperglycemia, underlying kidney function, and intake during the episode. Relatively more water loss than sodium loss results in hypernatremia that may be masked by the translocation of intracellular water to the extracellular space due to the osmotic effect of the hyperglycemia. To manage patients effectively, it is crucial to determine a corrected serum sodium concentration and use that value to estimate the relative loss or gain of water.
In this case, the normal serum sodium might suggest there is no water deficit, but this unlikely given the elevated glucose and serum osmolality.
To estimate the water deficit, it is necessary to first estimate the serum sodium, which requires correction for the effect of the elevated glucose. The most commonly cited correction factor for the serum sodium concentration is 1.6 mmol/L for every 100 mg/dL increase in glucose concentration above normal. In this case, the increase in glucose is 750 – 100 = 650 mg/dL. The corrected sodium is thus 140 + (6.5 × 1.6) = 150.4 mmol/L.
Free water deficit (FWD) may then be estimated by the following equation:
FWD = 0.6 × weight (kg) × ([serum Na / 140] – 1) = 0.6 × 72 × ([150.4/140] – 1) = 3.2 L
Thus, the closest approximation of this patient’s free water balance is a deficit of 3 L.
The management of diabetic ketoacidosis involves correction of volume depletion with IV fluid as well, as suppression of ketogenesis and correction of hyperglycemia with insulin. Potassium deficits are also common, and severe hypokalemia may result if potassium is not supplemented before insulin is given. Although the initial therapy should include isotonic crystalloid to restore extracellular volume, hypotonic fluids may also be required to address a water deficit, as in this case. The rate of correction of both the hyperglycemia and the hyperosmolality should be gradual to avoid cerebral edema.
Note that the sodium correction factor of 1.6 mmol/L per 100 mg/dL increase in glucose concentration is controversial. Alternative formulas have been suggested, including one that endorses a higher correction factor if the serum glucose is greater than 400 mg/dL. Application of this formula results in a corrected sodium of 159 mEq/L and a water deficit of 6 L. Whichever formula is used, the crucial concept is that correction of the serum sodium concentration is important in the setting of hyperglycemia, and allows the clinician to recognize an abnormality in water balance that may otherwise be masked. Therapy can be initiated with this in mind and modified as laboratory parameters change during treatment.

How well did you know this?
1
Not at all
2
3
4
5
Perfectly
26
Q

A 46-year-old woman with stage 3 CKD is referred to the outpatient clinic for evaluation of hypercalcemia. She has a history of bipolar disorder, which has been successfully treated with lithium carbonate for the past 13 years. She has no history of acute lithium toxicity, AKI, or proteinuria, and the etiology of CKD has not been defined. Although lithium discontinuation has been considered, her bipolar disorder was poorly controlled before treatment with lithium and she and her psychiatrist are reticent to stop this agent. The patient takes no other medications or supplements.
On physical examination, her blood pressure is 112/56 mm Hg and heart rate is 82/min. There is no edema, and heart and lung exams are normal.
Bone densitometry is normal.

Result Reference Range
Creatinine 1.4 mg/dL (stable) 0.5–1.1
Calcium 11.2 mg/dL 8.6–10.2
Ionized calcium 1.46 mmol/L 1.12–1.23
Phosphorus 3 mg/dL 3–4.5
PTH 266 pg/mL 10–65
Lithium 1 mEq/L therapeutic range, 0.6–1.2

Which one of the following is the MOST appropriate next step in treatment?

Select one:
A. Furosemide
B. Alendronate
C. Cinacalcet
D. Amiloride
E. Magnesium

A

The correct answer is: C
Cinacalcet

The most appropriate next step in treatment of this patient would be the addition of cinacalcet, a calcimimetic drug capable of suppressing parathyroid hormone.
The association between lithium and altered calcium homeostasis has been known for more than 50 years. Lithium may increase absorption of calcium from renal tubules and the gastrointestinal tract. In addition, lithium appears to decrease parathyroid gland sensitivity to calcium by inhibiting the calcium sensing receptor (CaSR), which results in an increase in PTH, a response analogous to that seen with familial hypocalciuric hypercalcemia. The result is typically a mild increase in serum calcium and PTH, but more significant increases can occur, perhaps by causing or exacerbating primary hyperparathyroidism (PHPT). Indeed, hyperplasia of all 4 glands and adenomas have been observed.
PHPT-mediated hypercalcemia may also be aggravated by lithium inhibition of CaSR activity in the loop of Henle. Normally, hypercalcemia activates the CaSR in the loop, which results in inhibition of the renal outer medullary K (ROMK) channel, leading to a decrease in potassium substrate for the sodium-potassium-chloride co-transporter on the luminal membrane, and a decrease in paracellular calcium reabsorption. Instead, lithium inhibits the CaSR and calcium reabsorption continues.
If lithium can be safely discontinued, mild hypercalcemia often resolves. Because altered calcium sensing appears to explain the pathogenesis of lithium-induced hyperparathyroidism, cinacalcet, which increases the sensitivity of the CaSR to calcium, would be expected to address this issue and help avoid surgical intervention. This was demonstrated in a case report describing 3 patients with lithium-mediated hypercalcemia who received cinacalcet 30–120 mg daily with subsequent normalization of calcium and PTH.
Amiloride has been used to treat lithium-induced nephrogenic diabetes insipidus but not PHPT; in fact, by blocking sodium entry into the principal cells, amiloride causes membrane hyperpolarization in the distal nephron and increases calcium reabsorption. Furosemide increases calcium excretion by blocking paracellular transport in the loop of Henle but has also been associated with secondary hyperparathyroidism. Stimulation of both calcium resorption from bone as well as calcium excretion in the urine could lead to total body calcium depletion. Additionally, furosemide may cause hypovolemia.
Magnesium levels in tissue, but not plasma, increase in response to high doses of lithium in rodent experiments, but there is no evidence that magnesium supplementation would affect lithium-mediated PHPT.
Alendronate (and other bisphosphonates) have been shown to improve bone mineral density among patients with primary hyperparathyroidism, but there is no information on the use of these agents specifically in lithium-mediated PHPT. The safety and efficacy of this class of medicines in CKD remains an area of controversy. In this case, because bone mineral density is normal, there would be no indication for this class of medication.

How well did you know this?
1
Not at all
2
3
4
5
Perfectly
27
Q

A 76-year-old man is referred for evaluation for kidney transplantation. He has had a progressive decline in kidney function over recent years attributed to diabetes and hypertension. His most recent estimated GFR was 16 mL/min/1.73 m2. He reports limited mobility due to back pain. His spouse notes that he has been more tired than usual. He smokes ½–1 packages of cigarettes per week. He does not have any chest pain, shortness of breath, lower extremity swelling, or claudication. He is interested in kidney transplantation to avoid dialysis and improve his energy level.
In addition to diabetes mellitus and hypertension, his medical history is notable for prostate cancer (Gleason 7), which was treated by prostatectomy 1 year ago. He currently has no evidence of disease.
His blood pressure (BP) has ranged from 150–170/80–90 mm Hg on amlodipine and furosemide.
Physical examination reveals BP 160/80 mm Hg, pulse 80/min, and body mass index (BMI) 38 kg/m2. Other than abdominal obesity and elevated BP, there are no abnormal findings.
Laboratory data shows:

Result Reference Range
Potassium 4.9 mEq/L 3.5–5.0
Total CO2 18 mEq/L 23–30
Creatinine 3.2 mg/dL 0.7–1.3
Hemoglobin A1c 6.5% 4–5.6
Prostate specific antigen undetectable No specific normal or abnormal level
Which one of the following is an ABSOLUTE contraindication to kidney transplantation for this patient?
Select one:

A. History of prostate cancer
B. BMI above 35 kg/m2
C. Current smoking
D. Uncontrolled hypertension
E. Age above 75

A

The correct answer is: A
History of prostate cancer

Recent history of prostate cancer is an absolute contraindication to kidney transplantation at this time. It is recommended that potential kidney transplant candidates with a history of cancer should have a waiting period free of recurrence of 2– 5 years, depending on the type and extent of malignancy. For prostate cancer, at least 2 years is generally recommended to minimize the risk of recurrence because of the potential presence of micrometastases that might expand once immunosuppression is introduced. For other cancers, such as myeloma, breast cancer, and melanoma, the waiting period is 5 years. Therefore, this patient should wait at least another year before being considered for kidney transplantation. It is worth noting that there are no strong data to support these exact waiting periods, although they are widely recommended by transplant guidelines.
There is no absolute age cut-off for potential recipients, and current data indicate that older transplant recipients, even those who are older than 75, have improved mortality compared with similarly aged candidates who remain on the waiting list. This survival benefit, however, is not realized until approximately 2 years after transplantation. Transplantation of a kidney from a living donor may confer benefit sooner. Although smoking is associated with worse kidney transplant outcomes, and some programs decline to transplant active smokers. However, it is not considered an absolute contraindication to kidney transplantation. Most programs strongly encourage patients to stop smoking prior to transplantation.
Uncontrolled hypertension is not an absolute contraindication to transplantation. Adherence (“compliance”) can influence candidacy and control of blood pressure and blood sugar are sometimes used to gauge this aspect.
Obesity is a relative contraindication to transplantation, although the mortality follows a J-shaped curve, which means that both very low and very high BMI carry increased risk of graft loss and patient death. In general, a BMI >40 kg/m2 is considered an absolute contraindication because of higher risk of mortality. Therefore, this patient with BMI 35 should be acceptable.

How well did you know this?
1
Not at all
2
3
4
5
Perfectly
28
Q

A 77-year-old man with stage 4 CKD due to type 2 diabetes is admitted to the hospital with a non-ST elevation myocardial infarction. Last week, he began taking ibuprofen daily for intermittent chest pain, which subsequently worsened. Today, he presented to the emergency department with substernal chest pain and received nitroglycerin, morphine, aspirin, oxygen, and clopidogrel with symptomatic improvement. Other home medications include atorvastatin, furosemide, metformin, lisinopril, and metoprolol.
On physical examination, his blood pressure is 112/64 mm Hg, heart rate is 88/min, arterial oxygen saturation is 100% with nasal cannula at 2 L/min, and pain is 0 on a scale of 0–10. There is no peripheral edema; jugular venous pressure is normal, and heart and lung exams are normal.

Result Reference Range
Hemoglobin 10 g/dL 14–18
Blood urea nitrogen 34 mg/dL 8–20
Creatinine 2.2 mg/dL
(baseline) 0.7–1.3
Estimated GFR 28 mL/min/1.73 m2 >60
Troponin I 4 ng/mL <0.04
Diagnostic coronary angiography with possible percutaneous coronary intervention (PCI) is scheduled.
Which one of the following medications is MOST appropriate to continue?

Select one:
A. Ibuprofen
B. Atorvastatin
C. Lisinopril
D. Furosemide
E. Metformin

A

The correct answer is: B
Atorvastatin

Atorvastatin is the most appropriate medication to continue for this patient at risk of AKI due to exposure to radiocontrast (contrast nephropathy, CN).
The true incidence and impact of CN has become controversial and many experts now feel it is much less common than previously thought. However, arterial procedures are higher risk than venous procedures and underlying CKD is an important risk factor. Multiple pathophysiologic events may contribute to CN including endothelial and tubular cytotoxicity related to reactive oxygen species and vasoconstriction. In addition, medullary hypoxia is believed to occur from the high viscosity of iodinated contrast and the tendency to cause sludging in the vasa recta.
A risk prediction tool for CN has been validated in the setting of percutaneous coronary intervention (PCI). Points are given for hypotension (systolic BP <80 mm Hg for ≥1 h), use of intra-aortic balloon pump, congestive heart failure, age >75, anemia (hematocrit <39% for men, <36% for women), diabetes, volume of contrast, and serum creatinine >1.5 mg/dL. This patient, with points given for age, anemia, diabetes, and serum creatinine, would have a score of at least 14, which confers a 26% risk of CN and approximately 1% risk of dialysis.
According to cardiology guidelines, statins should be started during hospitalization for acute coronary syndrome (ACS). In addition, 2 clinical trials of patients with CKD and ACS found that rosuvastatin was associated with a reduction in the incidence of CN and major adverse cardiovascular events.
Conversely, although furosemide has been theorized to protect against CN by blocking ATP depletion in the nephron, clinical trials have shown an increased risk of CN. Loop diuretics should be avoided in the absence of signs and symptoms of fluid overload.
Metformin, traditionally discontinued at time of contrast administration, is now permitted per the 2015 American College of Radiology Manual on Contrast Media for patients with estimated GFR ≥30 mL/min/1.73 m2. However, the guidelines continue to advise that metformin should be held for at least 48 hours post-procedure for patients with eGFR <30 mL/min/1.73 m2.
Continuation of angiotensin-converting enzyme inhibitors at time of contrast administration is controversial, but a recent meta-analysis suggested evidence of harm.
Ibuprofen may both block the cardiovascular benefits of aspirin and worsen the risk of AKI; it should be discontinued.

How well did you know this?
1
Not at all
2
3
4
5
Perfectly
29
Q

A 44-year-old man with obstructive sleep apnea is referred for evaluation of possible hypertension.
He has been treated with continuous positive airway pressure (CPAP) for the past 6 years. He had a sleep study performed 2 months ago that confirmed appropriate treatment of his sleep apnea. He recently underwent a 24-hour ambulatory blood pressure monitoring (ABPM) study for evaluation as part of a research study.
He has never been diagnosed with hypertension in the past and his blood pressure has ranged from 120–130/70–80 mm Hg at multiple physicians’ office visits.
His ABPM are below (see attachment):

His medical history is remarkable for a knee injury that required surgery in his teens, depression, and hyperlipidemia. Medications include simvastatin 20 mg daily and fluoxetine 10 mg daily. He does not use non-steroidal anti-inflammatory drugs (NSAIDs), tobacco, alcohol, or illicit substances. He does not add salt to his food and tries to eat 5 servings of fruit or vegetables daily. His mother has hypertension and had a stroke in her late 60s. His father has had hypertension since age 50 and has coronary artery disease.
On examination, he appears well. His blood pressure is 126/ 76 mm Hg, pulse is 84/min, weight is 80 kg, and body mass index (BMI) is 25 kg/m2. Arteriovenous nicking is present on fundoscopy. No abdominal bruit is heard. The remainder of the exam is normal.
Laboratory data:

Result Reference Range
Sodium 138 mEq/L 136–145
Potassium 4.2 mEq/L 3.5–5.0
Chloride 100 mEq/L 98–106
Total CO2 28 mEq/L 23–30
Blood urea nitrogen 16 mg/dL 8–20
Creatinine 0.9 mg/dL 0.7–1.3
Urine albumin-to-creatinine rati 68 mg/g <30
An echocardiogram demonstrates mild left ventricular hypertrophy.
Which of the following is the APPROPRIATE management for this patient?

Select one:
A. Repeat office BP in 4–8 weeks
B. Reassurance
C. Repeat ambulatory BP in 4–8 weeks
D. Initiate lisinopril
E. Repeat sleep study

A

The correct answer is: D
Initiate lisinopril

This patient has masked uncontrolled hypertension and antihypertensive therapy should be initiated.
Masked hypertension, defined as normal office blood pressure (BP) but elevated BP by home or ambulatory monitor, is a common finding and has been documented in as many as 10–40% of patients with low or satisfactory office blood pressure measurements. Masked hypertension may be particularly common among patients with worse cardiovascular risk factor profiles.
Ambulatory blood pressure monitoring is not always available in clinical practice but can be a useful adjunct in the diagnosis and management of hypertension. When this procedure is performed, experts recommend that there are at least 20 daytime measurements and 7 nighttime measurements. Day and night intervals are defined by the patient’s diary card. Practitioners should review the average 24-hour blood pressure and the average values during each interval and determine whether a fall (“dipping”) in blood pressure was present during sleep. Conservative thresholds for the diagnosis of hypertension are as follows:

Measurement Threshold for diagnosis of hypertension
24-hour average ≥130/80 mm Hg
Awake (or daytime) average ≥135/80 mm Hg
Asleep (or nighttime) average ≥120/70 mm Hg
Nocturnal “dipping” <10% (“non-dipper”)

This patient has elevated 24-hour, awake, and asleep blood pressure, but there was evidence of dipping later in the night. The nocturnal phenomenon of dipping is believed to improve risk stratification, but it is not clear whether this also applies when the mean asleep values are high.
In this patient, there is evidence of target organ damage with left ventricular hypertrophy, albuminuria, and early hypertensive retinopathy, as well as a family history of cardiovascular disease. His sleep apnea has recently been re-evaluated and his treatment regimen confirmed. There is little to be gained by delaying treatment to repeat either office BP measurement (consistently normal over multiple visits) or the ambulatory monitor (data quality appears good with over 80% of readings in hypertensive range). Note that lifestyle modification and elimination of contributing or interfering substances is always worthwhile

How well did you know this?
1
Not at all
2
3
4
5
Perfectly
30
Q

A 68-year-old man with no significant medical history is referred for evaluation of nephrotic syndrome. He has gained 20 kg over the last 6 months and has severe lower extremity edema. He also reports generalized diffuse bone pain particularly in his spine, pelvis, and knees. The pain is dull and worse with activity.
On examination he appears unwell. His blood pressure is 102/66 mm Hg, pulse is 84/min, and temperature is 37.1°C
He has conjunctival pallor and macroglossia. There is dullness to percussion with decreased air entry at his lung bases. Heart sounds are soft and there is a grade I/VI systolic ejection murmur. Abdomen is soft and nontender. Edema is 4+ extending from the feet to the sacrum. There is no focal bony tenderness. There are no joint effusions or joint tenderness.

(see attached lab data)

Furosemide is prescribed and kidney biopsy is scheduled.

Which of the following is the MOST likely cause of this patient’s generalized bone pain?

Select one:

A. Metastases to bone
B. Hypocalcemia
C. Lytic bone lesions
D. Renal osteodystrophy
E. Vitamin D deficiency

A

The correct answer is: E
Vitamin D deficiency

The most likely cause of this patient’s generalized bone pain is severe vitamin D deficiency and osteomalacia. This patient has severe nephrotic syndrome, which is likely due to AL amyloidosis, although a kidney biopsy would be required to confirm this. This diagnosis is suggested by the combination of macroglossia, severe nephrotic syndrome, and the small M-spike.
Patients with nephrotic syndrome can develop significant vitamin D deficiency because of urinary losses of vitamin D bound to vitamin D-binding protein. Prolonged, severe vitamin D deficiency with serum values <10 ng/mL results in secondary hyperparathyroidism and osteomalacia. In this setting, patients may complain of bone pain and tenderness, and muscle weakness. The spine, pelvis, and lower extremities are most commonly involved.
It would be difficult to exclude either lytic bone lesions related to myeloma without imaging or bone marrow biopsy, but diffuse bone pain would be less likely than pain localized to the site of the lesion. Although some myelomas are non-secretory, the faint monoclonal band favors amyloidosis over widespread myeloma. Destructive bone lesions have been reported with primary amyloidosis but this occurrence is rare, tends to involve the long bones, and would likely result in localized rather than diffuse pain.
Metastatic lesions can cause bone pain, but pain would be expected to localize to the lesion(s).
Renal osteodystrophy with prolonged hyperparathyroidism can cause bone pain but does not typically occur in the absence of advanced CKD or ESRD.
Hypocalcemia is not known to cause bone pain; instead, this can cause muscle cramping and signs of latent tetany on exam with a positive Chvostek or Trousseau sign.

How well did you know this?
1
Not at all
2
3
4
5
Perfectly
31
Q

A 25-year-old, transgender, African-American woman is referred for CKD stage G3a. She is infected with HIV but has never had an opportunistic infection, and her viral load is undetectable. Last year, she decided to transition to female. Vaginoplasty and orchiectomy are planned. She has a history of depression but is not currently taking antidepressants. Her review of systems is unremarkable. Her mother had diabetes and developed “kidney problems” during an admission for a myocardial infarction.
Her medications include a co-formulated pill of abacavir, lamivudine, and dolutegravir. She also takes spironolactone 50 mg twice daily, micronized progesterone 200 mg, and estradiol 2 mg twice daily.
On physical examination, she appears well. When seated, her BP is 100/70 mm Hg with a heart rate of 90/min. Standing, her BP is 96/66 mm Hg and heart rate is 98/min. She is asymptomatic. Her body mass index is 25 kg/m2 . The heart, lungs, and abdomen are normal and there is no edema.
Laboratory studies:

Result Reference range
Sodium 138 mEq/L 136–145
Potassium 4.6 mEq/L 3.5–5.0
Chloride 101 mEq/L 98–106
Total CO2 26 mEq/L 23–30
BUN 25 mg/dL 8–20
Creatinine 1.2 mg/dL 0.5–1.1 (female)
eGFR 55 mL/min/1.73 m2 >60
Urine albumin-to-creatinine ratio 6 mg/g <30

Which of the following is the MOST appropriate next step in this patient’s management?

Select one:
A. Prescribe lisinopril
B. Obtain kidney ultrasound
C. Collect 24-hour urine for creatinine clearance
D. Provide reassurance

A

The correct answer is: D
Provide reassurance

Reassurance is the most appropriate next step in the care of this patient. A transgender woman is an individual who has a female gender identity but was assigned male at birth. Gender affirming surgery and hormone therapy result in a spectrum of secondary sex characteristics on physical examination, depending on timing, intensity, and length of therapy. Transgender women may have breast development and loss of muscle mass as well as a decline in facial and body hair. Transgender women who are registered as female in their health care system and clinical laboratory may have seemingly abnormal laboratory studies if the female upper limit of normal is used. This is true for the serum creatinine, which is influenced by muscle mass. Both the Modification of Diet in Renal Disease (MDRD) study equation formula and the CKD-EPI formula use gender and race as a proxy for muscle mass, so individuals who are listed as female but retain male physiology and muscle mass may have falsely low eGFR. If male sex is used in either formula, a 25-year-old man with this creatinine would have an eGFR greater than 90 mL/min. Thus, this patient does not have evidence of reduced kidney function. In addition, she has normal levels of urinary albumin.
This patient is taking spironolactone for its anti-androgen receptor activity. This agent is also often used in doses as high as 200 mg daily in cisgender women to limit female pattern hair loss, treat acne vulgaris, or treat hirsuitism. Spironolactone, a potassium-sparing diuretic, may contribute to this patient’s relative hypotension. Although this patient does not have orthostatic hypotension (defined as a fall of 20 mm Hg systolic or 10 mm Hg diastolic BP), she may still have mild volume depletion with mild prerenal azotemia.
Finally, her medications for HIV may also affect the serum creatinine. Dolutegravir is an integrase inhibitor used to treat HIV infection. It is metabolized by the liver, and less than 1% of the drug is excreted unchanged in the urine. A metabolite of dolutegravir is excreted in the urine via organic cation transporters in the proximal tubule and can inhibit creatinine secretion; this may increase the serum creatinine by 0.1– 0.2 mg/dL. Lamivudine and abacavir are nucleoside-reverse transcriptase inhibitors. Rapidly changing care patterns and drug development in HIV has lead to the availability of many antiviral agents sometimes offered in different co-formulations with different brand names. The National Institutes of Health provides a superb resource for HIV medications that is regularly updated and details adverse effects and drug dosing guidelines.
Angiotensin-converting enzyme inhibitors are appropriate for patients with evidence of proteinuria and/or hypertension. Neither is present in this case.
A kidney ultrasound can be used to evaluate patients with reduced kidney function, but that is not the case here. Similarly, there is no reason to perform a 24-hour urine for creatinine clearance given the other considerations outlined above.

32
Q

A 70-year-old man with a history of hypertension, diabetes, and CKD stage G4/A2 is admitted with new left hemiparesis. He awoke with the symptoms but did not present for evaluation until 8 hours later when his son returned home from work. His usual medications are insulin, lisinopril, furosemide, metoprolol, amlodipine, and aspirin.
Examination reveals BP 180/90 mm Hg and pulse 64/min. Glasgow coma scale is 14. An S4 gallop is present. There is a left facial droop, severe left arm weakness, moderate left leg weakness, and left-sided neglect.
Laboratory data are notable for the following:

Result Reference Range
Hemoglobin 11 g/dL 14–18
BUN 60 mg/dL 8–20
Creatinine 2.8 mg/dL 0.7–1.3
Glucose 170 mg/dL 70–99 (fasting)
A CT scan of the brain demonstrates an acute stroke in the territory of the right middle cerebral artery, without hemorrhage.
Which of the following options is the MOST appropriate immediate Intervention for this patient’s BP?

Select one:
A. Enalaprilat
B. Labetalol
C. No immediate intervention
D. Hydralazine
E. Nicardipine

A

The correct answer is: C
No immediate intervention

The best initial management of this patient’s BP is to withhold treatment and observe his clinical course.
Hypertension is exceedingly common both before and after ischemic stroke. Previously normotensive patients often experience a rise in BP to hypertensive levels, and patients with preexisting hypertension experience even higher BP. The rise in BP associated with hemorrhagic stroke may be even more dramatic. The occurrence and severity of hypertension is thought to represent a form of autonomic dysregulation and may relate in part to the particular brain structures affected. BP often falls spontaneously over hours to days following stroke, even without specific intervention. Optimal management remains an area of controversy. Current guidelines call for permissive hypertension for up to 72 hours unless the patient is to be treated with thrombolysis, in which case the BP should be lowered to <185/110 mm Hg. In the absence of thrombolysis, guidelines suggest that hypertension should not be treated unless the BP exceeds 220/120 mm Hg with a target of 180–220 mm Hg systolic BP and <120 mm Hg diastolic BP. Short-acting IV therapy is preferred but there are no specific data to support one agent over another. Commonly used agents include labetalol (often given in small increments) and nicardipine (given as an infusion). Enalaprilat is the IV form of the angiotensin-converting enzyme inhibitor enalapril. The hypotensive response to this agent is unpredictable and may be considerable in patients with volume depletion. There is ongoing investigation of the potential for certain agents such as magnesium or transdermal nitroglycerin to confer neuronal protection, but there is no consensus. Also, of note, trials to raise BP using vasopressors for patients with stroke and hypotension have yet to yield strong evidence.
After 24–72 hours, oral agents can be resumed or started with an initial goal of a 15% reduction in BP over 24 hours and a target of <140/90 by 7 days. Ultimately, tighter control may be warranted to reduce long term risk. All patients should be monitored and BP goals re-evaluated if there is deterioration or hemorrhagic conversion.
The Glasgow Coma Scale is a neurological scale that is used to assess the level of consciousness by evaluated eye, verbal, and motor responses. The highest score is 15 (fully conscious).

33
Q

A 65-year-old man with stage G5 CKD related to diabetes mellitus and hypertension presents with progressive dyspnea, orthopnea, and edema. The dyspnea, which developed gradually, is severe enough to cause him to avoid stairs and limit his activity. Despite adherence to a sodium-restricted diet, he has gained 4 kg over the past few months and begun to sleep in a recliner. He maintains a good appetite and does not have cough, chest pain, nausea, or pruritus. There is no history of cardiopulmonary disease and he has never smoked.
One year ago, a right radial arteriovenous fistula (AVF) was created but failed to develop. Nine months ago, a right brachio-basilic vein AVF was created with transposition of the basilic vein. After 3 months, the AVF remained patent but had limited flow, prompting angiography of both the arterial and venous systems. Angioplasty of the arteriovenous anastomosis and the outflow basilic vein was performed with good angiographic results.
Medications include lisinopril, amlodipine, furosemide, metoprolol, aspirin, multiple vitamin, and sevelamer. The patient has not required an erythropoiesis-stimulating agent to date.
Examination shows BP 145/60 mm Hg, heart rate 90/min, and oxygen saturation 95% on ambient air. Jugular venous pulsations are elevated and bibasilar crackles are present. The cardiac rhythm is regular. There is an audible murmur throughout the precordium. The intensity of the murmur falls with gentle compression of the AVF, as does the heart rate. The liver is palpable at the right costal margin. Pretibial edema is 2+. The right upper arm AVF is well-developed with a low-pitched bruit and soft thrill and a proximal diameter of 2 cm.
Laboratory data reveals normal electrolytes, normal calcium, and phosphorus. Hemoglobin has decreased from 10.5 g/dL 3 months ago to 9.5 g/dL (reference range, 14–18). Serum albumin is 3.4 g/dL (reference range, 3.5–5.5) and estimated GFR is 14 mL/min/1.73 m2, both unchanged.
Chest x-ray demonstrates pulmonary vascular congestion. EKG demonstrates voltage criteria for left ventricular hypertrophy and is unchanged from baseline. Echocardiogram demonstrates normal left ventricular and right ventricular function and no valvular disease. Pulmonary artery pressure could not be estimated.
Which of the following is the MOST appropriate strategy to identify the cause of this patient’s symptoms?

Select one:
A. Pulmonary function testing
B. Therapeutic trial of dialysis
C. Measure flow in the arteriovenous fistula and cardiac output
D. Computed tomographic angiogram (CTA) of the pulmonary circulation
E. Dobutamine stress echocardiography

A

The correct answer is: C
Measure flow in the arteriovenous fistula and cardiac output

This patient with symptoms and signs of congestive heart failure (CHF) following creation of an AVF should undergo measurement of flow in the AVF.
Dyspnea on exertion is a nonspecific symptom that can indicate any of a number of disease processes including CHF, coronary artery disease (CAD), anemia, pulmonary hypertension, pulmonary embolism, intrinsic pulmonary disease, or even deconditioning. This patient’s symptoms emerged in the months following endovascular revision of an AVF and therefore could be related.
AVF have expected hemodynamic consequences including increased cardiac output, reduced peripheral vascular resistance, and increased sympathetic tone (resulting in increased heart rate, stroke volume, and contractility). These changes are usually tolerated among patients with good ventricular function but can result in high-output CHF among patients with limited cardiac reserve.
Rarely, patients with preserved ventricular function can experience high-output CHF if AVF flow is high (>2L/min). A ratio of the access flow (QA) to the cardiac output (CO) > 0.3 has been identified as a potential risk factor for CHF. If high-output CHF is identified, flow reduction by surgical revision of the AVF is often successful in terms of both symptom improvement and access patency.
Patients with CKD are predisposed to CAD and this should be considered in the differential diagnosis. However, this patient has no wall motion abnormalities on echocardiography and no changes on EKG to suggest that he suffered a myocardial infarction. CAD without infarction could explain DOE as an “anginal equivalent,” but would not explain the orthopnea or edema. Therefore, dobutamine stress echocardiography would not be the first diagnostic study in this patient.
Pulmonary function testing would be reasonable if there were abnormal findings on pulmonary exam, imaging, or a significant smoking history, but orthopnea and edema would not be typical symptoms unless there was also cor pulmonale, which was not seen on echocardiography.
Occult pulmonary thromboembolic disease can occur, and there is increasing recognition that thrombectomy of dialysis grafts and AVF can produce pulmonary emboli with either immediate or remote consequences. In this case, symptoms did not emerge until months after the endovascular procedure, which was angioplasty rather than thrombectomy. CT angiography of the pulmonary circulation could be considered if other testing is not diagnostic.
A therapeutic trial of dialysis would be a reasonable strategy once other reversible causes of decline have been excluded. This patient has symptoms and signs of expansion of extracellular fluid volume, which can occur as kidney function declines. However, this patient does not have other uremic symptoms or signs and his estimated GFR is unchanged. Because the institution of HD is associated with a decline in residual renal function, a “therapeutic trial” may result in a permanent commitment to renal replacement therapy. Therefore, initiation of dialysis would not be the first choice.

34
Q

A 64-year-old woman is admitted for severe right flank pain, nausea, and vomiting that began 28 hours ago. The pain is constant in nature.
Her past medical history is notable for hyperlipidemia and diabetes mellitus. She has a 30 pack-year smoking history but quit 2 years ago.
On physical examination, her temperature is 38°C, BP is 178/92 mm Hg, and heart rate is 86/min and irregular. She has diffuse abdominal tenderness.
Laboratory data:
Result Reference range
Leukocytes 15,000/uL 4000–11,000
Hemoglobin 12 mg/dL 12–16
Platelets 156,000/uL 150,000–450,000

Sodium 138 mEq/L 136–145
Potassium 4.6 mEq/L 3.5–5.0
Chloride 101 mEq/L 98–106
Total CO2 26 mEq/L 23–30
BUN 25 mg/dL 8–20
Creatinine 1.2 mg/dL 0.5–1.1
Amylase 342 U/L 25–125
Lipase 102 U/L 10–140
Lactate dehydrogenase 1234 U/L 80–225

Urinalysis is positive for blood and protein and microscopy shows 50 RBC/HPF and 3-5 WBC/HPF.

CT scan of the abdomen with contrast shows a wedge-shaped perfusion defect in the right kidney with mild perinephric stranding without hydronephrosis. Calcification and atherosclerotic disease is present in the abdominal aorta and there is mild narrowing of the orifices of the renal arteries. The gallbladder contains gallstones but no pericholecystic fluid or abnormal enhancement of the gallbladder wall is identified.
Which of the following is the MOST appropriate next step in this patient’s management?

Select one:
A. Angiography with intervention
B. Right upper quadrant ultrasound
C. Tamsulosin
D. Ciprofloxacin
E. Heparin

A

The correct answer is: E
Heparin

This patient, with an apparent renal infarction, should be treated with heparin for anticoagulation to prevent further thromboembolic complications.
Most renal kidney infarction have risk factors for thromboembolic events, such as atrial fibrillation, which may have been previously unrecognized. Although the renal circulation is one of the richest in the body, the kidneys are vulnerable to infarction from thromboembolism because the segmental arteries are functional “end arteries” that serve as the only supply of oxygenated blood to their particular segment of the kidney.
Patients typically present with the sudden onset of flank pain. Nausea and vomiting are also common symptoms, while fever and abdominal or flank tenderness are common signs. Even though the kidneys are in the retroperitoneum, inflammation can cause irritation of the peritoneum. Laboratory data is typically notable for a leukocytosis, an elevated lactate dehydrogenase (LDH), and hematuria. Case series have noted that the mean LDH is in the 600–800 U/L range, although it may be lower if there is a delay in diagnosis. Mild elevations in amylase and lipase are also common. Because renal infarction is relatively rare, pain may be erroneously attributed to a more frequently encountered problem such as renal colic, pyelonephritis, pancreatitis, or a hepatobiliary source.
The treatment of a renal infarction is usually conservative when symptoms have been present for >24 hours. Thrombectomy or thrombolysis can be considered if the event is more recent and there may be an opportunity to salvage tissue. Angioplasty, with or without stenting, can be performed if there is an underlying anatomic abnormality, such as fibromuscular dysplasia or dissection. In this case, the presence of atrial fibrillation, which likely accounts for the renal infarction, as well as the delay in presentation suggest that anticoagulation is preferable to angiography. Longer-term therapeutic anticoagulation should be considered for patients with risk for additional events, such as this patient with atrial fibrillation or patients with thrombophilia.
This presentation of atheroembolism to the kidney differs from that of renal infarction. Atheroembolism affects smaller vessels including arterioles, usually resulting in glomerular ischemia or interstitial nephritis from inflammatory cell infiltration rather than large territory infarction. Atheroembolism, usually results in a subacute but progressive decline in kidney function, but more severe episodes can result in AKI. Other clinical manifestations include livedo reticularis, purpura, digital ischemia, bowel ischemia, and pancreatitis. Eosinophilia and C3 hypocomplementemia may be seen in laboratory investigation.
Tamsulosin, an α1-adrenergic receptor antagonist, is often used for “medical expulsive therapy” for patients with renal colic, in order to promote relaxation of the ureter and hasten stone passage. The symptoms related to calculi of the urinary tract can mimic those of renal infarction, and CT with intravenous contrast can miss small stones. Antibiotics can be given for patients with pyelonephritis which may also present with flank pain, fever and pyuria. Gallstones are often found incidentally on abdominal CT and are a risk factor for cholecystitis, but other features of this condition were lacking. Right upper quadrant ultrasound and/or hepatobiliary scanning may be useful for patients with a clinical history and data consistent with cholecystitis or other biliary disorders. In this case, the imaging and laboratory results in this case favor the diagnosis of renal infarction.

35
Q

A 28-year-old man is referred for evaluation of proteinuria. He moved to the United States from China 6 months ago to pursue graduate studies. One month ago, he noticed bilateral ankle swelling, which has steadily worsened. Over the month he has gained 8 kg.
Medications are limited to a daily multivitamin. On examination, his BP is 132/78 mm Hg and pulse is 82/min. There is 2+ pitting edema to the thighs and periorbital edema.
Urine dipstick is positive for 3+ albumin and 1+ blood. Urine microscopy shows occasional RBCs and oval fat bodies. Abdominal ultrasound is normal.

Result Reference range
Hemoglobin 14.0 g/dL 14–18
Creatinine 0.88 mg/dL 0.7–1.3
Albumin 2.6 g/dL 3.5–5.5
24-hour urine protein 5300 mg <100
Hepatitis B virus (HBV) surface antigen (HBsAg) positive
HBV surface antibody (HBsAb) negative negative
HBV core IgM antibody negative negative
HBV core IgG antibody positive negative
Hepatitis C antibody negative negative
Antinuclear antibody negative ≤1:40
Interferon-g release assay negative negative
A kidney biopsy reveals thickened basement membranes on light microscopy. Immunofluorescence shows diffuse granular staining for IgG along the basement membranes, and electron microscopy localizes these deposits to the subepithelial space. Staining for phospholipase A2 receptor (PLA2R) is negative.
Which of the following studies should be performed NEXT in this patient?

Select one:
A. Serum free light chain assay
B. Anti phospholipase A2 receptor antibody
C. Chest x-ray
D. Hepatitis B e-antigen
E. Abdominal CT

A

The correct answer is: D
Hepatitis B e-antigen

This patient should undergo testing for hepatitis B e antigen (HBeAg) to confirm that the membranous nephropathy is secondary to chronic hepatitis B infection.
Membranous nephropathy is the most common kidney disease associated with hepatitis B infection, followed by MPGN and polyarteritis nodosa. Circulating HBeAg is specifically associated with hepatitis B-associated membranous nephropathy, and subepithelial HBeAg deposition is implicated in the pathogenesis of this disease. Approximately 60–80% of patients with membranous nephropathy associated with hepatitis B will have circulating serum HBeAg; the remainder will have antibodies to HBeAg.
This patient’s hepatitis B profile is consistent with chronic infection, which is the most commonly observed pattern in adults from regions in which hepatis B is endemic. Membranous nephropathy related to hepatitis B, while rare in United State natives, should be considered in immigrants or travelers from hepatitis B-endemic regions presenting with proteinuria. Most cases present in childhood. Adult onset hepatitis B-associated membranous nephropathy carries a higher risk of progressive kidney disease.
Antibodies against the phospholipase A2 receptor (PLA2R) are seen in 70% of cases of idiopathic membranous nephropathy, whereas this test is typically negative in hepatitis B-associated disease. Staining of kidney biopsy tissue for anti-PLA2R is more sensitive than the serologic assay for this antibody, so if the antibody is absent on the kidney biopsy, the serologic test will be also negative.
Because membranous nephropathy has been associated with malignancy, age-appropriate screening is recommended. In addition, if there are signs or symptoms that raise concern for a thoracic or abdominal neoplasm, a chest radiograph and/or abdominal CT should be performed. In this case, there are specific age-appropriate cancer screens and no symptoms or exam findings to warrant this investigation.
Serum free light chains are useful to screen for a monoclonal gammopathy, but membranous nephropathy would be an extremely rare presentation of this disorder.

36
Q

A 58-year-old man with autosomal dominant polycystic kidney disease (ADPKD) develops uremic symptoms and is started on CAPD. Five weeks later, he presents to the clinic complaining of a bulge in the right groin. He first noticed this bulge 1 week earlier after moving furniture in his living room. Subsequently, the bulge has increased in size and has become slightly painful and tender. Uremic symptoms have resolved and he is feeling otherwise well.
He currently performs 4 × 2.5 L CAPD exchanges daily and a 1-L dwell overnight. Urine volumes remain normal. He has not yet undergone adequacy testing or performed a peritoneal equilibration test (PET).
Current medications include furosemide 80 mg twice daily, lisinopril 10 mg daily, renal multivitamins, calcium acetate 1334 mg with meals, aspirin 81 mg daily, simvastatin 40 mg daily, and lactulose 30 mL as needed for constipation.
On examination, he appears well. His BP is 138/82 mm Hg, pulse is 74/min, and temperature is 36° C. His abdomen is distended, but soft and nontender. His kidneys are easily palpable. There is a protuberant hernia in his right groin that is nontender and easily reducible. The catheter exit site shows no erythema or purulent drainage. The remainder of the examination is normal.
A diagnosis of a PD associated inguinal hernia is established and surgical repair is planned.

Which of the following is the BEST next step in management?
Select one:

A. HD via a catheter for 6 weeks after hernia repair, then resume PD
B. Change to HD permanently after hernia repair
C. Continue PD without surgical repair unless symptoms progress

D. Resume low-volume supine PD 2 days after hernia repair
E. Manage conservatively without dialysis for 2 weeks after hernia repair then resume PD

A

The correct answer is: D
Resume low-volume supine PD 2 days after hernia repair

Surgical repair followed by resumption of PD after 2 days using low volumes and supine position is the best treatment for this patient.
The development of an inguinal hernia in a patient with underlying ADPKD is not unexpected, and multiple approaches to perioperative dialysis can be considered. If the patient already has permanent blood access that was established to serve as back-up, then HD can be performed until surgical healing is sufficient. In the absence of back-up blood access, it is possible to manage patients without resorting to HD. One successful protocol described by the University of Toronto calls for withholding PD for 2 days and then resuming with low dialysate volumes with the patient supine during exchanges. PD is then gradually advanced over the next month back to the preoperative regimen. The regimen used is as follows: PD thrice weekly for 10-hour sessions using low volumes for 2 weeks, then PD daily using 1.5 L per exchange for 2 weeks, then resume preoperative regimen.
Withholding PD for 2 weeks would risk return of uremic symptoms, but the patient should tolerate 2 days without dialysis well as he retains some residual kidney function.
Withholding surgery is unlikely to be successful because the hernia is already enlarging and becoming more symptomatic. If complications such as incarceration or obstruction occurred, the surgical procedure required could be more extensive and adversely affect the long-term success of PD for this patient.

37
Q

A 70-year-old man with ESRD caused by focal and segmental glomerulosclerosis (FSGS) presents for a routine clinic follow-up 7 years after his living donor kidney transplant. Over the preceding 6 months, he has experienced an increase in fatigue and lost 8 kg. Review of systems is otherwise negative.
His immunosuppression consists of tacrolimus 2 mg twice daily, mycophenolate mofetil 1000 mg twice daily, and prednisone 5 mg daily. Induction therapy included thymoglobulin. He has not experienced any rejection episodes post-transplant. Pretransplant viral serologies for the donor and the patient (recipient) were IgG positive for both CMV and EBV (D+/R+).
On physical examination, his temperature is 36.4°C, BP is 155/95 mm Hg, and pulse is 72/min. Jugular veins are not distended. There are no palpable lymph nodes, and the lungs are clear. An S4 gallop is present. Abdominal exam reveals tenderness to palpation of the left lower quadrant over transplant surgical scar but there is no guarding, rebound, overlying erythema, or bruit. The extremities have no edema.
Laboratory data shows:

Result Reference range
Leukocytes 8000/µL 4000–11,000
Hemoglobin 11 g/dL 14–18
Creatinine 1.6 mg/dL
(1.2 mg/dL 6 months ago) 0.7–1.3
Tacrolimus trough level 10 ng/mL 3 months post-transplant target 5–10
EBV polymerase chain reaction negative negative
Urine protein-to-creatinine ratio 0.4 mg/mg <0.2
Urinalysis shows 1+ protein and microscopy is unremarkable.
Anti-human leukocyte antigen (HLA) antibody testing is borderline positive, but no donor-specific antibody is detected on single antigen bead assay.
Kidney transplant ultrasound reveals an enlarged kidney with slightly increased echogenicity and no hydronephrosis. Resistive index is 0.9.

Which one of the following is the MOST likely diagnosis?

Select one:
A. BK virus infection
B. Chronic antibody-mediated rejection
C. FSGS recurrence
D. Post-transplant lymphoproliferative disorder
E. CMV infection

A

The correct answer is: D
Post-transplant lymphoproliferative disorder

Post-transplant lymphoproliferative disorder is the most likely diagnosis for this patient.
Significant weight loss and fatigue in a patient more than 5 years after kidney transplantation warrants investigation for a malignancy until or unless an alternative explanation is identified. The most prevalent malignancies after kidney transplant include skin cancer, Kaposi sarcoma, renal cell carcinoma, and post-transplant lymphoproliferative disorder (PTLD).
PTLD is a heterogenous group of diseases characterized by abnormal lymphoid proliferation following organ transplantation. The most common type is a non-Hodgkin lymphoma with approximately 50% exhibiting extra-nodal involvement including the allograft. PTLD usually has a bimodal presentation, peaking at the first year and then 7–10 years after transplant. Presentation within the first year after transplantation is strongly associated with EBV infection, while later PTLD is EBV-negative in approximately 40% of cases. The main risk factors for PTLD include recipient EBV seronegative status, older age, and higher degree of immunosuppression (including the use of T-cell depletion therapy and higher levels of maintenance immunosuppression, in particular tacrolimus). The use of belatacept, a selective T-cell costimulatory blocker, for maintenance therapy for patients who are EBV-naïve has also been associated with higher risk of PTLD.
The diagnosis of PTLD requires a tissue biopsy. In this case, because there are no other localizing signs, a kidney biopsy should be performed so that additional studies for clonality, cell type characterization, and EBV encoding region (EBER) in situ hybridization can be performed (EBER-1 and EBER-2 are the most abundant EBV RNAs in infected cells). Additional investigation to determine the extent of disease should be done, and often includes imaging with positron emission tomography (PET)-CT. Treatment options range from immunosuppression reduction to rituximab to combination chemotherapy with rituximab, depending on the malignant nature and extent of the PTLD.
Chronic antibody-mediated rejection is unlikely in the absence of circulating donor-specific antibodies, particularly with good maintenance immunosuppression. Chronic rejection tends to present with proteinuria rather than weight loss.
CMV infection can present with fatigue, weight loss, and graft dysfunction. However, this opportunistic infection tends to occur among patients who are CMV-naïve, or concomitantly with other systemic infections. Presentation may include fever, chills, diarrhea, and leukopenia, none of which symptoms or signs is present in this patient. CMV occurs more frequently within the first year after transplant, particularly after discontinuation of CMV prophylaxis. Late reactivation is uncommon in CMV IgG-positive recipients.
BK virus, named after the first patient described with this infection, tends to cause symptoms early after transplantation or after rejection treatment because of more intense immunosuppression. Patients with BK nephropathy are usually asymptomatic and are identified by an increased creatinine concentration, but they may also have urologic involvement with hematuria and ureteral strictures.
FSGS recurrence tends to occur within days to weeks of the kidney transplant and presents with nephrotic-range proteinuria, which is absent in this case.

38
Q

A 72-year-old woman with rheumatoid arthritis and stage 4 CKD attributed to diabetic nephropathy is hospitalized with confusion. Despite significant chronic pain from her rheumatoid arthritis and diabetic neuropathy, she remains active and continues to work as a teacher’s aide. Two days ago, she became ill with acute symptoms of gastroenteritis, including nausea, diarrhea, poor intake, and low-grade fever. On the day of admission, her husband had difficulty waking her and found her to be confused and lethargic. Her fingerstick blood sugar was 180 mg/dL (reference range, 70–99 [fasting]).
Her medical history is also significant for hypertension, gout, postherpetic neuralgia, and carpal tunnel syndrome.
Medications include insulin, oxycodone 5 mg three times daily as needed, aspirin 81 mg daily, tramadol 100 mg twice daily, gabapentin 300 mg three times daily, lisinopril 20 mg daily, and chlorthalidone 25 mg daily.
On examination, she is confused and inattentive. Her BP is 96/ 76 mm Hg, pulse is 74/min, and temperature is 37°C. She has decreased tissue turgor and has prominent myoclonic twitching of her upper and lower extremities, but exam is otherwise normal.

Laboratory data (attachment):

Which of the following is the MOST likely cause of her symptoms?

Select one:
A. Uremia
B. Salicylate
C. Oxycodone
D. Tramadol
E. Gabapentin

A

The correct answer is: E
Gabapentin

Gabapentin is the most likely cause of the symptoms in this patient. Gabapentin is cleared entirely by renal excretion and therefore accumulates when kidney function declines, as in this patient who developed AKI in the setting of volume depletion from a gastrointestinal illness. At baseline, her dose of 300 mg thrice daily was already higher than recommended for a patient with stage 4 CKD (300 mg twice daily would have been more appropriate) and resulted in toxicity when her kidney function declined. Myoclonus is a typical symptom of gabapentin toxicity. Gabapentin is cleared by HD, which can be used for treatment.
Myoclonic jerks have been reported to occur with opioids, but the propensity for this side effect varies with the specific metabolism and activity of the parent compound and its metabolites. There are only rare reports of oxycodone causing myoclonus, and in some of these cases, other agents (such as methadone), were co-administered. Only a small fraction of oxycodone is excreted unchanged in the urine, but the half-life of this agent and its metabolites are prolonged when kidney function is severely reduced. Thus, while oxycodone may certainly contribute to the altered mental status observed in this case, it is not the best explanation for the myoclonus.
Myoclonus can be seen with uremia; however, this would be a diagnosis of exclusion after gabapentin toxicity was addressed.
Tramadol is also largely renally cleared; 30% is excreted unchanged. Labeling recommendations suggest that the dosing should be 50–100 mg daily, but some experts favor the lower end of this scale. Expected side effects include sedation and confusion but not myoclonus.
Salicylate toxicity is a multiphasic syndrome that is consistent with some features in this case, including nausea, altered mental status, and an acid-base disorder. However, this patient’s dose of 81 mg daily is too low to cause toxicity and there is no reason to suspect an intentional or accidental overdose. Salicylate toxicity that is sufficiently severe to cause altered mental status typically leads to an anion gap metabolic acidosis and a respiratory alkalosis. This patient has only a small anion gap and also has hyperphosphatemia, which may contribute to the anion gap. Measurement of the pH would be helpful to clarify the acid-base disorder, but because the anion gap is relatively small and the bicarbonate is only mildly decreased, it is unlikely that a severe acid-base disorder is present. Rare cases of falsely normal anion gap have been reported with salicylate toxicity. However, in these instances, the chloride was falsely elevated secondary to interference from high concentrations of salicylate, and the bicarbonate values were still low.

39
Q

A 50-year-old man with ESRD secondary to diabetes returns for follow-up. He had a living-unrelated kidney transplant 12 months ago. Recently, an increase in his serum creatinine level prompted a kidney allograft biopsy.
His background includes a pre-transplant Panel Reactive Antibody (PRA) of 60%. He received a kidney from a close friend with a 4 out of 6 human leukocyte antigen (HLA) mismatch (A2 B2 DR0). Donor-specific antibodies and flow crossmatches against T- and B-cells were all negative at time of transplant. His current immunosuppression regimen includes tacrolimus, mycophenolate, and prednisone.
His serum creatinine has increased from a baseline of 1.3 to 2.6 mg/dL (reference range, 0.7–1.3) over 1 month, and he has developed a new circulating donor-specific antibody against HLA antigen B35. A kidney allograft biopsy shows significant peritubular capillaritis and glomerulitis but no tubulitis. Staining for C4d is positive in the peritubular capillaries.
Which of the following is the MOST appropriate treatment for this patient?

Select one:
A. Eculizumab and increased oral corticosteroids
B. Rituximab and pulse methylprednisolone
C. Anti-thymocyte globulin
D. IV immunoglobulin and plasmapheresis
E. Cyclophosphamide

A

The correct answer is: D
IV immunoglobulin and plasmapheresis

This patient has both clinical and histopathologic features of antibody-mediated rejection (AMR) and should receive treatment that includes IV immunoglobulin (IVIG) and plasmapheresis.
AMR is a form of rejection that is mediated by circulating alloantibodies against the transplanted kidney, which trigger complement activation and Fc-mediated cytotoxicity. AMR can be acute or chronic in nature and may relate to either preexisting or de-novo antibodies. In contrast to cellular rejection, which is primarily T-cell-mediated and targets the tubules, AMR targets the endothelium of peritubular capillaries and glomeruli. Sensitized patients with preformed anti-HLA antibodies are at higher risk of developing AMR, even in the absence of a positive crossmatch at the time of transplant. Diagnosis of AMR requires evidence of inflammation involving the capillaries on kidney biopsy and the presence of circulating donor-specific antibodies. The majority of cases, but not all, will also have evidence of complement activation through C4d positivity on immunofluorescence. Patients with AMR present with allograft dysfunction, proteinuria, or both. Although there is variation of practice between institutions, a mainstay of management for acute AMR involves the removal and neutralization of circulating alloantibodies with plasmapheresis and IVIG.
An increase in corticosteroid dose, or even pulse corticosteroids, is often part of the treatment for acute cellular rejection, alone or in combination with other immunosuppressive therapy depending on the severity of the rejection. The role of steroids in the management and prevention of isolated AMR is less well established. On most occasions, patients maintain their existing antirejection regimen and specific anti-AMR therapy is added. When a concomitant component of acute cellular rejection is present, pulse-dose steroids are frequently given.

Eculizumab, an anti-C5 antibody, is insufficient treatment for AMR by itself, because alloantibodies may continue to injure the endothelium through complement-independent mechanisms. It is sometimes added as “rescue therapy” if the AMR is unresponsive to other measures.
Rituximab-mediated depletion of B-cells that express CD20 is a reasonable adjunctive therapy but would be used in addition to IVIG and plasmapheresis, rather than as monotherapy.
Anti-thymocyte globulin targets primarily T-cells and is indicated in severe cellular rejection, which is not present in this case.
Cyclophosphamide is an effective immunosuppressive therapy that suppresses both T- and B-cells and is used for the treatment of aggressive glomerular diseases but has no defined role in the treatment of AMR.

40
Q

A 56-year-old man with ESRD due to hypertensive nephrosclerosis is evaluated for hypotension that began approximately 25 minutes after HD was initiated. This is the patient’s first treatment at this facility because he is visiting from out of state.
His medical history includes coronary artery disease, for which he had a cardiac stent implanted 1 year ago. He receives dialysis via a radiocephalic arteriovenous fistula (AVF) and receives heparin, 4000 units, as a bolus prior to the initiation of each treatment. His medications include lisinopril, metoprolol, amlodipine, aspirin, sevelamer, and renal vitamins. He is active and walks his dog regularly. His vital signs immediately before dialysis was initiated were unremarkable, and his ultrafiltration goal was set for 3 L based on his interdialytic weight gain. The AVF had a normal appearance.
Dialysis began uneventfully 25 minutes ago, but now, the patient feels unwell. His symptoms include nausea, back pain, and dyspnea. Vital signs are as follows: BP 110/60 mm Hg, heart rate 120/min but regular, temperature 37.9°C, and respiratory rate 22/min. His mental status is normal, skin warm and dry without rash, heart regular and tachycardic, lungs clear, abdomen soft with normal bowel sounds, and fistula normal.
Ultrafiltration is discontinued and nasal oxygen is provided.
Which of the following is the MOST appropriate next step in the management?

Select one:
A. Discontinue dialysis with immediate return of blood
B. Administer isotonic saline
C. Administer sublingual nitroglycerin
D. Continue dialysis with close observation
E. Administer cefepime and vancomycin

A

The correct answer is: D
Continue dialysis with close observation

This patient is most likely experiencing a type B dialyzer reaction, for which the appropriate management consists of supportive measures and close observation while continuing dialysis.
Type B dialyzer reactions typically occur 15–30 minutes after treatment is initiated. Characteristic symptoms include chest and back pain, nausea, and hypotension, all of which gradually subside even if dialysis is continued. Complement activation via the alternative pathway by the dialyzer membrane is thought to be the primary cause. These reactions were more common when cellulosic dialyzers were in common use but are infrequent in the era of biocompatible dialyzer membranes.
Type A reactions occur soon after initiation of dialysis, usually within 5–10 minutes. Symptoms range from mild allergic-type symptoms to life-threatening bronchospasm, hypotension, and cardiac arrest. The most severe type A reactions are triggered by an IgE-mediated reaction to substances such as ethylene oxide that leach from the dialyzer or circuit. Patients with allergies or eosinophilia may be predisposed. When a type A reaction occurs, dialysis should be discontinued without returning the patient’s blood from the circuit.
Patients taking angiotensin-converting enzyme inhibitors and exposed to a dialyzer composed of acrylonitrile and sodium methallyl sulfonate (AN-69) can experience a similar syndrome related to enhanced generation and impaired metabolism of bradykinin. Contamination of the dialysis circuit by bacterial products is yet another mechanism that can provoke type A reaction-like signs and symptoms. Because of the life-threatening nature and mechanism of these reactions, immediate cessation of dialysis without returning the patient’s blood is indicated. Other treatments that may be necessary include ventilatory support, epinephrine, volume expansion, corticosteroids, and antihistamines.
Although symptoms of type A and B reactions can overlap, in this case, the timing favors a type B reaction and justifies continuing treatment with close supervision.
The differential diagnosis also includes sepsis, but it would be premature to administer broad-spectrum antibiotics without more evidence to suggest a bacterial infection. Cardiac ischemia can provoke dyspnea and hypotension, but back pain would be relatively uncommon and hypotension would be exacerbated by sublingual nitroglycerin. Volume expansion with isotonic saline would not be unreasonable but may not be necessary if the observed symptoms and low BP resolve rapidly.

41
Q

A 38-year-old male found unconscious by his roommate is brought to the emergency department. His medical history is unremarkable, and he is not known to be taking any prescription medications.
His physical examination demonstrates an obtunded individual with a Glasgow Coma Scale score of 9 (eye 2, verbal 3, motor 4). His BP is 135/78 mm Hg, pulse 78/min, respiratory rate 29/min, saturation 98% on ambient air, and temperature 38.1oC. His physical examination is otherwise unremarkable, and he has no focal neurological signs.
His laboratory parameters include the following:

Result Reference Range
White blood cell count 11,500/µL 4000–11,000
Hemoglobin 13.8 g/dL 14–18
Platelets 251,000/µL 150,000–450,000

Sodium 143 mEq/L 136–145
Potassium 3.6 mEq/L 3.5–5.0
Chloride 106 mEq/L 98–106
Total CO2 14 mEq/L 23–28
Blood urea nitrogen 24 mg/dL 8–20
Creatinine 0.8 mg/dL 0.7–1.3
Glucose 110 mg/dL 70–99 (fasting)
Serum osmolality 304 mOsm/kg 275–295
Toxicology Pending

Arterial blood gas
pH 7.47 7.38–7.44
PaCO2 20 mm Hg 35–45

Urinalysis: pH 5, specific gravity 1.010, dipstick negative.

In addition to management of the airway, breathing, and circulation (“ABCs”), which of the following is the BEST initial management of this patient?

Select one:
A. Acetazolamide
B. Continuous renal replacement therapy
C. Dextrose
D. Insulin
E. Fomepizole

A

The correct answer is: C
Dextrose

This patient, who presents with features of salicylate poisoning and obtundation, should receive IV dextrose to treat for possible neuroglycopenia, a complication of salicylate poisoning.
The presence of a respiratory alkalosis in combination with an anion gap metabolic acidosis in a patient with altered mental status is consistent with salicylate poisoning, and this should be the working diagnosis until more data are available.
Salicylates uncouple oxidative phosphorylation and lead to an increase in endogenous lactic acid production (type B lactic acidosis). In addition, salicylates cross the blood-brain barrier and stimulate the medullary center to cause tachypnea resulting in a respiratory alkalosis. Hyperpyrexia can also occur. Salicylate toxicity results in a decreased level of consciousness from multiple mechanisms, including direct central nervous system toxicity, cerebral edema, and neuroglycopenia. Neuroglycopenia can occur despite the presence of a normal serum glucose level and should be empirically treated. Both experimental salicylate poisoning in animals and case reports in humans demonstrate remarkable improvement in neurologic status with IV dextrose administration in this setting. Other treatments for salicylate poisoning include urinary alkalinization to enhance excretion, which should be accomplished with sodium bicarbonate even in the presence of respiratory alkalosis.
Acetazolamide, although effective at urinary alkalinization, is contraindicated because acidification of the blood may increase penetration of salicylate into the brain and other tissues.
HD is indicated for severe poisonings with AKI, high salicylate levels, or pulmonary or cerebral edema. Acute HD is far superior to continuous renal replacement therapy for salicylate clearance, and the latter modality should be considered only if HD is not available or there is a contraindication to HD.
Diabetic ketoacidosis can lead to an anion gap metabolic acidosis and can rarely cause a respiratory alkalosis, but volume depletion, hyperglycemia, ketonuria, and glycosuria would be expected. Here, there is no indication for insulin, which could lead to hypoglycemia and hypokalemia, because the potassium level is also at the low end of normal.
Fomepizole is indicated for management of toxic alcohol ingestions, which are always an important consideration in patients presenting with altered mentation and metabolic acidosis with an elevated anion gap. The diagnosis would be further suggested by an elevated osmolar gap (OG). In this case, the OG is normal, which makes a toxic alcohol ingestion unlikely. The OG can be calculated as follows:
OG = measured plasma osmolality (Posm) minus calculated plasma osmolality:
Calculated Posm = 2 × [Na] + glucose (mg/dL)/18 + BUN (mg/dL)/2.8
Here, the calculated Posm = 2 × 143 + 110/18 + 24/2.8 + 0 = 300 mmol/kg
OG = 304 – 300 = 4 mOsm/kg.
This is within the expected range (<10). It is worth noting that the OG narrows as the toxic alcohol is metabolized to toxic metabolites, so measurement of the OG is less helpful when the presentation is long after the ingestion.
If thiamine deficiency is a concern, concomitant thiamine therapy should be given at the time of dextrose administration to prevent the development of Wernicke encephalopathy

42
Q

A 45-year-old man with stage 5 CKD due to diabetic nephropathy presents for follow-up. Despite increased fatigue, he continues to work full-time. His appetite is fair, weight is stable, and he does not have nausea, hiccups, dyspnea, chest pain, or diarrhea. He has chronic lower extremity swelling that is unchanged. His medications are insulin, losartan, indapamide, aspirin, and atorvastatin.
His BP is 135/90 mm Hg. Heart sounds are normal without pericardial rub or murmur. Lungs are clear. He has 1+ peripheral edema. Mentation is normal, and there is no asterixis. A radiocephalic arteriovenous fistula (AVF) placed 2 months previously appears mature.
To plan his availability for his employer, the patient would like to know whether he should begin dialysis.
Laboratory data (all parameters have been stable over the past 8 weeks):

Result Reference Range
Hemoglobin 9.8 g/dL 14–18

Sodium 137 mEq/L 136–145
Potassium 4.7 mEq/L 3.5–5.0
Chloride 105 mEq/L 98–106
Total CO2 19 mEq/L 23–28
Blood urea nitrogen (BUN) 70 mg/dL 8–20
Creatinine 5.5 mg/dL 0.7–1.3
Estimated GFR (eGFR) 12 mL/min/1.73 m2 >60
Urine albumin-to-creatinine ratio 900 mg/g <30
How should this patient BEST be counseled regarding the timing of dialysis initiation?

Select one:
A. Initiate dialysis at this time
B. Initiate dialysis when when BUN >100 mg/dL
C. Initiate dialysis when the AVF has been in place for 3 months
D. Initiate dialysis when eGFR <10 mL/min/1.73 m2
E. Initiate dialysis when symptoms are more severe

A

The correct answer is: E
Initiate dialysis when symptoms are more severe

This patient with stage 5 CKD but without significant uremic symptoms should be advised to delay initiation of dialysis until symptoms are more severe. It is important that this patient be followed closely for the development of uremic signs or symptoms so that dialysis can be initiated when indications emerge.
The timing of initiation of dialysis for patients with advanced CKD is controversial. One prospective trial that compared early and late initiation of dialysis concluded that early initiation of dialysis (eGFR 10–14 mL/min/1.73 m2) was not associated with better survival or clinical outcomes than late initiation (5–7 mL/min/1.73 m2). It should be noted, however, that in the late initiation group, 75% of the patients started dialysis with eGFR >7 mL/min/1.73 m2 because of uremic symptoms. A retrospective observational study that included 652 patients with advanced CKD also showed no significant difference in survival between early (eGFR >10 mL/min/1.73 m2) and late (eGFR <10 mL/min/1.73 m2) initiation of dialysis. Furthermore, a study based on data from the United States Renal Data System for 84,654 patients who were 67 years or older and initiated dialysis early (eGFR >10 mL/min/1.73 m2) had greater rates of all-cause mortality, cardiovascular mortality, and all-cause hospitalization. These studies suggest that initiation of dialysis based solely on laboratory data and eGFR is not warranted and could be detrimental in certain populations. It is suggested that the decision to initiate dialysis should be individualized and based on clinical indications rather than on the laboratory parameters alone. The National Kidney Foundation Kidney Disease Outcomes Quality Initiative (KDOQI) Hemodialysis Adequacy Work Group recommends that the decision to initiate maintenance dialysis therapy should be based primarily on the presence of symptomatic uremia, protein energy wasting, metabolic abnormalities, and volume overload rather than a specific level of kidney function.
Initiation of HD via an AVF, compared to a tunneled catheter, has been demonstrated to be associated with fewer complications and lower costs, so it is certainly to this patient’s advantage that an AVF has been established. The time to maturation of an AVF is commonly expected to be 12 weeks, but the actual time depends on patient and surgical factors. It may be possible to successfully cannulate a fistula by 6 weeks (or even sooner) if it demonstrates adequate size, flow, and wall thickness and its location is superficial enough to permit cannulation. A combination of clinical and ultrasound evaluation is often performed. The expertise of the cannulator also influences decisions around first cannulation. For this patient, without compelling reasons to start dialysis, a longer maturation time may improve fistula success, even if it appears mature on clinical grounds. Some studies have found that creation of an AVF is associated with a reduction in the rate of decline in GFR, raising the fascinating possibility that early creation of an AVF may actually delay the progression of CKD through changes in hemodynamic or microcirculatory parameters.

43
Q

A 23-year-old woman is evaluated for severe AKI complicating endometritis that occurred after premature rupture of membranes and fetal loss at 26 weeks of pregnancy. Seventy-two hours ago, she presented with pelvic pain and drainage, fever, hypotension, tachycardia, and tachypnea. The initial treatment included IV crystalloid and broad-spectrum antibiotics.
Despite normalization of her temperature and BP with this therapy, she has developed oliguria. Her urine output has been <0.1 mL/kg per hour for the past 24 hours and did not improve despite a trial of furosemide 80 mg IV. Her net fluid balance is 5 L positive.
BP is 122/50 mm Hg, heart rate 90 /min, and oxygen saturation 94% on 2 L/min supplemental oxygen. The heart and lungs are normal. She has lower abdominal tenderness. There is scant peripheral edema. Laboratory data include the following:

Result Reference Range
White blood cell count 12,000/µl 4000–11,000
Hemoglobin 9 g/dL 12–16
Platelet count 110,000/µl 150,000–450,000

Potassium 5.7 mEq/L 3.5–5.0
Total CO2 18 mEq/L 23–28
Creatinine 3.2 mg/dL 0.5–1.1
Which of the following options represents the BEST management strategy for this patient?

Select one:
A. Treatment with nesiritide
B. Continuous furosemide infusion
C. Treatment with vasopressin
D. Renal replacement therapy
E. Volume expansion guided by pulmonary artery catheter

A

The correct answer is: D
Renal replacement therapy

This patient with hyperkalemia due to severe, oliguric AKI secondary to sepsis should be treated with renal replacement therapy.
AKI is common, affecting approximately 20% of hospitalized patients. Severe AKI requiring renal replacement therapy occurs in 1–2% of these patients. Of the group requiring renal replacement, mortality is approximately 40%. Among patients who survive, approximately 60% recover kidney function, but many of these develop CKD. Recovery after 90 days is very uncommon.
The furosemide “stress test,” which consists of monitoring urine output after an IV bolus of furosemide (1 mg/kg for patients who are furosemide-naive), has been advocated as a strategy to help predict which patients with AKI are likely to require renal replacement. Normal response to furosemide can only occur if there is preserved renal blood flow, proximal tubule secretion, response at the thick ascending limb of the loop of Henle, and luminal patency; thus, an increase in urine output after furosemide may be a good prognostic sign. Furthermore, management of nonoliguric AKI tends to be less challenging because edema and some metabolic derangements may be addressed with diuretics. This patient did not show improvement in urine output with furosemide and has hyperkalemia. Thus, renal replacement therapy is appropriate.
Sepsis is the most frequent cause of severe AKI. Early fluid resuscitation has become a cornerstone of sepsis management. The use of “early goal-directed therapy” with a measure of adequate volume expansion such as central venous pressure showed promise in some, but not all, trials, and there is no evidence that pulmonary artery catheterization results in improved outcomes. In this patient who now has normal BP, there would be little to gain from insertion of a pulmonary artery catheter.
Norepinephrine has emerged as the pressor of choice to improve BP in patients with sepsis, but there is some evidence that vasopressin improves BP and promotes diuresis compared with norepinephrine. However, there are no data that show vasopressin treatment in the absence of hypotension results in improvement in clinical outcomes.
Nesiritide, a synthetic B-type natriuretic peptide, has been approved to treat acute decompensated heart failure. It has also been used to prevent and treat AKI in a variety of settings, including heart surgery, heart failure, and sepsis. So far, despite some promise as a prophylactic agent in patients undergoing heart surgery, nesiritide has not been shown to improve outcomes in AKI and is not recommended.
The delivery of furosemide by continuous infusion is a popular strategy to maintain a controlled diuresis but has not been shown to be more effective than intermittent dosing. Because she did not respond to an adequate bolus of furosemide, a response to an infusion is unlikely. Furthermore, there is no evidence that furosemide prevents or treats AKI, and some trials have found worse renal outcomes among patients treated with diuretics.

44
Q

A 67-year-old man with ESRD and a history of hypertension, stroke, and diabetes mellitus is undergoing his scheduled dialysis via a tunneled catheter. Due to difficulty in removing the catheter cap, the dialysis nurse used a hemostat to loosen it. The catheter was then aspirated and flushed, and dialysis was initiated uneventfully. The patient was receiving dialysis comfortably in the recliner chair when he developed sudden chest pain and shortness of breath; he called for help and then became unconscious. The dialysis nurse who responded checked and did not see foam in the venous blood line.
In addition to stopping the blood pump and providing 100% oxygen and cardiovascular support, which of the following is the next immediate BEST step in management?
Select one:

A. Place the patient on his left side and in Trendelenburg position
B. Deliver inhalational nitrous oxide therapy
C. Aspirate air directly from the right ventricle
D. Place the patient on his right side in the recumbent position
E. Aspirate air from the distal catheter port with a 30-mL syringe

A

The correct answer is: A
Place the patient on his left side and in Trendelenburg position

This patient with suspected air embolism should be placed on his left side in Trendelenburg position to attempt to trap the air in the apex of the right ventricle and prevent airlock of the right ventricular outflow or the migration of air up the venous circulation to the brain.
Air embolism during dialysis is a serious but rare complication resulting in chest pain, dyspnea, and possible death. Although small amounts of air can diffuse across the arteriolar wall into the alveolar spaces, larger amounts (≥50 mL) can cause pulmonary outflow tract obstruction, also termed “airlock.” Rate of air entry is also important; it is estimated that 300–500 mL of gas introduced at a rate of 100 mL/s can be acutely fatal. This rate can be attained through a 14-gauge catheter with a pressure gradient of only 5 cm H2O.
Air embolism was more common before the advent of safety devices on modern HD machines. These devices include ultrasonographic air detectors that detect bubbles in the circuit, a floating valve in a bubble trap within the venous circuit, and safety alarms, which automatically occlude the circuit and stop the blood pump if activated. The potential sources of air embolism to dialysis patients include preexisting air within the dialyzer and tubing that must be purged during the priming process, as well as air introduced through the administration of saline, heparin, or other medicines during the procedure. Because the blood pump exerts a negative pressure on the arterial limb of the circuit, any leak in the system before the blood pump can result in the entrainment of air. On the post-blood pump limb of the circuit, air could be introduced by inadvertent injection or by disconnection at the venous needle or catheter port allowing direct entry of air into the patient. This latter event would not be amenable to identification by an air detector. Indeed, air embolism related to a dialysis catheter (during insertion, removal, or use) now appears to be the most commonly reported mechanism. In this case, use of a hemostat to turn a stubborn cap likely resulted in a crack in the hub or extension tubing. Note that air entrained at the catheter hub or catheter extension tubing will not result in foam in the venous blood line, whereas air entrained in the dialysis circuit upstream of the catheter might result in this observable sign.
An air embolism while the patient is in recumbent posture usually manifests as chest discomfort and dyspnea with cough due to the entry of air into the heart. Large amounts of air may subsequently enter the cerebral circulation because of incomplete trapping by the lungs or via paradoxical embolism. With a patient in upright posture, air is more likely to enter the intracranial veins, resulting in altered consciousness and seizures.
The differential diagnosis of sudden onset of chest pain and loss of consciousness during dialysis includes acute cardiac ischemia, arrhythmia, dissection, stroke, and dialyzer reaction, among other possibilities. Foam in the venous line suggests the diagnosis of air embolism. A “millwheel” continuous cardiac murmur may be present. Transesophageal echocardiography and Doppler ultrasonography can be diagnostic but are rarely immediately available to the clinician responding to the emergency; thus, the diagnosis must be suspected on clinical grounds.
Treating air embolism includes clamping the venous line and stopping the blood pump immediately. In case of a crack in the hub or in the catheter, it is imperative to clamp proximally to the site of the air leak. The patient should be positioned supine on the left side with the chest and head tilted downward. Cardiorespiratory support and the administration of 100% oxygen by either mask or endotracheal tube are appropriate. Sitting up can lead to propagation of air into cerebral veins and is not appropriate. There is no role for nitric oxide inhalation for air embolism. It is not likely possible to aspirate air from the catheter port, especially if there is suspicion of a cracked hub. Although percutaneous aspiration of air from the ventricle may be required, it is not the immediate first step. Hyperbaric oxygen may be useful in severe cases but would not be immediately available.

45
Q

A 34-year-old woman presents to the emergency department complaining of fatigue and what she describes as “not thinking clearly” for the past month. Her medical history is remarkable for depression and smoking. She is taking venlafaxine and an oral contraceptive pill.
On examination, her BP is 100/60 mm Hg, heart rate 60/min, respiratory rate 16/min, oxygen saturation 99% on room air, weight 51.8 kg, and body mass index 17 kg/m2. She is alert and oriented, but slightly slow and deliberate in response to questions. Her neck veins are not visible at 45°, breath sounds are clear bilaterally, heart sounds are normal, and there is no edema. There are no focal neurological deficits but formal cognitive testing was not performed.
Laboratory values are:

Result Reference Range

Sodium 124 mEq/L 136–145
Potassium 4.2 mEq/L 3.5–5.0
Chloride 90 mEq/L 98–106
Total CO2 25 mEq/L 23–28
Blood urea nitrogen 6 mg/dL 8–20
Creatinine 0.7 mg/dL 0.5–1.1
Glucose 76 mg/dL 70–99 (fasting)
Serum osmolality 254 mOsm/kg 275–295

Random urine osmolality 65 mOsm/kg 38–1400
24-Hour urine collection
Volume 4 L Varies with intake
Sodium 10 mEq/L Varies with intake
Potassium 17 mEq/L Varies with intake
Urea 2.4 g/24 hours 12–20

Which of the following treatment options is the BEST management for this patient’s hyponatremia?

Select one:
A. Initiate chlorthalidone
B. Discontinue venlafaxine
C. Initiate tolvaptan
D. Initiate protein supplements
E. Restrict water intake to <1 L per day

A

The correct answer is: D
Initiate protein supplements

The best treatment for this patient with hyponatremia related to low solute intake and high fluid intake is to increase her solute intake with protein supplements. Urea or salt would also address the hyponatremia, but would lack the nutritional value of protein.
This is a case of hyponatremia secondary to low osmolar intake coupled with a relatively high water intake. Evidence for this diagnosis includes dilute urine, indicating suppression of antidiuretic hormone (ADH), along with very low serum and urine urea levels. This syndrome occurs when solute intake is so low that water excretion cannot meet intake, even with maximal dilution of the urine. In this syndrome, carbohydrate intake is usually sufficient to suppress catabolism of endogenous protein so that there is insufficient endogenous urea. When this syndrome occurs in individuals who consume large quantities of beer but almost no other food, it referred to as beer drinker’s potomania; when related to dietary restriction, it has been referred to as “tea and toast” hyponatremia. This young woman with a low body mass index, high fluid intake, and low solute intake may have an eating disorder. Her sensation of cognitive impairment could well relate to the hyponatremia, but other causes should also be considered.
Assuming that she is in solute balance, her daily solute intake can be estimated by her urinary excretion (note that 1 mEq/L of a univalent ion is the same as 1 mmol/L):
Daily solute excretion ≈ urine volume × (2 × urine [Na+] mmol/L + 2 × urine [K+]mmol/L) + urine urea mg/day ÷ 60 mg/mmol)
Daily solute excretion = 4 L/day (2 × 10 mmol/L + 2 × 17 mmol/L) + 2400 mg/day ÷ 60 mmol/mg = 256 mmol/day
Alternatively, daily solute excretion ≈ urine volume × urine osmolality = 4 L/day × 65 mOsm/kg = 260 mmol/day (note that 1 L ≈1 kg of water and 1 mOsm ≈ 1 mmol).
In this case, the daily solute excretion is approximately 260 mmol. Assuming that the most dilute urine that she can achieve is the measured 65 mmol/L, her daily water excretion is limited to 4 L. Intake of fluid above this limit is retained and results in hyponatremia. Because ADH is suppressed and kidney function is intact, provision of solute (as electrolytes or protein) will result in excretion of the retained water.
Severe fluid restriction, as with <1 L a day, can result in rapid correction, and osmotic demyelination has been reported when solute provision was combined with fluid restriction. Specific recommendations regarding the safe amount of solute provision and fluid restriction are lacking. Patients with severe or symptomatic hyponatremia should be treated under conditions where the change in plasma sodium can be monitored and intervention can be made if indicated. Some experts advocate the use of desmopressin in combination with the provision of solute so that the rate of correction can be safely controlled.
Tolvaptan is a competitive inhibitor of the V2 receptor for ADH. It is indicated for the treatment of hyponatremia mediated by ADH. In this case, ADH is suppressed, so tolvaptan would not be effective.
Chlorthalidone, by inhibiting the transport of sodium in the distal convoluted tubule, can inhibit maximal urinary dilution and could worsen hyponatremia in this setting.
Venlafaxine is potential cause of the syndrome of inappropriate anti-diuresis, but this would result in inappropriately concentrated urine, rather than the dilute urine observed in this case.

46
Q

A 62-year-old man with CKD stage G4/A3 related to diabetes and hypertension developed AKI in the setting of community-acquired pneumonia and sepsis. He became oliguric, and HD was initiated via a previously established arteriovenous fistula.
He is now receiving dialysis every other day for 4 hours. His medications include intravenous (IV) piperacillin-tazobactam, insulin, aspirin, simvastatin, and subcutaneous heparin for deep vein thrombosis prophylaxis.
His third dialysis treatment is interrupted by thrombosis of the extracorporeal circuit. For his fourth treatment, a heparin bolus, 50 U/kg, is given at the beginning of the treatment, but extracorporeal system thrombosis recurs.
Investigation reveals that the platelet count has fallen from a baseline of 180,000/µl to 85,000/µl. Heparin-induced thrombocytopenia (HIT) is suspected and confirmed with an enzyme-linked immunosorbent assay (ELISA) demonstrating the presence of heparin-dependent antibodies to platelet factor 4 as well as a positive result from a serotonin release assay. Heparin is discontinued.
Which of the following options is the BEST strategy for managing heparin-induced thrombocytopenia in this patient?

Select one:
A. Enoxaparin
B. Apixaban
C. Regional citrate anticoagulation
D. Argatroban
E. Warfarin

A

The correct answer is: D
Argatroban

This patient with AKI and HIT should be treated with argatroban. Patients with HIT are hypercoaguable and require systemic anticoagulation with a non-heparin alternative to prevent or treat thrombosis. Patients with HIT but without a thrombotic complication should receive anticoagulation for 4 weeks. Patients with HIT who experience a thrombotic event should receive anticoagulation for 12 weeks.
HIT is a life-threatening complication and occurs in up to 4% of patients who are exposed to heparin. The incidence in patients requiring dialysis appears to be similar to this background rate in the first 3 months of treatment, whereas the incidence among patients receiving chronic HD is much lower, at <1%. HIT results from an autoantibody directed against platelet factor 4 (PF4) which, when complexed with heparin, results in platelet activation and thrombophilia. This can result in arterial and venous thrombosis. HIT should be strongly suspected in the setting of acute-onset thrombocytopenia, a decrease in platelet count of ≥50% from a prior value (even without absolute thrombocytopenia), venous or arterial thrombosis, necrotic skin lesions at heparin injection sites, or acute systemic reactions (e.g., fever, chills, tachycardia, hypertension, dyspnea, cardiopulmonary arrest) occurring after IV heparin administration. Recurrent thrombosis of the extracorporeal circuit and/or the dialysis access can be an important clue in patients receiving HD or continuous renal replacement. The diagnosis can be suspected on the basis of the 4T Score, in which points are assigned for each of the following: the degree of thrombocytopenia, the timing of thrombocytopenia, the occurrence of thrombosis, and the absence of alternative causes. The diagnosis is then established by demonstration of heparin-dependent antibodies to PF4 and a serotonin release assay, which confirms not only that the antibodies are present but that they cause activation of the platelets.
Unfractionated heparin is the usual cause, though low molecular weight heparin can also cause HIT and cannot be used to treat HIT. The onset of HIT is typically 5–14 days after heparin exposure, but the onset can be immediate in patients with previous heparin exposure within the past 100 days. Although there is a correlation between the development of HIT and heparin dose, even minute heparin exposures (such as a dilute catheter locking solution) can precipitate or perpetuate the syndrome.
Options for treatment include argatroban, a direct-acting thrombin antagonist. This agent is metabolized by the liver and is a preferred alternative for patients with AKI or advanced CKD. Lepirudin, which was not offered as a choice in this case, is renally cleared and has a prolonged half-life in AKI, making it less attractive for patients with severe renal impairment, though it has been used successfully. Danaparoid and fondaparinux are factor Xa inhibitors that are additional non-heparin options.
Early warfarin administration has been associated with severe thrombosis and therefore warfarin should not be given until the platelet count normalizes, at which time it can be introduced, overlapping the parenteral anticoagulant.
Apixaban, an orally active factor Xa inhibitor, has an emerging role in HIT treatment and there is increasing experience with this agent in patients with advanced CKD and ESRD. Its main advantage is that it is orally active. However, argatroban is titratable and is the preferred agent in this patient with IV access.
Regional citrate anticoagulation might be effective in preventing thrombosis of the dialysis circuit during treatment but would not address the patient’s hypercoaguable state.
Although HIT does not always recur on repeat exposure to heparin after resolution, a repeat “challenge” is not recommended. Long-term options for patients who require dialysis include heparin-free HD, regional citrate anticoagulation, warfarin (after an appropriate duration of a parenteral non-heparin anticoagulant), apixaban, or conversion to PD.

47
Q

An 18-year-old woman presents to the emergency department with a 2-day history of progressive nausea and shortness of breath. She is otherwise healthy and is not taking any medications. On presentation, her BP is 120/80 mm Hg, heart rate 85/min, respiratory rate 22/min, oxygen saturation 95% on room air, and temperature 36.7oC. Her respiratory effort is increased, but otherwise the physical examination is normal.

Laboratory data include:
Result Reference Range
Serum
Sodium 137 mEq/L 136–145
Potassium 5.2 mEq/L 3.5–5.0
Chloride 113 mEq/L 98–106
Total CO2 12 mEq/L 23–28
BUN 6 mg/dL 8–20
Creatinine 0.7 mg/dL 0.5–1.1
Glucose 810 mg/dL 70–99 (fasting)
Albumin 4 g/dL 3.5–5.5
Serum osmolality 335 mOsm/kg 275–295

Arterial blood gas
pH 7.25 7.38–7.44
PaCO2 28 mm Hg 35–45

Urine
Sodium 31 mEq/L Varies with intake
Potassium 29 mEq/L Varies with intake
Chloride 33 mEq/L Varies with intake

Urinalysis: pH 5.0, +3 glucose, +3 ketones.
What is the MOST likely cause of this patient’s acid/base disturbance?

Select one:
A. Type A lactic acidosis
B. Diabetic ketoacidosis
C. Distal renal tubular acidosis
D. Type 4 renal tubular acidosis
E. Proximal renal tubular acidosis

A

The correct answer is: B
Diabetic ketoacidosis

This is a case of diabetic ketoacidosis (DKA) presenting with a normal anion gap. Early in the course of DKA, ketoacids are excreted with sodium or potassium in the urine and the serum anion gap can be normal or only minimally elevated. Retention of the ketoacids and expansion of the serum anion gap does not occur until volume contraction (due to urinary sodium loss related to osmotic diuresis and ketonuria) is severe enough to impair glomerular filtration.
In select settings, when the patient presents very early in the course or extracellular fluid volume has been maintained by adequate intake, the kidneys can maintain their ability to excrete ketoacids in the urine and the anion gap can be normal. In this situation, the patient would present with classical signs and symptoms of DKA, including high glucose and metabolic acidosis, but without an elevated anion gap. In this case, the patient is euvolemic and has preserved kidney function.
The urine anion gap, or measured urinary net charge gap, can be used to identify excretion of an unmeasured anion. Unmeasured anions can be bicarbonate or non-bicarbonate anions such as lactate or ketoacid anions.
Urine anion gap = (urine [Na+] + urine [K+]) – urine [Cl-]
The normal kidney response to metabolic acidosis is an increase ammonium excretion. Ammonium, an unmeasured cation, is primarily excreted with chloride, so the urine anion gap will be negative (the concentration of chloride exceeds the sum of the concentrations of sodium and potassium) and typically ranges from -20 to -50 mEq/L. The absence of a negative urine anion gap implies that a defect in urinary acidification may be the cause of the acidosis, such as in the case of classic distal (type 1) renal tubular acidosis (RTA). Clinical application of the urine anion gap assumes that there are no unmeasured urinary anions, such as bicarbonate (in proximal RTA) or ketoacids (in DKA). An alkaline urine pH can be a clue that bicarbonaturia is present. In this case, the urine anion gap is positive ((31 + 29) - 33 = 27) and is accounted for by the presence of another unmeasured anion, which in this case are the ketoacid anions, also identified by the urinary dipstick.
Type A lactic acidosis results from tissue hypoperfusion and should produce an elevated serum anion gap. In addition, there are no signs of volume depletion or other apparent mechanism for the development of type A lactic acidosis.
Type 4 RTA, hyporeninemic hypoaldosteronism, does not explain the very high serum glucose level or the ketonuria. Furthermore, this disorder is characterized by only modest or absent metabolic acidosis.
Glycosuria can accompany a proximal RTA, and urine pH may be high during intake of bicarbonate and then low once steady state is established. Likewise, the measured urinary net charge will be positive when bicarbonaturia is present, but will be negative when the proximal RTA achieves steady state. In this patient, glycosuria relates to hyperglycemia rather than proximal tubular dysfunction.
Classic distal (type 1) RTA results from failure of distal acidification of the urine and might be suspected on the basis of the non-anion gap metabolic acidosis and the positive measured urinary net charge. However, most patients with this disorder are unable to achieve a urine pH <5.5, and hyperkalemia is also uncommon. Therefore, distal RTA is not the best explanation for this patient’s presentation.

48
Q

A 47-year-old man presents with a headache and epistaxis. He had been feeling well until a progressively severe headache began about 1 week ago. Brisk epistaxis began today, prompting him to come to the emergency department. His medical history is notable for deep venous thrombosis that complicated right knee surgery for ruptured ligaments sustained while skiing. He received 3 months of warfarin for this problem. He is not currently taking any medications.
Examination reveals BP 180/110 mm Hg. Other findings include an S4 gallop and moderate lower extremity edema. Neurologic examination is normal.
Laboratory data are as follows:

Result Reference Range
Hemoglobin 11.6 g/dL 14–18
Platelet count 200,000/μL 150,000–450,000
BUN 85 mg/dL 8–20
Creatinine 6 mg/dL 0.7–1.3
Urinalysis reveals 3+ protein and 3+ blood. RBC and WBC casts are identified in the urine sediment.
BP is controlled, but epistaxis continues despite placement of a nasal pack by the emergency department physician. A kidney biopsy is planned, and empirical immunosuppression is begun with methylprednisolone IV.
What is the BEST initial strategy to improve this patient’s bleeding diathesis?

Select one:
A. Estradiol
B. Platelet transfusion
C. Red blood cell transfusion
D. Desmopressin
E. Hemodialysis

A

The correct answer is: D
Desmopressin

This patient with epistaxis and an upcoming kidney biopsy should receive IV desmopressin to address uremic platelet dysfunction.
Uremia may be complicated by cutaneous and/or mucosal bleeding. There may also be enhanced sensitivity to aspirin and a higher risk of serious bleeding during invasive procedures. The pathogenesis of uremic bleeding is multifactorial and relates in part to impairment of platelet aggregation and adhesiveness due to intrinsic dysfunction of the glycoprotein IIb/IIIa receptor. Anemia and high levels of nitric oxide are also contributing factors. Although not all uremic patients require specific treatment, those who have active bleeding or require an invasive procedure warrant intervention to improve platelet function.
Desmopressin is an analogue of vasopressin. This agent is believed to increase the release of von Willebrand factor from the vascular endothelium. It may also increase platelet membrane glycoprotein expression. Administration of desmopressin can improve the bleeding time within 1 hour, an effect that may last for 4–8 hours. Although a second dose may be given, efficacy decreases, perhaps because of depletion of the stores of factor VIII/von Willebrand factor.
Both HD and PD can improve the platelet dysfunction observed in severe uremia, but multiple treatments are required to obtain maximal improvement.
Correction of anemia to a hemoglobin >10 g/dL, whether by RBC transfusion or erythropoiesis-stimulating agents, has been shown to decrease bleeding time. However, this patient’s hemoglobin is already >10 mg/dL
Conjugated estrogens have also been used to treat bleeding associated with uremic platelet dysfunction, although the mechanism of benefit is unclear. This strategy has been associated with thrombotic events, so it would not be preferred in this patient with a history of deep venous thrombosis. In addition, although the effect of conjugated estrogens begins within the first day, the peak effect is typically seen at days 5–7.
Infusion of cryoprecipitate is another option to address uremic platelet dysfunction.

49
Q

A 37-year-old woman was admitted with acute gallstone pancreatitis, tachycardia (pulse 160/min), and hypotension (mean arterial pressure 50 mm Hg) and subsequently underwent fluid resuscitation with isotonic saline at a rate of 10 mL/kg per hour. Norepinephrine was added for additional hemodynamic support. Despite these interventions, anuria ensued within the first 24 hours of her hospital stay.
The patient’s hemoglobin was initially elevated at 18 g/dL (reference range, 12–16) consistent with hemoconcentration, and the platelet count was mildly low at 120,000/µl (reference range, 150,000–450,000). Kidney ultrasound demonstrated normal anatomy and no evidence of arterial or venous compromise. HD was initiated for management of azotemia and volume overload. Her course was complicated by deep venous thrombosis (DVT). The hematologic abnormalities corrected with supportive care. Over 2 weeks, her pancreatitis resolved, and she made a complete recovery except for anuric AKI. She remains anticoagulated for the DVT.
CT with IV contrast demonstrated enhancement of only of the subcapsular rim of both kidneys. There was no evidence of hydronephrosis, edema, or local complications of the pancreatitis.
Which of the following diagnoses BEST explains this patient’s course and radiographic findings?

Select one:
A. Acute tubular necrosis
B. Rapidly progressive glomerulonephritis
C. Severe nephrocalcinosis
D. Renal cortical necrosis
E. Abdominal compartment syndrome

A

The correct answer is: D
Renal cortical necrosis

The CT findings of a “rim sign” and the persistent anuric AKI precipitated by severe acute pancreatitis are diagnostic of renal cortical necrosis.
This patient experienced bilateral renal cortical necrosis, a rare form of severe AKI usually associated with severe shock and characterized by oligo-anuria and limited or absent recovery. Pregnancy-related events such as septic abortion, placental abruption, and post-partum thrombotic microangiopathy account for the largest number of cases. Hemolytic uremic syndrome, severe forms of glomerulonephritis, pancreatitis, and other forms of shock are among the various causes as well. The abrupt onset of severe oligo-anuria (<100 mL per day of urine) limits the differential diagnosis to bilateral renal artery occlusion, complete urinary tract obstruction, severe rapidly progressive glomerulonephritis (RPGN), and bilateral acute cortical necrosis. In this case, the pancreatitis-related shock has precipitated renal cortical necrosis.
Although serologic data were not reported and a kidney biopsy was not performed, there are no other clinical features to suggest RPGN. Abdominal compartment syndrome is an important consideration in oliguric AKI complicating pancreatitis but likewise would not explain the radiologic rim sign nor the persistent AKI after resolution of the acute illness. Nephrocalcinosis can follow acute pancreatitis but would not cause anuria.
Two patterns of renal cortical necrosis have been described:
1. Diffuse cortical necrosis, which involves global cortical destruction except for a thin rim of preserved subcapsular and juxtamedullary tissue
2. Patchy cortical necrosis of up to half of the cortical tissue
The pathogenesis of renal cortical necrosis is not well understood, but it is believed to result from profound renal ischemia from severely reduced renal perfusion or intravascular coagulation with microvascular injury. The relatively high incidence in obstetric emergencies suggests the possibility of some relatively unique vulnerability of the renal circulation in this setting. The incidence of renal cortical necrosis has declined significantly in the last 3 decades. This has been attributed to improved access to medical care, particularly in developing countries. The renal prognosis of bilateral diffuse cortical necrosis is uniformly poor, and advanced CKD or ESRD is the expected outcome. It differs from acute tubular necrosis in which the pathology is largely limited to the tubules and demonstrates a significant capacity for recovery.

50
Q

A 66-year-old man with ESRD due to analgesic nephropathy has received HD via a left forearm arteriovenous fistula for 14 years. Over the last year, he has developed pain and stiffness in both shoulders. The pain is located anterolaterally and is aggravated by abduction of the shoulders. He also has neck pain and tingling in both hands, especially at night.
Examination of the shoulders reveals mild tenderness to palpation and protuberance of soft tissue above the shoulders, more prominent on the left. There is no redness or warmth. Radiography of the shoulders shows bone cysts with sclerotic margins that have grown in size compared with images obtained 6 months ago.

Result Reference Range
PTH 680 ng/mL 130–585 (dialysis patients only)
Calcium 10.1 mg/dL 8.6–10.2
Phosphorus 5.6 mg/dL 3–4.5
Electromyography and nerve conduction studies show results consistent with bilateral carpal tunnel syndrome.
Which of the following diagnostic tests is MOST likely to identify the cause of this patient’s shoulder pain?

Select one:
A. Synovial tissue biopsy
B. Magnetic resonance imaging
C. Sestamibi parathyroid scan
D. Technetium bone scan
E. Aspiration of joint fluid

A

The correct answer is: A
Synovial tissue biopsy

This patient should undergo synovial biopsy to establish the diagnosis of β-2 microglobulin amyloidosis. This diagnosis is likely for this patient based on the long duration of dialysis dependence, the distribution of symptoms (involving the neck, both shoulders, and hands), soft tissue prominence over the shoulders (“shoulder pad sign”), and bone cysts on radiography.
β-2 microglobulin is normally filtered at the glomerulus with reabsorption and catabolism in tubules. Reduced clearance in patients with ESRD results in deposition of β-2 microglobulin fibrils in bones and periarticular and other soft tissues, which causes dialysis-related amyloidosis. Osteoarticular deposition causes an erosive and destructive osteoarthropathy, spondyloarthropathy, and carpal tunnel syndrome.
Conventional radiographs of the joint affected by dialysis-associated amyloidosis can show radiolucencies of variable size within the medullary or cortical bone due to cortical destruction. These bone cysts are identified most commonly around hips, shoulders, and carpal bones. Fine sclerotic margins are usually present. Definitive diagnosis of dialysis-related amyloidosis is made by staining biopsy tissue from the affected joint or bone cyst with Congo red and the demonstration of telltale apple-green birefringence under polarized light.
MRI with gadolinium is an excellent imaging modality for intraosseous, periarticular, and soft tissue abnormalities, and unenhanced MRI could confirm bone cysts and identify structural damage such as rotator cuff tear. MRI would not specifically identify the cause of the imaging abnormalities as dialysis-associated amyloidosis.
Technetium bone scanning is useful to identify metastatic disease, but this is not a likely explanation for the patient’s symptoms. Although hyperparathyroidism can result in bone cysts and potentially explain the shoulder and neck pain, it would be less likely to result in bilateral carpal tunnel syndrome. Therefore, a sestamibi parathyroid scan, which might show abnormal findings, would not necessarily explain the symptoms.
Pseudogout is common in patients receiving dialysis because of deposition of calcium pyrophosphate crystals, and presents with an inflammatory arthritis. Aspiration of the joint can establish this diagnosis and would be useful to differentiate gout, pseudogout, or infectious arthritis, but these diagnoses would not explain the constellation of signs and symptoms seen in this patient.
Once the diagnosis of β-2 microglobulin amyloidosis is established, efforts can be directed to enhance clearance. Considerations might include high-flux dialysis using more biocompatible dialyzer membranes, hemofiltration, immunoadsorption, and kidney transplantation.

51
Q

A 45-year-old woman presents to the emergency department with headache and gait instability progressing over the past 2 days.
Five weeks prior to this presentation, a diagnosis of granulomatosis with polyangiitis (GPA) was established on the basis of arthralgias, rash, mononeuritis multiplex, serologic investigations, and skin biopsy. She was treated with IV cyclophosphamide and methylprednisolone and then prescribed oral prednisone, vitamin D, and calcium. Because of a history of a sulfa allergy and dapsone was prescribed for pneumocystis prophylaxis. Daily levothyroxine for chronic hypothyroidism was continued.
Three days ago, she received her second infusion of cyclophosphamide. At that time, she reported that her musculoskeletal symptoms had improved considerably along with resolution of her rash.
On examination, the BP is 130/80 mm Hg, pulse 75/min, respiratory rate 16/min, oxygen saturation 98% on ambient air, and temperature 36.8°C. The jugular venous pressure is 3 cm above the sternal angle; the heart, lungs, and abdomen are normal; and there is no edema. She has fading purpuric lesions on her extremities, and no fresh lesions are identified. She is alert and oriented, but her calculations are slow and her gait is slow and cautious. Sensory changes in the left foot and weakness on left foot dorsiflexion are stable.

Result Reference Range
White blood cell count 7,500/µl 4000–11,000
Hemoglobin 12 g/dL 12–16
Platelet count 153,000/µl 150,000–450,000

Sodium 118 mEq/L
(134 mEq/L 3 days ago) 136–145
Creatinine 0.9 mg/dL 0.5–1.1
C-reactive protein 1.2 mg/dL
(improved from 9) ≤0.8
Erythrocyte sedimentation rate 56 mm/h
(improved from 104) ≤20
Thyroid stimulating hormone 4.3 mU/L 0.5–4
Urine
Sodium 45 mEq/L Varies with intake
Osmolarity 501 mOsm/L 38–1400
Urinalysis shows 1+ blood and 1+ protein (unchanged from presentation)
Chest x-ray is normal.

In addition to treatment of symptomatic hyponatremia, which of the following actions SHOULD be undertaken?

Select one:

A. Prescribe liothyronine (T3)
B. Prescribe broad-spectrum antibiotics
C. Change dapsone to atovaquone
D. Change cyclophosphamide to rituximab
E. Change prednisone to methylprednisolone

A

The correct answer is: D
Change cyclophosphamide to rituximab

The most appropriate change in this patient’s regimen is to replace cyclophosphamide with rituximab. Cyclophosphamide is the likely cause of this patient’s syndrome of inappropriate antidiuretic hormone (SIADH) and should be replaced with an alternative agent to treat the underlying granulomatosis with polyangiitis (GPA).
This woman presents with symptomatic hypo-osmolal hyponatremia. The elevated urine osmolality confirms the presence and action of antidiuretic hormone (ADH). The diagnosis of SIADH is further supported by findings of euvolemia by history and examination, and the absence of low urinary sodium concentration, which implies that the renin-angiotensin-aldosterone axis is not activated. Because she does not have a physiologic stimulus for release of ADH, her presentation is consistent with SIADH.
Cyclophosphamide is known to cause SIADH, but the mechanism has not been established with certainty. Enhanced release of ADH has been postulated, as has upregulation of aquaporins in the collecting duct, perhaps secondary to effects of cyclophosphamide on cytokines. Symptomatic hyponatremia related to cyclophosphamide is a rare event but is an important consideration because it can cause severe complications such as seizures. The risk of SIADH appears greatest in those receiving high dose IV cyclophosphamide, but there are case reports of severe hyponatremia in patients treated with low dose IV or even oral preparations. Because patients receive IV fluids with the cyclophosphamide infusion and are advised to have a high oral fluid intake when taking cyclophosphamide to prevent bladder toxicity, the risk for hyponatremia may be inadvertently increased. Hyponatremia related to cyclophosphamide infusion is typically apparent by 12–48 hours after exposure and usually resolves over several days.
Although the erythrocyte sedimentation rate and C-reactive protein level are elevated, both are improved from baseline, which along with her symptomatic improvement suggests that the GPA is responding to therapy. The urinalysis remains abnormal, but it is too early in her course to consider this a treatment failure. Therefore, there is no indication to escalate the dose of corticosteroids. Infection is always a concern in immunocompromised patients who present with neurologic symptoms. In this case, the neurologic symptoms can be explained by hyponatremia. In the absence of other features to imply infection, empiric use of antibiotics is unwarranted.
A variety of medications have been implicated as a cause of cause of SIADH, but to date, dapsone is not among them. Furthermore, the development of SIADH in this case is temporally linked to the cyclophosphamide infusion, and cyclophosphamide is a well-known cause of SIADH. Thus, changing dapsone to atovaquone is unlikely to improve the situation.
Severe hypothyroidism can result in hyponatremia, but such patients usually have profound clinical features of hypothyroidism, not simply a low serum sodium level. This patient lacks other clinical features of hypothyroidism and her thyroid stimulating hormone value is near the normal range. Therefore, hypothyroidism is a less likely explanation for her severe hyponatremia. Many medications and foods can interfere with the bioavailability of levothyroxine and hypothyroidism can occur if this goes unrecognized. In this case, the addition of calcium to her medications may have resulted in reduced absorption of levothyroxine. Appropriate initial treatment is to educate her on properly timing the consumption of her medications, rather than to add liothyronine or increase the dose of thyroxine.
Reference(s)

52
Q

A 35-year-old Tanzanian man who moved to the United States one year ago is evaluated for nausea, fever, and flu-like symptoms approximately 2 weeks after starting therapy for suspected pulmonary tuberculosis. He had initially presented with a protracted cough, mild weight loss, and upper lobe consolidation on chest x-ray. Sputum was obtained for culture and an anti-tuberculous regimen consisting of isoniazid, rifampin, pyrazinamide, and ethambutol was initiated. Three days after starting these medications, he began to experience nausea, fever, and body aches. Examination at that time revealed a very thin man with low-grade fever, abnormal breath sounds in the lung apices and bilateral upper abdomen and flank tenderness. Laboratory investigation was notable only for a mild increase in liver aminotransferase levels. A viral syndrome was suspected, and he was advised to return in one week if symptoms did not improve.
His medical history is notable for prior treatment for tuberculosis more than 5 years ago, but additional details are not available. His only medications are his anti-tuberculous agents.
Examination is unchanged from 10 days ago; low-grade fever and bilateral abdominal and flank tenderness persist.

Laboratory data are as follows (attachment):

Urinalysis: specific gravity 1.014, 1+ protein, 1+ glucose; microscopy: 1–4 RBC/HPF and 1–4 WBC/HPF.
Which of the following is the BEST explanation for the patient’s AKI ?

Select one:

A. Renal tuberculosis
B. Isoniazid-related hepatic and kidney toxicity
C. Rifampin-related acute interstitial nephritis
D. Pyrazinamide-related urate nephropathy
E. Hepatorenal syndrome

A

The correct answer is: C
Rifampin-related acute interstitial nephritis

This patient with AKI most likely has rifampin-related acute interstitial nephritis.
Of the commonly prescribed anti-tuberculosis (anti-TB) agents, rifampin is the most likely to cause AKI. This is estimated to occur in <2% of patients. The pathogenesis of AKI from rifampin is likely from anti-rifampin antibodies, which may explain the observation that the syndrome develops rapidly after a second exposure. Patients tend to present with nausea, vomiting, abdominal pain, fever, and flu-like symptoms. Only a fraction of patients with a clinical diagnosis of AKI related to rifampin have had kidney biopsies. When kidney biopsy has been performed, the histopathologic findings reported include acute interstitial nephritis, acute tubular necrosis, or both. Although the AKI can be severe and dialysis may be needed, recovery is the rule, occurring in >90% of patients when the drug is discontinued. Abnormalities of aminotransferases, anemia, thrombocytopenia, and hemolysis are often observed.
Isoniazid (INH) and pyrazinamide (PZA) are the most frequent causes of hepatotoxicity among common anti-TB agents, and INH is usually implicated in the most severe cases. PZA is also associated with hyperuricemia and gout. Hyperuricemia is a well-recognized component of tumor lysis syndrome, and there is emerging concern that uric acid can contribute to AKI in other scenarios. In this case, the uric acid is not sufficiently elevated to likely have a direct role in causing AKI.
Hepatorenal syndrome is a form of AKI that occurs in the setting of advanced chronic liver disease with complications of cirrhosis including ascites or severe acute liver failure, neither of which is present in this case.
The genitourinary tract, including the kidneys, represent the most frequent site of extrapulmonary TB in many series. AKI is uncommon, but when it does occur, it is usually attributable to inflammation and obstruction of the urothelium. Pyuria and hematuria are often prominent. Although an ultrasound examination or alternative imaging of the kidneys and bladder would be reasonable, renal TB is not the most likely explanation for this patient’s clinical presentation.

53
Q

A 48-year-old woman presents for evaluation of an abnormal ultrasound finding. A few weeks ago, she experienced abdominal pain and went to a local emergency department where an abdominal ultrasound was performed. Her pain subsequently resolved, but the ultrasound of the abdomen revealed an abnormality of both kidneys prompting referral.
One year ago, the diagnosis of primary hyperparathyroidism was established on the basis of an elevated PTH, persistently elevated serum calcium ranging from 10.3 to 10.9 mg/dL (reference range, 8.4–10.2), and 24-hour urinary calcium of 609 mg (reference range, <250). She was found to have a parathyroid adenoma. Additional investigations at that time included normal electrolyte levels, urinalysis findings, and chest x-ray.
Six months ago, she had a parathyroidectomy of the right inferior parathyroid gland resulting in normalization of her serum calcium and PTH levels.
On examination, she appears well, her BP is normal, and physical examination is unremarkable.

The sagittal view of the right kidney is shown below:

Which of the following options BEST explains the findings present on this ultrasound image?

Select one:
A. Polycystic kidney disease
B. Acquire kidney cysts
C. Nephrolithiasis
D. Nephrocalcinosis
E. Hydronephrosis

A

The correct answer is: D Nephrocalcinosis

The ultrasound image demonstrates a bright, hyperechoic medullary interstitium with acoustic shadowing characteristic of nephrocalcinosis. In this case, it is related to previous long-standing primary hyperparathyroidism.
Nephrocalcinosis is defined as the parenchymal deposition of either calcium oxalate or calcium phosphate in contradistinction to luminal deposition, which is referred to as nephrolithiasis. Both conditions can coexist because they share predisposing factors. In this case, several small, punctate examples of nephrolithiases are identified by ultrasound, in addition to the diffuse parenchymal calcification. The distribution of nephrocalcinosis is most often medullary. Cortical nephrocalcinosis is unusual and may develop from severe AKI such as cortical necrosis. Nephrolithiasis appears most often as a bright, echogenic focus with shadowing. Hydronephrosis would appear as echo-lucent dilatation of the collecting system. Both acquired kidney cysts and polycystic kidney disease would demonstrate variable numbers of round, echo-lucent cortical lesions. In this case, there is a fluid-filled echo-lucent structure but it is extra-renal in location.
There are many possible causes of nephrocalcinosis, but in this patient, who had primary hyperparathyroidism and documented hypercalciuria, there is no reason to invoke an alternative cause. The otherwise normal preoperative electrolyte levels and chest x-ray, coupled with the prompt normalization of her serum calcium and PTH postoperatively, clearly identify primary hyperparathyroidism as the underlying etiology.
Any inherited or acquired condition that results in hypercalciuria can provoke nephrocalcinosis. Patients with a classic distal renal tubular acidosis can develop nephrocalcinosis. The chronic systemic metabolic acidosis leads to buffering of acid by bone and release of calcium and phosphate, which can precipitate in the tubules. This is exacerbated by the fact that patients with distal renal tubular acidosis cannot maximally acidify the urine, so calcium phosphate precipitation is favored. Chronic sarcoidosis can lead to hypercalcemia and hypercalciuria with nephrocalcinosis.
Nephrocalcinosis may be clinically silent and discovered incidentally when abdominal imaging is performed for some other purpose. Although it may improve with time, it would not be expected to resolve within 1 month and may in fact persist indefinitely. Advanced nephrocalcinosis is visible on plain radiography, but CT and ultrasound are more sensitive imaging techniques. Nephrocalcinosis is typically asymptomatic unless it is associated with other conditions such as nephrolithiasis

54
Q

A 53-year-old man developed ESRD from diabetic nephropathy and started in-center HD 4 years ago. He no longer makes urine. He is 180 cm tall and his target weight is 100 kg. He gains 3.5–5.5 L between treatments. He has a radial arteriovenous fistula that is working well. His current dialysis prescription is blood flow 400 mL/min, dialysate flow 600 mL/min, and a dialyzer with a mass transfer-area coefficient (KoA) of urea 1170 mL/min. During his last session, he dialyzed for 215 min, and his Kt/V was 1.14.

Which of the following would lead to the LARGEST increase in his single-pool Kt/V (spKt/V)?

Select one:
A. Increasing his dialyzer KoA from 1170 to 1480 mL/min
B. Increasing his dialysate flow rate from 600 to 800 mL/min
C. Decreasing his ultrafiltration volume from 4 to 3 L per session
D. Increasing his treatment time from 215 to 240 min
E. Increasing his blood flow rate from 400 to 500 mL/min

A

The correct answer is: D
Increasing his treatment time from 215 to 240 min

Increasing the dialysate flow, the blood flow, or the KoA of the dialyzer will all increase the clearance (K) during the treatment and therefore increase single-pool Kt/V (spKt/V). Increasing the treatment time will also increase the spKt/V. The relative effect of each change is shown in the table. The estimated spKt/V is calculated using a V of roughly 50 L from the Watson anthropometric equation.
Ultrafiltration (UF) will increase clearance because urea is removed through both diffusion and convection. Decreasing the UF goal will therefore decrease clearance (K) and decrease the spKt/V, but by a relatively modest amount. Increasing his target weight will also decrease his spKt/V, because the V will be larger. Note that all the treatment times needed to achieve an estimated spKt/V of 1.4 are longer than 4 hours, even with a very large KoA dialyzer. A treatment time extending beyond 4 hours would also be appropriate to keep the UF rate <10 mL/kg per hour, which is associated with less intradialytic hypotension and lower all-cause mortality.

55
Q

A 26-year-old woman with no significant medical history is referred for evaluation of new-onset hypertension that was discovered during her annual gynecologic examination. At that visit, her BP was 162/100 mm Hg. She has no family history of hypertension. At her last evaluation 1 year ago, her BP was 120/70 mm Hg.
Review of systems is notable for occasional headaches and anxiety.
She does not take prescription medications or any over-the-counter medications other than a multivitamin. She does not use recreational drugs or consume alcohol.
On examination, she is well appearing. BP is 158/98 mm Hg on the right and 156/100 mm Hg on the left. Her height is 168 cm, weight 73 kg, and body mass index 25.8 kg/m2. The physical examination, including the fundoscopic examination, is normal. No bruits are appreciated. Laboratory data include potassium 3.8 mEq/L (reference range, 3.5–5.0) and creatinine 0.8 mg/dL (reference range, 0.5–1.1). Her urinalysis is negative for blood and protein.
A renal artery duplex scan shows a lesion highly suggestive of unilateral renal artery stenosis. CT angiography confirms the diagnosis of fibromuscular dysplasia.
Renal artery angiogram and angioplasty are planned.

Which of the following options SHOULD also be performed in this patient?

Select one:

A. Measurement of plasma metanephrines
B. Measurement of aldosterone and renin in the adrenal vein at the time of angiography
C. Measurement of peripheral vein renin levels before and after captopril
D. Magnetic resonance angiography of the cervical and cranial arteries
E. Coronary angiography

A

The correct answer is: D
Magnetic resonance angiography of the cervical and cranial arteries

This patient with fibromuscular dysplasia (FMD) affecting the renal circulation should also undergo imaging of the carotid and intracranial circulation.
FMD is a systemic, non-inflammatory vasculopathy affecting small- and medium-sized vessels that can manifest as dissection, aneurysm formation, or stenosis of any vascular bed. Although renal artery involvement is most common, carotid artery involvement occurs in up to 21% of patients and results in a stroke or other cerebrovascular event. Intracranial involvement can also occur. For these reasons, imaging of the cervicocephalic arteries as well as cerebral vasculature has been recommended in patients with FMD because of the possibility of therapeutic intervention.
Plasma or urine metanephrine testing is useful to screen for pheochromocytoma but would not be needed in this case in which an alternative cause for hypertension has been established.
Peripheral vein renin measurement for the diagnosis of renovascular hypertension is less commonly used since the widespread availability of non-invasive imaging for anatomic diagnosis. The reported sensitivity, even with captopril stimulation, remains <100%. Therefore, in this case, the results of peripheral renin testing would be unlikely to change the course of management.
Adrenal vein sampling of aldosterone and renin is used to diagnose and localize an aldosterone-producing adenoma and would not be useful in this situation.

56
Q

A 30-year-old man with no significant medical history presents to the emergency department with a 1-month history of nausea, vomiting, and weakness.
He takes occasional non-steroidal anti-inflammatory drugs for weakness, joint aches, and muscle pain, but takes no other medications and has no family history of kidney disease.
He lived in Nicaragua until approximately 1 month ago, when he came to the United States to join family. Currently he is not working, but he previously worked on a sugar cane farm for many years. He does not smoke or drink alcohol.
On physical examination, he is 168 cm and weighs 68 kg. His BP is 110/70 mm Hg. His examination findings are unremarkable, and there is no edema.

Result Reference Range
Hemoglobin 10.5 g/dL 14–18
BUN 45 mg/dL 8–20
Creatinine 5.5 mg/dL 0.7–1.3
Uric acid 8.5 mg/dL 3–7
Urinalysis: pH 5, specific gravity 1.015, 1+ protein, 1+ glucose
A kidney biopsy is performed. Eight of 15 glomeruli are globally sclerotic. Four glomeruli have evidence of ischemia. Tubular atrophy and interstitial fibrosis are seen in 60% of the parenchyma. There are occasional patchy areas of increased monocyte infiltration.

What is the most LIKELY cause of patient’s kidney disease?

Select one:
A. Primary focal and segmental glomerulosclerosis
B. Acute interstitial nephritis
C. Familial juvenile hyperuricemic nephropathy
D. Balkan endemic nephropathy
E. Mesoamerican nephropathy

A

The correct answer is: E
Mesoamerican nephropathy

This patient has a clinical history and kidney biopsy consistent with Mesoamerican nephropathy (MaN), an endemic cause of CKD in Central America whose pathogenesis is poorly understood. The majority of cases to date have been reported in Nicaragua and El Salvador, but there are also reports from Guatemala and Costa Rica. There has been a substantial increase in the prevalence of CKD and ESRD in these regions. Similar presentations of CKD have been reported in other tropical and subtropical regions around the globe. For this reason, some authors now prefer the term CKD of unknown cause to MaN.
MaN primarily affects young men who are exposed to heat stress from laboring outdoors in extreme temperatures. Repeated episodes of AKI from volume depletion or cyclical hypertonicity may play a role in the pathogenesis of this disease. In addition, it has been postulated that some workers may be reluctant to drink the water because of concern for contamination. An increase in uric acid out of proportion to the severity CKD is often seen and may also contribute to the pathogenesis. Risk factors appear to include low body mass index, male sex, and consumption of sugar-containing rehydration beverages or sugar cane itself. Other potential factors include heavy metal exposure, insecticides, and non-steroidal anti-inflammatory drugs.
Clinically, patients present with non-proteinuric CKD that is most consistent with chronic tubular interstitial nephritis. They are usually asymptomatic until late in the course. Kidney biopsy reveals interstitial fibrosis but also evidence of glomerular ischemia with glomerular basement wrinkling.
Primary focal and segmental glomerulosclerosis (FSGS) is unlikely in this case because there is minimal proteinuria and the kidney biopsy is not consistent with this diagnosis. Primary FSGS is characterized by diffuse effacement of foot processes by electron microscopy as well as focal and segmental sclerosis.
Acute interstitial nephritis typically results in diffuse interstitial inflammation. The chronicity of the observed findings, including tubular atrophy and interstitial fibrosis, would not be expected.
Familial juvenile hyperuricemic nephropathy is characterized by progressive CKD and early-onset gout. This disorder is a type of autosomal dominant tubulointerstitial kidney disease characterized by a mutation in the uromodulin gene. Several mutations have been described and newer classification schema recommend describing this as uromodulin-associated kidney disease (UKD). Uromodulin, also known as Tamm-Horsfall glycoprotein, is produced in the cells of the thick ascending limb of the loop of Henle. This protein may play a role in limiting calcium crystallization and preventing urinary tract infections. The pathology seen in patients with UKD is typically a diffuse tubulointerstitial fibrosis. Glomerular changes are unusual and uric acid crystals are not seen.
Balkan endemic nephropathy is caused by chronic aristolochic acid exposure and also causes chronic interstitial nephritis, although it does not cause the glomerular wrinkling that is suggestive of recurrent ischemia. Because this patient is from Central America, MaN is much more likely.

57
Q

A 60-year-old woman with locally invasive pancreatic cancer is evaluated for AKI. She was diagnosed with pancreatic cancer 9 months ago and underwent proximal pancreaticoduodenectomy with excision of 2 positive lymph nodes. Six months ago, she began a chemotherapy protocol that included cisplatin and gemcitabine every 28 days.
She was doing well until 2 weeks ago when she presented with a headache, and her BP was noted to be 190/110 mm Hg. She began taking amlodipine.
One week ago, she was admitted to the hospital with shortness of breath and edema. Her BP was 200/100 mm Hg, and she had evidence of volume overload. She was treated with diuretics and discharged. Laboratory test results from that admission revealed a creatinine level of 1.3 mg/dL, increased from a baseline of 0.7 mg/dL (reference range, 0.5–1.1).
At the time of nephrology consultation, she complained of fatigue. Her most recent chemotherapy treatment was 30 days ago. Her current medications are amlodipine 5 mg daily and furosemide 20 mg twice daily. Examination was significant for BP 160/90 mm Hg. She had pallor and mild tachycardia, but findings were otherwise unrevealing.

Laboratory data are as follows:
1 month ago Current result Reference range
Hemoglobin 10 g/dL 7 12–16
Platelets 230,000/µL 100,000 150,000–450,000
Creatinine 1.3 mg/dL 1.6 0.5–1.1
Lactate dehydrogenase (LDH) 484 U/L 80–225

Urinalysis shows 100 mg/dL of protein and 5–10 RBC/HPF.
A kidney biopsy yields 12 glomeruli, many with global endothelial cell swelling, and occasional microthrombi are seen within the capillary lumens. Two interlobular arteries show intraluminal obliteration by fibrin clot. The tubular interstitial compartment has mild acute tubular necrosis and interstitial fibrosis.

Image, silver stain x 400: CJASN 13(2): 300–317, 2018 (image attached)

Which is the most appropriate next step to address this patient’s AKI?

Select one:
A. Stop cisplatin
B. Start prednisone
C. Start heparin
D. Stop gemcitabine
E. Increase amlodipine

A

The correct answer is: D
Stop gemcitabine

The most appropriate next step for this patient with evidence of AKI due to a thrombotic microangiopathy (TMA) is to discontinue gemcitabine.
Gemcitabine is a nucleoside analog that has activity against pancreatic, breast, and lung cancer. This agent is one of several anti-neoplastic drugs that can cause a TMA. Mitomycin C and vascular endothelial growth factor inhibitors can also have this adverse effect.
This patient demonstrates the clinical features of gemcitabine-associated TMA (gTMA), which is seen in 0.4–5% of patients receiving this drug. It can occur at any time after exposure to gemcitabine, but most commonly presents after a median exposure time of 7 months or a cumulative dose of 20,000 mg/m2. Clinical symptoms can be insidious and slowly progressive. Patients will most commonly present with hypertension, edema, and AKI. Non-nephrotic proteinuria and hematuria are frequently present as well. Hematologic findings of TMA include anemia, thrombocytopenia, high LDH, and low haptoglobin. Diagnosis can be made by kidney biopsy or by peripheral smear if there are findings consistent with a TMA such as fragmented red blood cells and schistocytes. The mechanism of gTMA is not entirely clear but may be related to direct drug toxicity to endothelial cells. There is no standard treatment for gTMA. Recognition of the clinical entity and discontinuation of the drug is most important. If kidney dysfunction does not improve with discontinuation of the gemcitabine, then options include plasmapheresis, steroids, and eculizumab, which have been used with varying degrees of success.
Cisplatin is nephrotoxic but causes tubular toxicity, rather than TMA; discontinuation of this agent in the setting of AKI is important, but this would not address the TMA. Increasing amlodipine may improve BP slightly but would likewise would not address the underlying pathology. Anticoagulation, with heparin or other agents, does not appear to have a role in the treatment of this disorder.

58
Q

A 70-year-old woman is referred for evaluation of lower extremity edema and proteinuria. She has had progressive leg swelling and a 3-kg weight gain over the last few months. A cardiac evaluation, including transthoracic echocardiogram, was unrevealing. Her medical history includes diabetes mellitus for the past 10 years, which has been well controlled, and hypertension. Her medications are glipizide and lisinopril.
Physical examination is notable for a BP of 146/80 mm Hg and pulse of 78/min. She has normal jugular venous pressure. The heart and lungs are normal. There is 3+ lower extremity edema.
Her laboratory data include:

Result Reference Range
Hemoglobin 11.6 g/dL 12–16
Platelets 374,000/µL 150,000–450,000

Sodium 137 mEq/L 136–145
Potassium 4.2 mEq/L 3.5–5.0
Chloride 108 mEq/L 98–106
Total CO2 23 mEq/L 23–28
BUN 56 mg/dL 8–20
Creatinine 1.78 mg/dL 0.5–1.1
Glucose 85 mg/dL 70–99 (fasting)
Albumin 2.3 g/dL 3.5–5.5

Urine protein-to-creatinine ratio 8.2 <0.2

Urinalysis reveals 1+ protein and moderate blood. Microscopy reveals occasional granular casts.

Kidney biopsy demonstrates that 4 of 16 glomeruli are globally sclerotic, while the remaining have a membranoproliferative pattern. Congo red staining is negative. Immunofluorescence demonstrates 3+ IgG and 3+ lambda in a granular pattern, involving the mesangium and the capillary loops. Electron microscopy demonstrates randomly arranged fibrils of 15–30 nm in diameter in the mesangium with some extension into the capillary basement membranes.
Image x 46,000: CJASN 6(4): 775–784, 2011
Which ONE of the following is the MOST likely explanation for this patient’s presentation?
Select one:

A. Fibrillary glomerulonephritis
B. Collagenofibrotic glomerulopathy
C. Light chain deposition disease
D. Immunotactoid glomerulonephritis
E. Renal amyloidosis

A

The correct answer is: A
Fibrillary glomerulonephritis

This patient who presents with AKI, edema, and proteinuria has features of fibrillary glomerulonephritis on kidney biopsy. Fibrillary GN is a fibrillar glomerular deposition disease characterized by Congo red-negative (non-amyloid) fibers, which deposit in the mesangium and glomerular capillary walls. These fibrils are randomly arranged and typically 15–24 nm in diameter.
The clinical presentation and light microscopic findings in fibrillary GN can vary, but hypertension, proteinuria (often as part of the nephrotic syndrome), hematuria, and AKI are common. Global sclerosis and MPGN may be seen. Immunofluorescence is typically positive for IgG, C3, and both kappa and lambda, but a more restricted pattern can also be present. Although most cases are idiopathic, malignancy, plasma cell dyscrasias, and autoimmune diseases have been associated with some cases. In this case, the heavy proteinuria and the lambda restriction by immunofluorescence suggest that this patient may have a clonal plasma cell disorder. DNAJB9, a heat shock protein, is emerging as a specific biomarker for this disorder that can be demonstrated by immunohistochemical staining of biopsy material or measurement in serum.
In contrast, the fibrillar glomerular deposition disease known as immunotactoid GN is characterized by parallel arrays of microtubules that are usually >30 nm in diameter. Patients with immunotactoid GN also have a variable presentation but tend to experience a better renal survival than those with fibrillary GN.
Renal amyloidosis is characterized by acellular lesions on light microscopy that have positive periodic acid-Schiff staining. Deposits can appear amorphous or nodular. The gold standard for diagnosis is Congo red positivity and apple-green birefringence under polarized light. Characteristic electron microscopy findings include randomly distributed fibrils 10–12 nm in diameter, smaller than those seen in fibrillary GN.
Collagenofibrotic glomerulopathy is a rare glomerulopathy with organized deposits. These deposits are made of type 3 collagen, and the pathognomonic electron microscopic feature is fibers in a transverse band pattern approximately 60 nm in diameter. Light microscopy shows a lobular appearance of glomeruli that can be confused with Kimmelstiel-Wilson nodules typically seen in diabetic nephropathy. Immunofluorescence is generally unremarkable. Pathogenesis of this disease is unknown.
Light chain deposition disease is often associated with a monoclonal gammopathy and displays a nodular pattern of glomerular changes with tubular basement membrane thickening. It does not typically have organized deposits on electron microscopy.

59
Q

A 37-year-old woman with a history of autosomal dominant polycystic kidney disease (ADPKD) presents to the emergency room with a 2-day history of fever, back pain, and nausea. She is not allergic to any antibiotics.
She has a BP of 98/50 mm Hg and a pulse of 110/min. She is diaphoretic and appears to be in pain. Severe left-sided costovertebral angle tenderness is present. Her laboratory data are significant for serum creatinine 3.4 mg/dL (baseline, 3; reference range, 0.5–1.1) and potassium 5.3 mEq/L (reference range 3.5–5.0). Urinalysis reveals 2+ leukocyte esterase, and there are 40–50 WBC/HPF with many bacteria on urine microscopy. She is admitted to the hospital.
Which is the following is the MOST appropriate initial therapy for this patient?
Select one:

A. Ciprofloxacin
B. Tobramycin
C. Cefepime
D. Meropenem
E. Trimethoprim-sulfamethoxazole

A

The correct answer is: A
Ciprofloxacin

The most appropriate initial antibiotic therapy for this patient with a urinary tract infection with systemic signs and symptoms complicating underlying ADPKD is ciprofloxacin.
Choice of antibiotics for a urinary tract infection that may involve an infected cyst is a major challenge. Although the microbiology is the same as that seen with pyelonephritis, it is more difficult to achieve therapeutic antibiotic levels in an infected cyst compared with renal parenchyma with normal architecture. Drug delivery is independent of glomerular filtration when cysts do not communicate with renal tubules. Instead, antibiotics gain access to the cyst lumen primarily by diffusion from the blood. This limits the utility of water-soluble antibiotics, such as aminoglycosides and β-lactam antibiotics that do not penetrate cysts well. Instead, lipid-soluble antibiotics, such as quinolones and trimethoprim-sulfamethoxazole, should be considered first-line treatments, because these drugs appear to penetrate kidney cysts and achieve adequate concentrations in cyst fluid. Both have good oral bioavailability but IV administration is reasonable in a patient with significant nausea.
Although trimethoprim-sulfamethoxazole can penetrate kidney cysts, potassium retention and hyperkalemia can occur. This effect is usually mild but life-threatening cases have been reported. Risk factors appear to include dose, age, kidney function, and other medications or conditions that predispose to hyperkalemia. Therefore, this drug would not be the most appropriate initial therapy in this patient with preexisting hyperkalemia and CKD.
Cefepime is a fourth-generation cephalosporin, which can be used to treat urinary tract infections. Although first- and second-generation cephalosporins have limited cyst penetration, third-generation cephalosporins do appear to be efficacious for the treatment of infected cysts in ADPKD. There are case reports that detail success with cefepime, but it would not be used for initial therapy.
Meropenem is a carbapenem that has broad-spectrum activity against gram-negative organisms, including Pseudomonas aeruginosa, anaerobes, and gram-positive organisms. It is useful for organisms with extended spectrum β-lactamases and would not be recommended as first-line therapy. This agent must be given in a lower dose for reduced kidney function.
Urinary tract infections are common in patients with ADPKD and occur in 30–50% of patients at some point in their lives; however, cyst infections are much less common. The presentation of an infected cyst may be similar to that of pyelonephritis, or patients may have a focal region of tenderness. If pain or fever persists for >72 hours after initiation of appropriate antibiotics, imaging can be obtained, ideally with CT to exclude other pathology such as a perinephric abscess, nephrolithiasis, or obstruction. As CT scanning, even with IV contrast (which may be relatively contraindicated depending on kidney function), may be limited in its ability to identify a culprit cyst among many that may have inflammatory changes, there is increasing use of positron emission tomography scanning in this setting. Aspiration of a cyst that appears to be the focus of infection can be considered for both diagnostic and therapeutic purposes. Female sex and recent instrumentation are risk factors for these types of infections.

60
Q

A 19-year-old man with ESRD of unclear etiology presents for a routine visit 4 months after receiving a living-related kidney transplant from his father.
He reports feeling well. He has a good appetite and exercise tolerance. He does not report any fever or urologic symptoms.
His immunosuppression consists of tacrolimus 2 mg twice daily, mycophenolic acid 720 mg twice daily, prednisone 5 mg daily, and trimethoprim-sulfamethoxazole daily. He had a history of prior blood transfusion, and Panel Reactive Antibody (PRA) score prior to transplant was 8%. He had an antibody against human leukocyte antigen (HLA)-DR 16, but no donor-specific antibodies were present. He received induction with anti-thymocyte globulin.
On physical examination, his temperature is 36.1°C, BP is 120/60 mm Hg, and pulse is 76/min. Abdominal examination reveals a well-healed incision over the right kidney allograft. The extremities are without edema, and the remainder of the exam is normal.
His laboratory data include:

Result Reference Range
White blood cell count 5,820/µL 4000–11,000
Hemoglobin 11.8 g/dL 14–18
Platelets 123,000/µL 150,000–450,000

Sodium 138 mEq/L 135–145
Chloride 106 mEq/L 98–106
Total CO2 21 mEq/L 23–28
Potassium 4.9 mEq/L 3.5–5.0
Creatinine 1.7 mg/dL (nadir 1.3) 0.7–1.3
Tacrolimus trough 5 ng/mL 5–10 (>3 months post-transplant)
Urinalysis is negative for protein; the sediment has occasional WBCs and very rare RBCs.
Repeat anti-HLA testing is negative for donor-specific antibodies. Kidney ultrasound reveals no hydronephrosis. Resistive indices are elevated at 0.8.

Which ONE of the following is the MOST likely diagnosis?

Select one:

A. BK virus nephropathy
B. Obstructive nephropathy
C. Acute tubular necrosis
D. Tacrolimus nephrotoxicity
E. Antibody-mediated rejection

A

The correct answer is: A
BK virus nephropathy

The most likely cause of an asymptomatic rise in the serum creatinine in this patient is BK virus nephropathy.
An asymptomatic elevation of the plasma creatinine level that occurs 3–6 months after transplantation is concerning for BK nephropathy, an infection mediated by a polyomavirus that almost exclusively affects a kidney allograft in the setting of immunosuppression. Prevalence varies according to the immunosuppression protocol used, but rates as high as 10% have been reported. The combination of BK viremia of >10,000 copies/mL and allograft dysfunction is highly suspicious for this diagnosis. The urinalysis in BK virus nephropathy is variable. There may be pyuria, hematuria, and inflammatory cells, or the urinalysis may be normal. Urine cytology may reveal BK-infected cells, which have been called “decoy” cells because of their resemblance to renal carcinoma cells, but this finding is neither sensitive nor specific. Electron microscopy of the urine has also been performed to diagnose BK viruria. Identification of viral aggregates in the urine, termed Haufen, by EM is emerging as a potential noninvasive diagnostic technique.
A kidney biopsy with specific staining for the virus, using an antibody targeting the related SV40 virus, is usually performed to confirm the diagnosis and exclude any concurrent process such as rejection. There are no proven strategies to treat BK virus infection. Recent guidelines based on non-randomized data recommend reducing immunosuppression as the main treatment strategy. This approach remains problematic, because immunosuppression reduction can lead to rejection. In one study that included routine protocol biopsies, one-quarter of all graft losses were due to BK nephropathy. Biopsies performed later in the course of BK nephropathy showed interstitial fibrosis and tubular atrophy. Randomized trials using levofloxacin as prophylaxis failed to show any benefit in preventing BK viremia or nephropathy. To date, there is no established role for antiviral therapy.
Although acute cellular rejection is a potential explanation for the increase in creatinine in this patient, antibody-mediated rejection is not likely in the absence of circulating donor-specific antibodies. Tacrolimus nephrotoxicity is unlikely in the setting of a low-normal trough.
Obstruction is a common cause of kidney dysfunction post-transplant, although the absence of hydronephrosis on ultrasound is reassuring to exclude this diagnosis. Of note, chronic BK virus nephropathy can be associated with ureteral stenosis and obstructive nephropathy.
Ischemic tubular injury and necrosis is a frequent cause of allograft dysfunction, particularly after deceased donor kidney transplantation, but is unlikely to occur at this point after a live-donor transplant in the absence of an acute illness. Renal allograft arterial stenosis could potentially lead to ischemic injury, but the absence of hypertension or bruit makes this diagnosis improbable. The finding of elevated resistive indices is nonspecific and may reflect renal parenchymal disease.

61
Q

A 45-year-old man is evaluated for decreased kidney function. He is co-infected with HIV and hepatitis B virus. He has done well with antiretroviral therapy for 12 years, and viral load for both infections is undetectable. His most recent regimen includes darunavir and a combination pill composed of tenofovir disoproxil fumarate and emtricitabine. He has taken this combination for the past 4 years.
He also has type 2 diabetes mellitus that has been controlled by diet.
On physical examination, his BP is 117/78 mm Hg and his heart rate is 86/min. He appears well and is in no distress. There is no edema.
His laboratory parameters include the following:

Result Reference Range
Serum
Sodium 137 mEq/L 136–145
Potassium 3.4 mEq/L 3.5–5.0
Chloride 107 mEq/L 98–106
Total CO2 20 mEq/L 23–28
BUN 24 mg/dL 8–20
Creatinine 1.6 mg/dL
(1.1 mg/dL 4 months ago) 0.7–1.3
Glucose 128 mg/dL (non-fasting) 70–99 (fasting)
Uric acid 2.2 mg/dL 3–7
Calcium 9.2 mg/dL 8.6–10.2
Phosphorus 2.3 mg/dL 3–4.5
25-hydroxyvitamin D 17 pg/mL 30–60
PTH 108 pg/mL 10–65

Urine protein-to-creatinine ratio 0.9 g/g <0.2
Urine albumin-to-creatinine ratio 368 mg/g <30
Urinalysis: pH 6, specific gravity 1.010, 1+ protein, no blood, no glucose. Microscopy: occasional granular casts.
Which of the following tests can be used to CONFIRM a proximal tubulopathy in this patient?

Select one:
A. Fractional excretion of uric acid
B. Tubular reabsorption of phosphate
C. Urine calcium-to-creatinine ratio
D. Fractional excretion of phosphate
E. Urine potassium-to-sodium ratio

A

The correct answer is: A
Fractional excretion of uric acid

The most appropriate test to confirm a proximal tubulopathy in this patient is the fractional excretion of uric acid (FEUA).
This patient has CKD in the setting of longstanding use of tenofovir disoproxil fumarate (TDF). TDF is a nucleoside reverse transcriptase inhibitor that has activity against HIV and hepatitis B. This agent has been an important addition to the antiretroviral armamentarium because it is well tolerated, it is dosed once daily, and it has been co-formulated with other agents to limit pill burden. TDF is structurally similar to adefovir and cidofovir, both of which are known to cause proximal tubulopathy. Similarly, TDF has also been shown to cause a proximal tubulopathy with Fanconi syndrome or AKI, and TDF has also been associated with CKD. In large trials, the risk appears to be <2%, but the insult can be severe. These agents are excreted by glomerular filtration and proximal tubular secretion via organic anion transporters. Nephrotoxicity has been attributed to mitochondrial dysfunction from disruption of mitochondrial DNA. A tenofovir prodrug, tenofovir alafenamide (TAF), is now available and can achieve efficacy with 90% lower systemic exposure to tenofovir. TAF appears to cause less kidney toxicity than TDF formulations and should be used preferentially for patients with CKD.
This patient shows evidence of tubular dysfunction. He has an increased creatinine, and there are granular casts in the urine. He also has proteinuria, but less than 50% of this appears to be albumin based on a comparison of the protein-creatinine ratio and the albumin-creatinine ratio. The discrepancy may represent tubular proteins such as β-2 microglobulin or retinol-binding protein. These urinary proteins are more commonly measured in the evaluation of pediatric disorders.
Hypophosphatemia and phosphaturia are common with TDF nephrotoxicity, and TDF has also been associated with osteomalacia and osteoporosis, which are likely attributable to chronic phosphaturia. An elevated fractional excretion of phosphate (FEPhos) in the setting of a low serum phosphate confirms renal phosphate losses. Indeed, if proximal tubule function is normal, the FEPhos should be <5% in the setting of hypophosphatemia. In contrast, the complementary value, the tubular reabsorption of phosphate or 1-(FEPhos), is expected to be high in the setting of hypophosphatemia. In this case, these tests cannot be relied on to implicate proximal tubular damage because there is evidence of secondary hyperparathyroidism from vitamin D deficiency, and this also causes renal phosphate wasting.
Most patients with CKD have elevated serum phosphorus and uric acid levels, but these levels are low in this patient. Hypouricemia, in the absence of urinary losses, may be related to medications that lower uric acid levels such as allopurinol, febuxostat, and rasburicase. Hypouricemia that is associated with renal losses occurs in the setting of proximal tubulopathy or with volume expansion (as with IV fluids or the syndrome of inappropriate antidiuretic hormone). The FEUA should be <10% in hypouricemic individuals with normal proximal tubular function.
Use of the urine potassium-sodium ratio has been advocated as a strategy to determine the potential response to mineralocorticoid receptor antagonists in patients with resistant edema from cirrhosis or congestive heart failure. This test would not be useful to evaluate the function of the proximal tubule.
A random urinary calcium-creatinine ratio has been used to screen for hypercalciuria in lieu of a 24-hour collection in children with hematuria or nephrolithiasis. Unfortunately, this test does not consistently correlate with the 24-hour urinary calcium. In addition, there is no role for this test in the evaluation of proximal tubule function.

62
Q

A 76-year-old man receiving maintenance HD for longer than 1 year seeks advice regarding his advanced directive.
He has been admitted to the hospital to undergo revision of his arteriovenous fistula. Except for difficulties with his arteriovenous fistula, his course on dialysis has been uneventful. His medical history includes congestive heart failure. He is ambulatory within his home but uses a wheelchair when he goes out. He is currently living in his home with his wife but has previously required skilled nursing facility support when recuperating from an exacerbation of congestive heart failure. Although he is content with his current functional status, he is uncertain whether a return to a skilled nursing facility would be acceptable.
In the event of a cardiopulmonary arrest during this hospitalization, what is the MOST likely outcome for this patient?
Select one:

A. Prolonged hospitalization
B. Unsuccessful resuscitation
C. Discharge to a skilled nursing facility
D. Discharge to home

A

The correct answer is: B
Unsuccessful resuscitation

The most likely outcome of a cardiopulmonary arrest in this patient is unsuccessful resuscitation.
A recent study of outcomes for in-hospital cardiopulmonary resuscitation (CPR) found a significantly greater risk of mortality among patients with ESRD compared with those without, although the risk of death was extremely high in both patient populations. Almost 3/4 of all patients who underwent CPR experienced in-hospital mortality. In addition, patients with ESRD were less likely to be discharged home after in-hospital CPR than those without ESRD. This study, which examined the outcome of in-hospital CPR in more than 50,000 patients with ESRD compared with more than 300,000 patients without ESRD, also found greater length of stay (LOS) among patients with ESRD who survived (mean 12 days versus 7). Patients with ESRD were younger than those without ESRD but had more comorbid conditions. Note that, although the odds ratios for in-hospital mortality and nursing home discharge for ESRD-CPR patients were statistically significant at 1.24 and 1.13, respectively, the absolute differences in risk were small, as shown below.

The following table is adapted from the study:

It is important for nephrologists to have discussions with their patients regarding goals of care and advanced directives, and to counsel patients regarding the risk of adverse outcomes in patients who require CPR. Based on a desire to avoid future care in a skilled nursing facility, some patients may choose to avoid CPR in the event of a cardiopulmonary arrest; alternatively, some would choose to accept CPR and make a decision about nursing home care if/when the issue arises. It may also be useful to point out to dialysis patients that over 10% survive CPR and return to their home.

63
Q

A 39-year-old woman who is a kidney transplant recipient presents for follow-up after a biopsy of the allograft performed for an asymptomatic increase in the serum creatinine from 1.3 to 1.7 mg/dL. She has ESRD secondary to IgA nephropathy, and her living-unrelated kidney transplant was 2 years ago. She has had an unremarkable post-transplant course without previous episodes of rejection or infection. Her kidney transplant was a 5-human leukocyte antigen (HLA) mismatch.
Her pre-transplant viral serologies were as follows:

              CMV IgG	EBV IgG Donor	  Positive	Negative Recipient  Negative	Negative

Her immunosuppression regimen includes tacrolimus 4 mg twice daily, mycophenolic acid 360 mg three times daily, and prednisone 5 mg daily.
The kidney allograft biopsy performed 1 week ago showed acute tubular injury with epithelial vacuolization. No tubulitis was identified. There was no evidence of IgA deposition or mesangial expansion. C4d staining was negative. Allograft ultrasound performed at the time of the biopsy was normal.
Her serum creatinine is 1.9 mg/dL (reference range, 0.5–1.1), and she has a urine protein-to-creatinine ratio on a random specimen of 0.7. Tacrolimus trough level is 7 ng/mL (reference range for therapeutic trough level >3 months post-transplant, 5–10).
Which of the following is the MOST appropriate next step in her management?

Select one:

A. Replace tacrolimus with sirolimus
B. Increase the dose of mycophenolic acid and stop tacrolimus
C. Replace tacrolimus with belatacept
D. Stop tacrolimus
E. Decrease tacrolimus dose

A

The correct answer is: E
Decrease tacrolimus dose

This patient has evidence of allograft dysfunction due to calcineurin-inhibitor (CNI) toxicity and her dose of CNI should be reduced.
CNIs are effective immunosuppressive agents that block the calcineurin signal required for the activation of T-cells. The incorporation of CNIs into immunosuppressive regimens post-transplantation was associated with a significant reduction in acute rejection rates. However, CNIs have significant toxicities including kidney toxicity. CNIs cause vasoconstriction and acute tubular injury with vacuolization. Thrombotic microangiopathy can also occur. Chronic injury appears as atrophy and fibrosis of the interstitium in a striped pattern (“striped fibrosis”), often with prominent medial hyalinosis of the arterioles. The initial management strategy consists of dose reduction of the CNI. If this is unsuccessful, conversion to an alternative immunosuppressive is indicated. Note that trough levels provide reasonable targets for CNI dosing, but patients remain at risk for both rejection and toxicity even when levels are maintained within target range.
Conversion to belatacept is an attractive strategy for patients with CNI toxicity, but patients who are EBV-negative are not considered candidates because of the higher risk of post-transplant lymphoproliferative disorders.
Conversion to sirolimus, a mammalian target of rapamycin inhibitor, could be a reasonable strategy as well, although recent trials have reduced the enthusiasm for the use of this class of drugs based on the higher risk of rejection and alloantibody generation after conversion. In particular, patients with >500 mg/d of proteinuria or estimated GFR <40 mL/min/1.73 m2 are considered poor candidates for conversion based on published trials. Because this patient has significant proteinuria, she would not be a candidate for mTOR inhibitor conversion.
Discontinuation of tacrolimus would risk acute rejection as shown in recent randomized studies with tacrolimus withdrawal. Similarly, stopping tacrolimus even with adjustment of mycophenolic acid dose is not an effective strategy for preventing rejection.

64
Q

A 45-year-old woman with a history of hypertension is admitted to the ICU after a motor vehicle accident in which she incurred abdominal injuries and a fractured femur. Prior to arrival in the ICU, she underwent surgery for repair of a lacerated liver and a perforated ileum.
Over the past 48 hours, her BP has improved and she no longer requires vasopressors, but her cumulative fluid balance is 5 L positive. Her urine output is 40 mL/h. She is intubated and ventilated, with a fraction of inspired oxygen (FiO2)of 50% and a positive end-expiratory pressure of 10 cm H2O. Her IV fluids, including medications, have been reduced to a total of 40 mL/h, and she is receiving enteral nutrition at 10 mL/h by nasogastric tube. Current medications are morphine sulfate as needed for pain and sedation, midazolam as needed for sedation, piperacillin-tazobactam, and omeprazole.
Examination reveals BP 112/62 mm Hg and heart rate 95/min. There is no elevation of jugular venous pressure. Heart, lung, and peripheral vascular exams are normal with the exception of mild lower extremity edema. Abdomen is non-distended but hypoactive with mild diffuse tenderness. There is no pulmonary edema on chest x-ray.

Result Reference Range
Sodium 136 mEq/L 136–145
Potassium 5.2 mEq/L 3.5–5.0
Chloride 102 mEq/L 98–106
Total CO2 16 mEq/L 23–28
BUN 70 mg/dL 8–20
Creatinine 4.1 mg/dL (3.9 yesterday) 0.5–1.1

Arterial blood gas on 50% FiO2
pH 7.32 7.38–7.44
PaCO2 32 mm Hg 35–45
PaO2 90 mm Hg 80–100

Which of the following interventions is MOST appropriate for this patient?
Select one:
A. Sodium bicarbonate
B. Continuous renal replacement therapy
C. Hemodialysis
D. Sodium polystyrene sulfonate
E. Adjust drug dosing for AKI

A

The correct answer is: E
Adjust drug dosing for AKI

This patient with severe AKI should have medications doses decreased as needed to account for decreased renal clearance. Neither morphine nor midazolam are preferred in the setting of AKI, and both should be discontinued in favor of alternatives that are less likely to cause toxicity due to prolonged half-life or accumulation of toxic metabolites. Fentanyl and hydromorphone are the preferred potent parenteral narcotic analgesics. Lorazepam is the preferred benzodiazepine, although frequent dosing or infusion can result in accumulation of the diluent, propylene glycol.
Sodium polystyrene sulfonate has been associated with acute intestinal injury; its safety and efficacy in this situation is not established, and it should be avoided. The role of newer potassium binding agents remains to be established, but caution would be warranted in this patient who has had recent bowel surgery. Furthermore, this patient’s potassium level is only just above normal, and she is nonoliguric, so enteral binding of potassium is not indicated.
The role of sodium bicarbonate to correct mild to moderate metabolic acidosis in critically ill patients has not been established. Although normalization of pH may have beneficial effects, including improvement in vascular tone and responsiveness, there are multiple potential adverse consequences to sodium bicarbonate infusion including hypernatremia, volume overload, and paradoxical intracellular acidosis, among others. Most clinical investigations have addressed the use of bicarbonate in patients with more severe metabolic acidosis manifested by pH <7.2. The BICAR-ICU trial found that correction of pH with 4.2% sodium bicarbonate solution to a target of 7.3 from a starting point of <7.2 was beneficial in a subgroup of patients with moderate to severe AKI. In this patient, whose pH is 7.32, sodium bicarbonate infusion would not be appropriate.
Although there is emerging evidence that accumulation of extracellular fluid during critical illness is correlated with mortality, current evidence does not support early initiation of either HD or continuous renal replacement therapy. Likewise, the mild hyperkalemia and metabolic acidosis do not mandate initiation of renal replacement, and current evidence favors a conservative approach.
A large, multicenter, randomized trial with 620 patients with severe AKI was published in 2016, comparing an early versus late strategy for initiation of dialysis in the ICU. AKI was defined as an increase in serum creatinine >3 times baseline or an absolute increase to >4 mg/dL or oliguria for >24 hours. Patients in the early group were initiated once these criteria had been met, whereas in the late group, traditional indications for acute dialysis (refractory volume overload, acidosis, or hyperkalemia) were used. Overall, there was no difference in mortality between the 2 groups. Patients in the early initiation group were more likely to develop hypophosphatemia, and there was a higher incidence of catheter-related infections. Almost half of the patients in the late initiation group never required dialysis. Patients in the delayed initiation group had an earlier diuresis, an early marker of kidney function improvement.
A subsequent single-center study involving 231 patients with AKI did show a mortality benefit for early dialysis initiation in the ICU. However, almost all patients required renal replacement therapy, and the difference in time to initiation was <24 hours, suggesting that there may have been differences between the groups at baseline that accounted for the noted mortality benefit. A subsequent meta-analysis including >1700 patients found no improvement in mortality or need for long-term dialysis in patients undergoing an early initiation strategy. Thus, the best evidence currently is that an early initiation strategy does not confer any benefit to the patient and may be associated with several adverse outcomes.

65
Q

A 45-year-old woman with a history of morbid obesity, hypertension, and obstructive sleep apnea is referred for evaluation of proteinuria. She is asymptomatic, and her only medication is amlodipine 5 mg daily.
Her examination is notable for a BP of 136/86 mm Hg. She is obese (body mass index 42 kg/m2), and she has mild bilateral lower extremity pitting edema.
Laboratory studies reveal a serum creatinine of 1.1 mg/dL (reference range, 0.5–1.1). Serologic testing is normal. By 24-hour urine collection, her creatinine clearance is 101 mL/min, and her urine protein excretion is 2.8 g/24 h. Kidney ultrasound shows bilateral enlarged kidneys with normal corticomedullary differentiation.
A kidney biopsy is performed and reveals glomerulomegaly, mesangial expansion, podocyte hypertrophy, and focal and segmental glomerulosclerosis affecting <25% of glomeruli.
Lisinopril is initiated and the patient is referred for bariatric surgery.
Which of the following is the MOST likely consequence of sustained weight loss in this individual?

Select one:
A. Increased creatinine clearance
B. Decreased proteinuria
C. Increased levels of leptin
D. Decreased risk of nephrolithiasis
E. Decreased levels of adiponectin

A

The correct answer is: B
Decreased proteinuria

Sustained weight loss is most likely to decrease proteinuria by >25%.
Obesity is an independent risk factor for CKD. The earliest manifestation of obesity-related glomerulopathy is hyperfiltration, which has been attributed to an increase in renal plasma flow and afferent arteriolar vasodilation. This results in an increased GFR, glomerular hypertension, and subsequently proteinuria. Glomerulomegaly is almost universal in patients with obesity-related glomerulopathy, along with focal and segmental glomerulosclerosis, mesangial expansion, and podocyte hypertrophy. In addition to glomerular changes, obesity and hyperfiltration are associated with an increase in sodium reabsorption in the proximal tubule; this increase can contribute to hypertension and in turn increase glomerular hyperfiltration. The increase in proximal reabsorption of sodium may decrease delivery to the macula densa, which may result in afferent arteriolar vasodilation and a further increase in glomerular filtration.
For patients with morbid obesity, bariatric surgery can result in sustained weight loss and a number of beneficial effects on the kidney. One study found a 33% reduction in proteinuria after 6 months among patients with morbid obesity who lost weight compared with no reduction in patients who did not lose or who gained weight. GFR is likely to decline after weight loss because intraglomerular pressure and hyperfiltration decrease.
Obesity is an independent risk factor for kidney stones. Although weight loss may decrease stone risk to some extent, this is balanced by the relative increase in kidney stone risk in patients who have bariatric surgery that results in malabsorption; these procedures increase the risk of calcium oxalate stones from enteric hyperoxaluria. Thus, there is no evidence that weight loss will reduce kidney stone risk for these patients.
Leptin, a hormone produced in adipose tissue, has been associated with increased fibrogenesis in the kidney (via transforming growth factor-β) and promotes proximal tubular sodium reabsorption, which may contribute to hyperfiltration, as detailed above. Adiponectin deficiency leads to podocyte effacement and fusion in mice, which is reversed by administration of the hormone. Lower levels of adiponectin have been associated with insulin resistance, cardiovascular disease, and glomerular injury in humans. Patients with morbid obesity have low levels of adiponectin and elevated leptin. Bariatric surgery has been shown to decrease leptin and increase adiponectin.

66
Q

A 41-year-old woman with a history of allogeneic stem cell transplant for chronic lymphocytic leukemia is admitted to the ICU. Two weeks prior to admission she was treated with azithromycin for a respiratory tract infection but failed to improve. Now, she presents with fever, productive cough, generalized weakness, and progressive dyspnea. Medications on admission include tacrolimus, fluconazole, acyclovir, and omeprazole.

On examination, she is awake but lethargic. Her BP is 64/44 mm Hg, heart rate 102/min, and temperature 38.9°C. Her examination is notable for coarse crackles in the right lung field. A chest x-ray reveals dense right-sided consolidation.

Result Reference Range
WBC count 22,000/µL (92% neutrophils) 4000–11,000

Sodium 134 mEq/L 136–145
Potassium 5.8 mEq/L 3.5–5.0
Chloride 100 mEq/L 98–106
Total CO2 12 mEq/L 23–28
BUN 60 mg/dL 8–20
Creatinine 1.8 mg/dL 0.5–1.1
Ionized calcium 0.9 mmol/L 1.12–1.23
Lactic acid 8.5 mmol/L 0.7–2.1

Arterial blood gas on 25% FiO2
pH 7.33 7.38–7.44
PaCO2 19 mm Hg 35–45
PaO2 100 mm Hg 80–100

Which of the following fluid solutions is the BEST choice for initial resuscitation in this patient?

Select one:

A. 0.9% saline
B. Lactated Ringers
C. 4% albumin
D. 5% dextrose with sodium bicarbonate
E. 6% hydroxyethyl starch

A

The correct answer is: B
Lactated Ringers

Among these choices, the most appropriate IV fluid for initial resuscitation of this patient with sepsis is lactated Ringers.
In patients with sepsis, effective circulating volume can be severely reduced, and significant amounts of IV fluid may be required to improve intravascular volume. The best choice of fluid remains controversial. There are three major types of replacement fluids:
1. Crystalloid solutions (this includes saline solutions and buffered solutions such as Ringer’s lactate or sodium bicarbonate-containing solutions)
2. Colloid solutions (this includes albumin-containing solutions, which may also have significant sodium chloride content, and fluids with hyperoncotic synthetic starches, such as hydroxyethyl starch, gelatin, or dextrans)
3. Blood products or substitutes
When crystalloid fluids are infused, the fluid distributes in the extracellular space between the vasculature and the interstitium as governed by capillary Starling forces. There has been growing interest in balanced solutions, such as lactated Ringers, which have lower and more physiologic concentrations of chloride. Hyperchloremic solutions including 0.9% saline have been shown to cause or worsen metabolic acidosis and are associated with a reduction in renal perfusion. Balanced solutions have a neutral or favorable effect on acid-base equilibrium and recent trials have suggested a reduction in mortality, need for renal replacement therapy, and persistent renal impairment with the use of balanced solutions in critically ill patients. Although balanced solutions contain physiologic amounts of potassium (for example 3mEq/L), they have not been shown to cause or worsen hyperkalemia.
Use of colloid-containing fluids would theoretically limit transudation of fluid into the interstitium because of the high oncotic pressure of the colloid solution. A large randomized trial (SAFE [Saline versus Albumin Fluid Evaluation] Study) comparing saline with albumin solutions for resuscitation found no difference between the 2 groups in terms of overall survival and the incidence of AKI. In the subgroup of patients with head injury, there was an increased mortality risk with the use of albumin. Although there is no proven clinical advantage of crystallioid over albumin in patients with sepsis, significantly lower cost and better availability make saline the most appropriate choice. In addition, most currently available albumin solutions also have sodium concentrations in the 140 mmol/L range.
Pentastarch and hydroxyethyl starch synthetic are colloid solutions intended to limit the development of edema related to volume resuscitation. Both are associated with an increased risk of mortality in patients with sepsis compared with crystalloid solutions. In addition, hydroxyethyl starch is associated with an increased risk for AKI and the need for renal replacement therapy. Thus, these colloid solutions should be avoided in patients with sepsis.
The role of IV bicarbonate for the correction of metabolic acidosis in sepsis is an area of active investigation. Bicarbonate administration is associated with a number of potential harms in patients with sepsis, including paradoxical acidosis related to increased arterial and tissue pCO2, hypernatremia, volume overload, and lower ionized calcium levels. Rebound alkalemia can also occur when potential bicarbonate is converted to bicarbonate as the underlying cause of the elevated anion gap is addressed. Possible benefits include improved vascular and cardiac responsiveness. Correction of severe acidosis (<7.2) in a subgroup of patients with moderate or severe AKI was found to be beneficial in the BICAR-ICU trial. Patients with less severe acidosis have not been extensively studied but there is unlikely to be a major physiologic impact of mild metabolic acidosis. As a result, the use of IV bicarbonate should be reserved for those patients with severe acidosis and at least moderate AKI.
In this case, the patient has a nearly normal arterial pH, and restoring perfusion should lead to metabolism of the lactate and resolution of the metabolic acidosis. In addition, she already has a low ionized calcium level and bicarbonate therapy could exacerbate this issue.
Thus, current evidence favors the use of a buffered crystalloid for volume resuscitation in the setting of sepsis. The ideal amount of resuscitation fluid is also a topic of debate. The surviving sepsis campaign made a strong recommendation for aggressive early volume expansion (30 ml/kg) but there is increasing concern about the potential for detrimental effects of fluid overload and a call for more judicious volume administration.

67
Q

A 48-year-old man receiving maintenance HD complains of painful nodules in his left lower abdomen. He has been receiving dialysis for 8 years for ESRD due to diabetic nephropathy. He recently had osteomyelitis of his left great toe. Evaluation at that time included magnetic resonance imaging with gadolinium and a bone biopsy.
Medications include labetalol, cinacalcet, calcitriol, renal vitamin, insulin, and ciprofloxacin.
The patient is in visible discomfort. His BP is 170/95 mm Hg, heart rate 80/min, temperature 36.8°C, and body mass index 45 kg/m2.
Abdominal examination reveals 2 firm, exquisitely tender, subcutaneous plaques of approximately 8 cm diameter with an overlying violaceous hue.
The patient’s symptoms are managed with analgesics. One month later, one of the lesions ulcerates and forms a black eschar. A punch biopsy is performed at the margin of the lesion, which subsequently heals poorly. Histopathologic findings include calcification of dermal arterioles and surrounding fat necrosis.

Laboratory data include the following:

Result Reference Range
PTH 400 pg/mL 130–585 (dialysis patients only)
Calcium 9.5 mg/dL 8.6–10.2
Phosphorus 6 mg/dL 3–4.5
Albumin 3.2 g/dL 3.5–5.5

Which of the following is a RISK factor for the etiology of these abdominal lesions?

Select one:

A. Gadolinium
B. Cinacalcet
C. Male sex
D. Ciprofloxacin
E. Calcitriol

A

The correct answer is: E
Calcitriol

These abdominal lesions are caused by calciphylaxis, for which calcitriol has been identified as a risk factor.
Calciphylaxis is also known as calcific uremic arteriolopathy (CUA). This condition has been reported in 1% of patients with ESRD and rarely in patients with CKD or without kidney disease. The disease is characterized by extremely painful, violaceous plaques or nodules involving the skin and subcutaneous fat, which typically ulcerate and form eschars. There may be purpuric lesions that radiate from the areas of the nodules. Poor healing after biopsy is also a notorious feature. Historically, distal calcification of the toes and fingers leading to digital gangrene or acral involvement of the nose or penis was commonly observed. In recent years, it is more common to observe involvement of more central areas of adiposity including the abdomen, buttocks, breasts, and thighs.
Often the diagnosis can be established on clinical grounds. When performed, biopsies of the plaques show calcification (calcium deposition, usually composed of calcium and phosphorus) in the media and intima of small- and medium-sized blood vessels that lie within the subcutaneous fat and dermis. Inflammation of the surrounding fat (panniculitis) and fat necrosis can also be seen. The calcium deposition in small arteries and arterioles is often followed by intimal hyperplasia with endothelial fibrosis that can narrow the vessel lumen leading to ischemia in more severe lesions. Although the pathogenesis is not well understood, one potential cause may be the stimulation by uremic toxins of vascular smooth muscle cells causing differentiation into osteoblast-like (bone) cells that deposit bone matrix.
Because calciphylaxis is a rare disease, well-designed studies addressing risk factors and treatment are lacking. Factors associated with development of the lesions include obesity, female sex, long dialysis vintage, diabetes mellitus, insulin injections, hypercoagulable states (especially with warfarin use), autoimmune diseases, liver disease, and malignancy. Mineral metabolism disorders such as high levels of serum calcium, phosphate, and PTH have been implicated as well as an elevated calcium-phosphorus product, low serum albumin, and elevated alkaline phosphate levels. Medications associated with development of CUA include corticosteroids, warfarin, calcitriol, calcium-containing supplements and phosphate binders, and IV iron. CUA may be present without any of these laboratory abnormalities or risk factors.
Cinacalcet has not been implicated and is often used to reduce PTH levels and improve mineral metabolism. First generation gadolinium contrast agents, such as gadodiamide, were occasionally associated with nephrogenic systemic fibrosis (NSF), a skin condition characterized by thickened skin, brawny plaques, and pain. In this condition, there is fibrosis on skin biopsy but no arteriolar calcification. Newer formulations of gadolinium, such as gadoteric acid, appear to have minimal risk for inciting NSF. Ciprofloxacin and other antibiotics have not been implicated in CUA.

68
Q

A 48-year-old woman presents to the emergency department with gross hematuria. She also reports severe headaches, diaphoresis, and palpitations that coincide with micturition. These symptoms have progressed over the past 4–6 weeks to the point that she now purposefully “holds her urine” as long as she possibly can. She has not experienced fevers, chills, or flank pain. Prior to these symptoms, she felt completely well. She has no history of urinary tract infections, kidney stones, headaches, or hypertension and takes no medications. Her father has hypertension that is controlled with a thiazide diuretic. Her mother is well.
On examination, immediately after she has provided a urine sample, she appears very anxious. Her heart rate is 120/min and regular; BP is 180/90 mm Hg. The eyes are normal and the thyroid gland is not palpable. The heart is tachycardic, regular, and without extra sounds. The abdomen is soft and nontender and without appreciable mass, and there is no costovertebral angle tenderness. A fine tremor is present, but the neurologic examination is otherwise normal.
Blood tests reveal normal kidney function and electrolytes. The random blood glucose is mildly elevated. Urinalysis reveals 4+ blood and 1+ protein and is negative for leukocyte esterase.
Microscopic examination of the urine reveals isomorphic RBCs that are too numerous to count. No WBCs or casts are detected.

Which of the following options represents the MOST likely explanation for this patient’s symptoms?

Select one:
A. Renal tuberculosis
B. Transitional cell carcinoma of the bladder
C. Interstitial cystitis
D. Paraganglioma of the bladder
E. Loin pain-hematuria syndrome

A

The correct answer is: D
Paraganglioma of the bladder

The most likely diagnosis for this patient with gross hematuria and post-micturition anxiety, headaches, palpitations, and hypertension is a paraganglioma of the bladder.
Although pheochromocytoma (pheo) has been termed the “10% tumor,” because there are many features that are said to be present in ~10% of cases, the location is extra-adrenal up to 25% of cases and hereditary in 30%. Pheos that arise outside the adrenal glands are referred to as paragangliomas. In this discussion, the term “pheo” refers to both adrenal and extra-adrenal forms. Pheos arise from neural crest cells, which typically develop into sympathetic and parasympathetic paraganglia throughout the body. Approximately 95% of these tumors are benign. Like the adrenal medulla, pheos may secrete catecholamines, including norepinephrine, epinephrine, and dopamine. Presenting symptoms depend on whether the catecholamine secreted is predominantly norepinephrine, which is most common, or epinephrine. The secretion of dopamine is least common and may correlate with a higher tendency to be malignant. Norepinephrine binds to α-receptors, which leads to vasoconstriction and hypertension. Epinephrine typically exerts its effects on β-2 receptors, leading to vasodilation and orthostatic hypotension. Paroxysmal hypertension is a classic feature of pheos, but sustained hypertension may be equally common. Other characteristic symptoms and signs include angina, anxiety, nausea, hyperglycemia, tremor, and weight loss. Close to half of all patients with pheos may not have hypertension, and some may even be completely asymptomatic.
The metabolites of norepinephrine and epinephrine are termed metanephrines and are more easily detected in plasma and urine studies than the catecholamines themselves. As such, plasma free metanephrines and urinary fractionated metanephrines are laboratory tests with excellent sensitivity and specificity for the detection of pheos. Measurement of urinary vanillylmandelic acid, another metabolic product of catecholamine, lacks the sensitivity of metanephrine testing but has good specificity. CT and MRI are both excellent initial imaging tests, although the latter may be better for detecting metastatic disease. 123I-metaiodobenzylguanidine (MIBG) scintigraphy is a functional test that may also be helpful in excluding metastatic disease or identifying an occult pheo not visualized with conventional imaging. Given the high frequency of hereditary tumors, genetic testing for mutations is warranted.
In this case, the patient’s classic symptoms suggest the diagnosis of paraganglioma of the bladder. Micturition leads to shrinking of the bladder, which stimulates the tumor and results in secretion of catecholamines. The bladder tumor can also explain the gross hematuria. The absence of pyuria argues against a urinary tract infection such as renal tuberculosis. Interstitial cystitis can also lead to severe pelvic symptoms, including pelvic pain, frequency, and urgency, but does not cause hypertension, anxiety, or gross hematuria. Loin pain-hematuria syndrome can be associated with gross or microscopic hematuria, but the most common symptom, flank pain, is lacking. Transitional cell carcinoma might explain the hematuria but does not account for the other aspects of her presentation.

69
Q

A 45-year-old man presents to the emergency department with a 2-week history of severe back pain. The pain is in the lower back and is described as sharp and lancinating, with radiation to the left buttock and calf when standing upright. He has been taking acetaminophen 500 mg three times daily and ibuprofen 800 mg four times daily for 2 weeks without relief. He has a history of hypertension and diastolic heart failure, controlled with chlorthalidone 25 mg daily and lisinopril 10 mg daily. Initial laboratory testing reveals a serum creatinine level of 2.9 mg/dL, increased from 1.2 mg/dL at a routine visit 3 months ago.
On examination, BP is 168/95 mm Hg, and heart rate is 75/min and regular. There is no costovertebral angle tenderness. No rash or edema is appreciated. There is tenderness in the lumbar region (L3/L4), and when the straight leg test is performed, the pain is reproduced.
Laboratory data include the following:

Result Reference range
Sodium 139 mEq/L 136–145
Potassium 5.3 mEq/L 3.5–5.0
Chloride 100 mEq/L 98–106
Bicarbonate 17 mEq/L 23–28
BUN 45 mg/dL 8–20
Creatinine 2.9 mg/dL 0.7–1.3

Urinalysis: brown color, specific gravity 1.014, 1+ protein, moderate blood. Microscopy reveals 2–4 isomorphic RBCs/HPF, 1–2 WBCs/HPF, and 8–12 granular casts per low-power field.

Kidney ultrasound shows that the right kidney is 10.5 cm and the left kidney is 11 cm. There is no hydronephrosis or nephrolithiasis; post void residual is 15 mL

Which of the following MOST accurately describes the mechanism of AKI in this case?
Select one:

A. Inhibition of prostaglandin-mediated afferent arteriolar vasodilation
B. Disruption of the glomerular basement membrane by T-lymphocytes
C. Medullary ischemia caused by diversion of blood to the renal cortex
D. Infiltration of the interstitium by T-lymphocytes
E. Prostaglandin-mediated potentiation of antidiuretic hormone

A

The correct answer is: A
Inhibition of prostaglandin-mediated afferent arteriolar vasodilation

Inhibition of prostaglandin-mediated afferent arteriolar vasodilation is the most likely cause of this patient’s AKI.
Prostaglandins are produced by the enzyme cyclooxygenase and can contribute to both local and systemic pain and inflammation. Nonsteroidal anti-inflammatory drugs (NSAIDs) are cyclooxygenase inhibitors that are widely used to treat these symptoms. Under normal circumstances, this is well tolerated because renal vasodilatory prostaglandins are not necessary to maintain renal perfusion and glomerular filtration. However, when conditions such as heart failure or volume depletion are present, renal vasodilatory prostaglandins are needed to offset the vasoconstriction mediated by angiotensin II, norepinephrine, vasopressin, and endothelin. Inhibition of prostaglandin synthesis in this setting can lead to a decline in renal perfusion and AKI that may be functional or parenchymal. This patient’s underlying heart failure, use of a diuretic and angiotensin converting enzyme inhibitor (which limits angiotensin II-mediated efferent arteriolar vasoconstriction, a key component in glomerular autoregulation) all predispose to a dependence on renal prostaglandin action to maintain renal perfusion. His use of ibuprofen inhibits afferent arteriolar vasodilation, resulting in reduced renal perfusion and ischemic injury. Indeed, there is evidence of acute tubular necrosis (ATN) based on the urine sediment.
T-lymphocytic interstitial infiltration is observed in cases of acute interstitial nephritis (AIN) associated with NSAID use. Typically, the onset of symptoms is weeks to months after initial exposure to NSAIDs. Although some cases feature pyuria and systemic manifestations such as eosinophilia, fever and rash, many cases present with bland urine sediment and an absence of systemic signs and symptoms. Although AIN cannot be excluded and many studies have confirmed that AIN cannot be diagnosed reliably by any technique other than kidney biopsy, the urine sediment in this case suggests acute tubular necrosis rather than AIN.
Medullary ischemia from diversion of blood flow is the proposed mechanism of papillary necrosis, a rare complication of NSAID use, especially when combined with acetaminophen and caffeine. This diagnosis is classically associated with phenacetin but has also been reported with chronic ibuprofen exposure, typically over years. The radiographic features of papillary necrosis include small kidneys with irregular contours. On ultrasound, the collecting system may appear dilated or mishapen because of the abnormal papillary architecture or obstruction from a sloughed papilla.
Nephrotic syndrome is associated with NSAID use, and both minimal change disease and membranous nephropathy have been reported. The mechanism underlying glomerular pathology is not clear but appears to be a consistent risk across all classes of NSAIDs. The time course is similar to NSAID-associated AIN, with a latency of weeks to months after first exposure.
NSAIDs potentiate the effects of antidiuretic hormone and impair water excretion. This effect would not directly change glomerular filtration rate, and in this case no dysnatremia is evident.

70
Q

An 83-year-old man is hospitalized for AKI associated with bladder outlet obstruction. Four days ago, he presented to the emergency department with nausea and abdominal pain and was found to have severe bladder distention and bilateral hydronephrosis attributable to prostatic enlargement. Creatinine was 4.5 mg/dL at the time of presentation. A urinary catheter was inserted on admission and remains in place. On the second hospital day, the patient became disoriented and agitated.
On examination today, the patient’s temperature is 37.5°C. BP is 145/88 mm Hg with pulse 87/min. He is alert, anxious-appearing, and intermittently yelling for help. He is oriented to self only. Cardiopulmonary examination findings are unremarkable. His abdomen is soft with normoactive bowel sounds. There is no tenderness to palpation or guarding. He is moving all extremities but is not following directions.
Current laboratory test results include the following:

Result Reference Range
Sodium 143 mg/dL 136–145
Potassium 3.6 mg/dL 3.5–5.0
BUN 32 mg/dL 8–20
Creatinine 1.6 mg/dL 0.7–1.3
Venous blood gas
pH 7.36 7.32–7.41
pCO2 38 mm Hg 42–53
Bicarbonate 19 mg/dL 24–28

Urinalysis (2 days ago): cloudy, pH 5, specific gravity 1.021, 1+ leukocyte esterase, negative nitrite, and 1+ protein. Microscopy identified 25 WBCs/HPF, 6 RBCs/HPF, and 2+ bacteria.

Urine culture from that specimen yielded 10,000 colony-forming units (CFU)/mL of Enterococcus sp.

Which of the following findings are MOST consistent with a catheter-associated urinary tract infection (UTI) in this patient?

Select one:
A. Hematuria
B. Pyuria
C. Proteinuria
D. Urine culture with 104 CFU/mL bacteria
E. Cloudy appearance of urine

A

The correct answer is: D
Urine culture with 104 CFU/mL bacteria

According to the International Practice Guidelines from the Infectious Diseases Society of America (IDSA), a catheter-associated urinary tract infection (CAUTI) is defined by the presence of ≥103 CFU/mL of pathogenic bacteria and symptoms that are compatible with a UTI. In this patient, there is a low-grade fever and altered mental status accompanied by a urine culture with 10,000 (104) CFU/mL of bacteria, above the 103 CFU/mL threshold, so a CA-UTI is likely.
The diagnosis of CAUTI is challenging due to frequent colonization and almost universal pyuria that develops in the setting of an indwelling urinary catheter. Diagnosis and treatment of CAUTI based on pyuria and bacteriuria in the absence of symptoms is discouraged, as unnecessary antibiotic therapy can contribute to formation of antibiotic resistance as well as complications such as C. difficile colitis. Most cases of CAUTI will demonstrate high colony counts, but defining a relatively low bacterial count threshold for CAUTI is embraced by the IDSA because peri-urethral contamination is less likely in patients with catheters. In contrast, patients with condom catheters may have high concentrations of bacteria, which reflect skin contamination rather than an established UTI. In this case, a higher threshold of 105 CFU/mL is used to diagnose catheter-associated asymptomatic bacteriuria. Because patients with indwelling catheters typically have bacterial colonization, the culture result alone is insufficient to establish the diagnosis of CAUTI, and the diagnosis is contingent on both urine culture and the presence of clinical signs and symptoms. Symptoms of UTI in a patient with an indwelling catheter may include those typical of UTI, such as suprapubic and/or flank pain and tenderness, but in some cases symptoms may be limited to systemic features such as fever, altered mental status, or hypotension.
The presence or degree of pyuria does not help distinguish CAUTI from asymptomatic bacteriuria in a catheterized patient. The association between pyuria and CAUTI is strongest in gram-negative infections and less reliable in those caused by gram-positive cocci or yeast.
Proteinuria and albuminuria are commonly detected in the setting of a symptomatic UTI. There are multiple potential reasons for this finding. Fever can cause transient proteinuria, and alkaline urine from an infection with a urease-producing organism can cause a false-positive result for albumin by dipstick. In addition, patients with recurrent UTIs may have structural parenchymal abnormalities associated with secondary focal and segmental glomerulosclerosis. Proteinuria in the setting of a UTI may require additional attention once the infection has resolved, but itself is nonspecific and does not confirm the diagnosis of CAUTI.
Hematuria is commonly observed in patients with indwelling urinary catheters and does not distinguish UTI from asymptomatic bacteriuria or other causes of hematuria such as trauma associated with the catheter.
Urine appearance or odor is not helpful in diagnosing UTI in patients with catheters.

71
Q

A 35-year-old woman has stage 5 CKD due to autosomal dominant polycystic kidney disease (ADPKD) but has not yet initiated dialysis. She has recently been accepted for kidney transplantation. Several potential donors have been identified, all of whom have a compatible blood type and are healthy without evidence of kidney disease. Her Panel of Reactive Antibodies (PRA) test is 0%. The human leukocyte antigen (HLA) match profile of her potential donors is as follows:
Potential Donor Age (years) # of HLA mismatches
Father 65 1
Cousin 25 6
Uncle 60 2
Sister 38 3

Assuming all potential donors are equally willing and able to donate a kidney, which of the following options would provide this patient with the BEST chance for a long-lasting kidney transplant?

Select one:
A. Living donation from her father
B. Living donation from her cousin
C. Living donation from her uncle
D. Deceased donation with 0 HLA mismatches
E. Living donation from her sister

A

The correct answer is: E
Living donation from her sister

This patient with stage 5 CKD should receive a transplant from her 38-year-old sister with 3 HLA mismatches.
The following donor characteristics are important in determining long-term graft outcome: donor-recipient size match, donor age, HLA match, and biological first-degree relationship of donor and recipient. A kidney transplant from a younger donor is associated with a significantly longer expected duration of function, which is of particular importance to younger recipients. The novel allocation policy in the United States has attempted to maximize kidney life-years by matching younger donors (estimated post-transplant survival (EPTS) score <20%) with better quality kidneys (kidney donor profile index (KDPI) score <20%).
A lower degree of HLA mismatching is associated with long-term graft survival, although not a lower rate of early rejection. Long-term allograft survival is best in HLA-identical transplants, particularly living related. Long-term allograft survival is worst in randomly matched cadaver kidneys. Therefore, it is reasonable to attempt to select a living-related donor for this patient, which will take into account both the age of the donor and the HLA match. Among the potential donors, her sister offers the best combination of age and HLA match. Although her father has a better HLA match, this advantage is negated by his relatively older age. Even though her cousin is younger, the higher degree of HLA mismatch will negatively affect the survival of the allograft.

72
Q

A 58-year-old man presents for evaluation of a 6-week history of fever, weight loss, abdominal pain, and rash. His medical history is significant for hypertension and hyperlipidemia. Current medications are lisinopril 20 mg daily and atorvastatin 20 mg daily.
Physical examination reveals BP 162/97 mm Hg, mild peripheral edema, and palpable purpura affecting the lower limbs.
Laboratory data include the following (attached):

Renal arteriogram demonstrates microaneurysms and segmental occlusion in small renal arteries.

Which ONE of the following is MOST appropriate initial treatment for this patient?

Select one:
A. Prednisone and cyclophosphamide
B. Prednisone and tenofovir
C. Prednisone and methotrexate
D. Prednisone and plasma exchange
E. Pulse methylprednisolone and prednisone taper

A

The correct answer is: A
Prednisone and cyclophosphamide

The preferred initial therapy for polyarteritis nodosa (PAN) is the combination of cyclophosphamide and corticosteroids.
PAN is a rare systemic vasculitis involving medium-sized arteries and results in aneurysms, stenosis, occlusion, and potentially rupture of the involved vessel and distal ischemia. The diagnosis is established by the clinical findings and characteristic imaging abnormalities. Arteriography is considered the gold standard for imaging, but CT and MRI may also be useful. Complement levels are typically normal and anti-neutrophil cytoplasmic antibody (ANCA) is usually negative (in contrast, ANCA testing is often positive in cases of small vessel vasculitis). PAN may be associated with hepatitis B or C infection or hairy cell leukemia and is considered secondary if these conditions are present. Most cases in the United States are primary and are seen in middle-aged or older adults with a slight male predominance. If PAN is associated with hepatitis B or C infection, antiviral therapy is indicated. In this case, there is no evidence of a viral hepatitis. Furthermore, tenofovir disoproxil fumarate is contraindicated in the presence of AKI or advanced CKD. Because it is potentially less nephrotoxic, tenofovir alafenamide could be considered for viral hepatitis associated PAN, but would still not be first-line treatment in the setting of AKI.
PAN may affect any organ; skin, kidney, and neurologic involvement are particularly common. Skin manifestations are diverse and may include painful nodules, purpura, bullae, vesicles, or livedo reticularis. Because of the “preglomerular” location of the kidney lesion, kidney involvement is manifested by hypertension and reduced filtration. The urine sediment may be surprisingly bland without nephritic or nephrotic features. Mononeuritis multiplex occurs in up to 70% of patients, affecting both motor and sensory nerves. Patients can also have central nervous system involvement, including stroke, hemorrhage, and seizures.
Untreated PAN is associated with a high mortality, with AKI and gastrointestinal, cardiac, and cerebrovascular complications being among the most common causes of death in these patients. Treatment with corticosteroids alone is satisfactory for a mild presentation with only skin involvement. The combination of cyclophosphamide and corticosteroid therapy is recommended for more severe presentations that include AKI and is associated with a higher rate of remission and improved survival. Relapse is relatively uncommon after remission, but maintenance therapy is often provided for up to 18 months.
Plasma exchange has been used in hepatitis B-associated cases but has not been shown to be effective for primary PAN. Azathioprine and methotrexate have both been used for maintenance of remission, but neither is preferred for induction therapy; in addition, methotrexate is contraindicated in AKI.

73
Q

A 65-year-old man presents for routine kidney transplant follow-up. He had ESRD secondary to membranous nephropathy and received a kidney transplant 8 years ago. He has been well and has had no episodes of acute rejection. He has had several cutaneous squamous cell carcinomas (SCCs). Recently, he has required the removal of one or more SCC monthly. He is currently scheduled for a Mohs procedure to remove an invasive SCC on his temple.

Current medications include tacrolimus, mycophenolate mofetil, prednisone, lisinopril, and atorvastatin.
Physical examination reveals a BP of 139/62 mm Hg, heart rate 76/min. The skin exam is notable for multiple actinic lesions but the general exam and kidney allograft are normal.
His laboratory parameters include:

Result Reference Range
WBC count 6200/µL 4000–11,000
Hemoglobin 11.3 g/dL 14–18
Platelets 198,000/µL 150,000–450,000

Sodium 141 mEq/L 136–145
Potassium 4.9 mEq/L 3.5–5.0
BUN 29 mg/dL 8–20
Creatinine 1.4 mg/dL
(1.3 mg/dL at baseline) 0.7–1.3
Tacrolimus trough 4.3 ng/mL 5–10, >3 months post-transplantation

Urine protein-to-creatinine ratio 0.3 <0.2
Urinalysis is notable for trace proteinuria.

Which of the following options represents the MOST appropriate change in this patient’s immunosuppressive regimen to decrease the risk of subsequent squamous cell carcinomas?

Select one:

A. Replace tacrolimus with sirolimus
B. Discontinue mycophenolate mofetil
C. Prescribe sirolimus
D. Discontinue tacrolimus
E. Discontinue prednisone

A

The correct answer is: A
Replace tacrolimus with sirolimus

Replacement of tacrolimus with sirolimus is the most appropriate change in this patient’s immunosuppressive regimen because it is likely to decrease the incidence of further squamous cell carcinomas (SCCs).
A recent meta-analysis that reviewed 21 randomized trials comparing immunosuppressive regimens with and without sirolimus found a 40% reduction in the risk of malignancy overall and a 56% reduction in the risk of non-melanoma skin cancer in patients treated with sirolimus compared with control patients. The effect was most noticeable in patients who converted to sirolimus from an established immunosuppressive regimen. Similarly, the CONVERT trial studied patients whose maintenance therapy was either cyclosporine or tacrolimus. Patients who switched to sirolimus had a lower rate of malignancy at 12 and 24 months compared with patients who continued a calcineurin inhibitor (CNI)-based regimen. In another study of patients with a history of cutaneous SCC who switched to sirolimus, new SCC developed in 22% of patients in the sirolimus group (6 after withdrawal of sirolimus) and 39% in the CNI group. Furthermore, the time to onset of SCC was 15 months in the sirolimus group versus 7 months in the CNI group. Unfortunately, there was also a higher incidence of adverse events in the sirolimus group, including proteinuria, edema, and pneumonitis. An increase in proteinuria was also observed in the CONVERT study.
The aforementioned meta-analysis found that sirolimus use was associated with an increased risk of death (hazard ratio, 1.43). The increased risk of death was due to cardiovascular and infection-related deaths. A meta-analysis of studies of sirolimus used as primary immunosuppression after kidney transplant found no difference in mortality. Another systematic review of 29 studies found that patients who were converted from a CNI to an to a mammalian target of rapamycin inhibitor within 1 year after transplant had a higher GFR at 1 year compared with those who continued taking a CNI. However, there was no difference in graft survival, an observation also made in the CONVERT trial.
Thus, in summary, conversion from a CNI-based regimen to a sirolimus-based regimen reduces the risk of cancer but is associated with a higher risk for proteinuria, edema, and pneumonitis. Other changes in immunosuppression regimens, such as eliminating tacrolimus, mycophenolate, or prednisone have not been shown to reduce cancer incidence and would likely increase the risk for rejection. The addition of sirolimus to a CNI-based immunosuppression regimen has not been specifically addressed in recent studies.

74
Q

A 55-year-old woman with multiple sclerosis presents to her primary care physician for evaluation of bilateral flank pain. An ultrasound reveals bilateral, large stones. A consulting urologist obtains a CT scan of the abdomen that demonstrates bilateral staghorn calculi. There is partial obstruction of the right kidney. Serum creatinine is 2 mg/dL (reference range, 0.5–1.1), urine pH is 7 (reference range, 4.5–8.0), and urine culture reveals 100,000 colony-forming units (CFU)/mL of penicillin-sensitive Corynebacterium. Ampicillin is initiated.

Which of the following is the MOST appropriate next step in therapy?

Select one:
A. Tiopronin
B. Hydrochlorothiazide
C. Ciprofloxacin
D. Surgical nephrolithotomy
E. Acid irrigation of the renal pelvis (chemolysis)

A

The correct answer is: D
Surgical nephrolithotomy

Surgical removal is the best treatment option for this patient with staghorn calculi and urinary tract infection.
Struvite stones comprise only 10–15% of all stones, but they are very aggressive and frequently grow rapidly from one renal calyx to the next, resulting in a “staghorn” appearance. These stones can only form in the presence of urease-producing bacteria such as Proteus, Corynebacterium, Haemophilus, and others (but never Escherichia coli). Urease allows for the concurrent presence of both ammonium ions and a high pH in the urine. The stones are often called “triple phosphate” stones because they are formed of three cations (ammonium, magnesium, and calcium) bound to the anions phosphate and carbonate. These crystals have a “coffin lid” appearance on urine microscopy. Because the stones are large, often obstructing the flow of purulent urine, severe infections and rapid loss of kidney function can ensue, which is why they have also been termed “stone cancer.” Patients who develop struvite stones are often predisposed to kidney infections due to associated genitourinary abnormalities, neurogenic bladder, and indwelling catheters.
Recognition of struvite stones should prompt urgent urologic referral to prevent the severe morbidity that can result. Initial therapy includes treatment with an appropriate antibiotic to treat the urinary tract infection, although antibiotics are typically unable to penetrate the stones themselves. The choice of antibiotic should be based on susceptibility data and side effect profile. In this patient, there would be no immediate advantage to ciprofloxacin over ampicillin. Stones can form even when the bacterial colony count is <100,000 CFU/mL, so the microbiology lab should be instructed to identify and check the antibiotic sensitivities of all bacteria. The mainstay of therapy is urologic removal of the stones. Any infected stone fragments left in place can lead to recurrent stone formation. Once most of the stone burden is removed, antibiotics are continued until the urine is sterile, with subsequent tapering. If small fragments remain that cannot be surgically removed, chemolysis with hemacidrin, an acid substance used to dissolve calculi, can be infused via a nephrostomy tube or a catheter in the ureter. Chemolysis is not without side effects and is only considered after significant surgical debulking of the stone has been performed and if the patient is medically stable. Thiazide diuretics are used to prevent calcium stones caused by idiopathic hypercalciuria. Tiopronin may be used to limit nephrolithiasis in patients with cystinuria who form cystine stones.

75
Q

A 56-year-old woman with no past medical history is admitted to the ICU because of increasingly severe shortness of breath without chest pain, cough, or fever. Her review of systems is positive for hand swelling and stiffness, as well as Raynaud syndrome.
On examination, she appears distressed, but younger than her stated age. Her respiratory rate is 22/min, BP is 195/108 mm Hg, and heart rate is 98/min. She has jugular venous distension to the angle of the jaw when sitting at 45°. An S4 gallop is present. Crackles are present at the lung bases, and there is trace lower extremity edema. There is no synovitis present, and her skin is normal.
Laboratory results:

Result Reference range
Sodium 139 mEq/L 136–145
Potassium 4.9 mEq/L 3.5–5.0
Total CO2 22 mEq/L 23–30
Creatinine 2 mg/dL 0.5–1.1
Troponin I <0.01 ng/mL ≤0.04

Urine protein-to-creatinine ratio 0.8 g/g <0.2

Urinalysis: 2+ protein, no blood, and microscopy is unremarkable
Chest x-ray shows bilateral pulmonary edema, and kidney ultrasound reveals normal size and echogenicity. Duplex examination is negative for renal artery stenosis. Echocardiogram shows evidence of diastolic dysfunction.

Which of the following laboratory tests is MOST LIKELY to establish the diagnosis?
Select one:

A. Anti-RNA polymerase III antibodies
B. Plasma renin and aldosterone
C. B-type natriuretic peptide
D. Angiotensin-converting enzyme
E. Anti-neutrophil cytoplasmic antibodies

A

The correct answer is: A
Anti-RNA polymerase III antibodies

Measurement of RNA polymerase III antibody level is indicated in this patient with features of scleroderma renal crisis (SRC).
SRC is a life-threatening complication of scleroderma affecting 10% of patients with systemic sclerosis. SRC is characterized by the abrupt onset of hypertension and AKI. Pulmonary edema and hypertensive encephalopathy may also occur. Typically, SRC manifests within 5 years of onset of systemic sclerosis, and most patients with SRC will have characteristic skin features. However, SRC may be the presenting manifestation in as many as 20% of patients. A high index of suspicion is needed to identify these patients. As in this patient, the initial manifestations of systemic sclerosis often include Raynaud syndrome as well as swelling and stiffness of the hands. Angiotensin-converting enzyme inhibitors (ACEI) are the mainstay of therapy. As in this patient, the initial manifestations of systemic sclerosis often include Raynaud syndrome as well as swelling and stiffness of the hands. Angiotensin-converting enzyme inhibitors (ACEI) are the mainstay of therapy.
Almost all patients with systemic sclerosis (SS) have antinuclear antibodies, with the speckled and centromere patterns being most common (while a nucleolar pattern is more specific). Anti-RNA polymerase III antibodies are associated with an increased risk of SRC as well as more rapid progression of cutaneous involvement. The prevalence of these antibodies varies widely geographically and ranges from 15 to 59%. Topoisomerase antibodies (Scl-70) are present in 30% of patients with diffuse SS and absent in limited SS. Anti-centromere antibodies are present in about half of patients with limited SS. In this case, identification of anti-RNA polymerase III antibodies would strengthen the suspicion for SRC and support treatment with an ACEI.
Measurement of renin and aldosterone levels is a reasonable screening test if primary hyperaldosteronism is suspected, but this diagnosis would not explain the features of this case.
BNP is a marker of cardiac myocyte stress and can be used clinically to help distinguish patients in congestive heart failure (CHF) from patients with other causes of respiratory symptoms. In this case, CHF can be inferred from the pulmonary edema evident on physical examination and chest x-ray and the echocardiogram showing diastolic function. BNP would likely be elevated but would not help to identify the underlying cause. It is also noteworthy that BNP is renally excreted, so a high level is less specific in the setting of AKI or advanced CKD.
ANCAs are serologic markers of microscopic polyangiitis and granulomatosis with polyangiitis, among other related disorders. Although these disorders are relatively frequent causes of pulmonary-renal syndromes, they generally present with active urine sediment, systemic inflammatory signs and symptoms, and hemoptysis. The sediment in this case is bland.
ACE levels are sometimes elevated in sarcoidosis, which can demonstrate pulmonary and renal involvement. However, sarcoidosis would not explain the features of this case.